SlideShare a Scribd company logo
By
D.Rajesh Asst. Prof.
EEE Department
6 October 2020 D.Rajesh Asst.Prof. EEE Department 1
 Unit-i Electric Power Generation& Economic
Considerations
 Unit-ii Power Supply Systems& Distribution
Systems
 Unit-iii Inductance & Capacitance Calculations
 Unit-iv Performance Of Transmission Lines
 Unit-v Overhead Line Insulators
6 October 2020 D.Rajesh Asst.Prof. EEE Department 2
POWER SUPPLY SYSTEMS& DISTRIBUTION
SYSTEMS:
 Transmission and distribution systems- D.C 2-wire
and 3- wire systems, A.C single phase , three phase
and 4-wire systems, comparison of copper efficiency.
 Primary and secondary distribution systems,
concentrated & uniformly distributed loads on
distributors fed at both ends, ring distributor,
voltage drop and power loss calculation,
 Kelvin’slaw.
6 October 2020 D.Rajesh Asst.Prof. EEE Department 3
Power
Transmission
D.C
D.C. two-wire
D.C. two-wire
with mid-point
earthed
D.C. three-wire
A.C
Single-phase A.C.
system
Single-phase two-
wire
Single-phase two-
wire with mid-
point earthed
Single-phase
three-wire
Two-phase A.C.
system
Two-phase four-
wire
Two-phase three
wire.
Three-phase A.C.
system
Three-phase
three-wire
Three-phase four-
wire
6 October 2020 D.Rajesh Asst.Prof. EEE Department 4
6 October 2020 D.Rajesh Asst.Prof. EEE Department 5
Overhead
Under ground
(i) Same power (P watts) transmitted by each system.
(ii) The distance (l metres) over which power is
transmitted remains the same.
(iii) The line losses (W watts) are the same in each
case.
(iv) The maximum voltage between any conductor
and earth (𝑉
𝑚) is the same in each case.
6 October 2020 D.Rajesh Asst.Prof. EEE Department 6
Max. voltage between conductors = 𝑉
𝑚 ,
Power to be transmitted = P= 𝑉
𝑚𝐼1 1
∴ Load current, 𝐼1 = P/ 𝑉
𝑚
If 𝑅1 is the resistance of each line conductor, then,
𝑅1 = ρ l/ 𝑎1
where 𝑎1 is the area of X-section of the conductor.
copper losses in line W = 𝐼1
2
𝑅1 + 𝐼1
2
𝑅1
=2𝐼1
2
𝑅1
6 October 2020 D.Rajesh Asst.Prof. EEE Department 7
1
1
2
The maximum voltage between any conductor and earth is 𝑉
𝑚 so that maximum
voltage between conductors is 2 𝑉
𝑚. 𝑅2 is the resistance of each line conductor
Power to be transmitted
P= 𝑉
𝑚𝐼2+ 𝑉
𝑚𝐼2=2 𝑉
𝑚𝐼2
Load current, 𝐼2 = P/2 𝑉
𝑚
Since from assumption 1 the power transmitted is same
for all systems so from eq 1& 3
2𝑉
𝑚𝐼2= 𝑉
𝑚𝐼1
𝐼2 =
𝐼1
2
copper losses in line W = 𝐼2
2
𝑅2+ 𝐼2
2
𝑅2 = 2 𝐼2
2
𝑅2
From eq 4
W=2(
𝐼1
2
)2
𝑅2 =
𝐼1
2
2
𝑅2
6 October 2020 D.Rajesh Asst.Prof. EEE Department 8
3
4
5
 Compare eq 2 & 5
2𝐼1
2
𝑅1 = (
𝐼1
2
2
) 𝑅2
𝑅1
𝑅2
=
1
4
𝑎1
𝑎2
=4 (where R = ρ l/ 𝑎)
𝑎1=X-area of conductor in 2 wire dc system
𝑎2= X-area of conductor in 2 wire dc system with mid-point earthed
Then
volume of copper wire Two−wire D.C system with mid−point earthed
volume of copper wire Two−wire D.C system
=
𝑎2∗𝑙2
𝑎1∗𝑙1
=
1
4
𝑎1∗2𝑙
𝑎1∗2𝑙
=
1
4
where 𝑙1 = 𝑙2=l+l
Hence, the volume of conductor material required in Two−wire D.C system with
mid−point earthed is one-fourth of that required in a two-wire D.C system with
one conductor earthed.
6 October 2020 D.Rajesh Asst.Prof. EEE Department 9
If the load is balanced, the current in the neutral wire is zero. Assuming balanced loads,
Power to be transmitted
P= 𝑉
𝑚𝐼3+ 𝑉
𝑚𝐼3=2 𝑉
𝑚𝐼3 6
Since from assumption 1 the power transmitted is same
for all systems so from eq 1& 6
2𝑉
𝑚𝐼3= 𝑉
𝑚𝐼1
𝐼3 =
𝐼1
2
7
Let 𝑅3 is the resistance of each line conductor,
the area of X-section of neutral wire to be half that of the outer wire,
copper losses in line W = 𝐼3
2
𝑅3+ 𝐼3
2
𝑅3 = 2𝐼3
2
𝑅3 8
From eq 7
W=2(
𝐼1
2
)2
𝑅3 =
𝐼1
2
2
𝑅3 9
6 October 2020 D.Rajesh Asst.Prof. EEE Department 10
 Compare eq 2 & 8
2𝐼1
2
𝑅1 =
𝐼1
2
2
𝑅3
𝑅1
𝑅3
=
1
4
𝑅3 = ρ l/ 𝑎3 where 𝑎3 is the area of X-section of the conductor.
𝑎1
𝑎3
=4
Then
volume of copper wire Three−wire D.C
volume of copper wire Two−wire D.C system
=
𝑎3∗𝑙3
𝑎1∗𝑙1
=
1
4
𝑎1∗2.5𝑙
𝑎1∗2𝑙
=
2.5
8
where 𝑙3=l+l+0.5l
=0.3125
Hence, the volume of conductor material required in Three−wire D.C system is 0.3125
times of that required in a two-wire D.C system with one conductor earthed.
6 October 2020 D.Rajesh Asst.Prof. EEE Department 11
The maximum voltage between conductors is 𝑉
𝑚, so that r.m.s. value of voltage
between them is
𝑉𝑚
2
. Assuming the load power factor to be cos φ, load current 𝐼4
Power supplied P=
𝑉𝑚
2
𝐼4 cos φ 10
Since from assumption 1 the power transmitted is same
for all systems so from eq 1& 10
𝑉𝑚
2
𝐼4 cos φ= 𝑉
𝑚𝐼1
𝐼4=
2𝐼1
cos φ
11
Let 𝑅4 is the resistance of each line conductor
copper losses in line W = 𝐼4
2
𝑅4+ 𝐼4
2
𝑅4 = 2𝐼4
2
𝑅4
=2(
2𝐼1
cos φ
)2 𝑅4 =4
𝐼1
2
𝑐𝑜𝑠2φ
𝑅4 12
6 October 2020 D.Rajesh Asst.Prof. EEE Department 12
 Comparing copper losses with 2 wire d.c system
4
𝐼1
2
𝑐𝑜𝑠2φ
𝑅4= 2𝐼1
2
𝑅1
𝑅4
𝑅1
=
𝑐𝑜𝑠2φ
2
𝑎4
𝑎1
=
2
𝑐𝑜𝑠2φ
13
volume of copper wire 1−phase 2− wire
volume of copper wire Two−wire D.C system
=
𝑎4𝑙4
𝑎1𝑙1
=
2
𝑐𝑜𝑠2φ𝑎12𝑙
𝑎12𝑙
=
2
𝑐𝑜𝑠2φ
Hence, the volume of conductor material required in this system is
2
𝑐𝑜𝑠2φ
times
that of 2-wire d.c. system with the one conductor earthed.
6 October 2020 D.Rajesh Asst.Prof. EEE Department 13
 The two wires possess equal and opposite voltages to earth (i.e., 𝑉
𝑚).
 Therefore, the maximum voltage between the two wires is
𝑉𝑚
2
. The
r.m.s. value of voltage between conductors is =2
𝑉𝑚
2
= 2 𝑉
𝑚.
Assuming the power factor of the load to be cos φ, load current 𝐼5
 Power supplied P= 2 𝑉
𝑚𝐼5 cos φ 14
Since from assumption 1 the power transmitted is same
for all systems so from eq 1& 14
2 𝑉
𝑚𝐼5 cos φ= 𝑉
𝑚𝐼1
𝐼5=
𝐼1
2 cos φ
15
6 October 2020 D.Rajesh Asst.Prof. EEE Department 14
Let 𝑅5 is the resistance of each line conductor
copper losses in line W = 𝐼5
2
𝑅5+ 𝐼5
2
𝑅5 = 2𝐼5
2
𝑅5
=2(
𝐼1
2 cos φ
)2 𝑅5 =
𝐼1
2
𝑐𝑜𝑠2φ
𝑅5
Comparing copper losses with 2 wire d.c system
𝐼1
2
𝑐𝑜𝑠2φ
𝑅5 =2𝐼1
2
𝑅1
𝑅5
𝑅1
= 2𝑐𝑜𝑠2φ
𝑎5
𝑎1
=
1
2𝑐𝑜𝑠2φ
volume of copper wire 1−phase 2− wire 𝑤𝑖𝑡ℎ 𝑚𝑖𝑑 − 𝑝𝑜𝑖𝑛𝑡 𝑒𝑎𝑟𝑡ℎ𝑒𝑑
volume of copper wire Two−wire D.C system
𝑎5𝑙5
𝑎1𝑙1
=
1
2𝑐𝑜𝑠2φ 𝑎12𝑙
𝑎12𝑙
=
1
2𝑐𝑜𝑠2φ
Hence the volume of conductor material required in this system is
1
2𝑐𝑜𝑠2φ
times that of 2-wire d.c. system with one conductor earthed.
6 October 2020 D.Rajesh Asst.Prof. EEE Department 15
 If the load is balanced, the current through the neutral wire is zero.
Assuming balanced load,
 The two wires possess equal and opposite voltages to earth (i.e., 𝑉
𝑚).
 Therefore, the maximum voltage between the two wires is
𝑉𝑚
2
. The r.m.s.
value of voltage between conductors is 2
𝑉𝑚
2
= 2 𝑉
𝑚. Assuming the
power factor of the load to be cos φ, load current 𝐼6
 Power supplied P= 2 𝑉
𝑚𝐼6 cos φ 16
Since from assumption 1 the power transmitted is same
for all systems so from eq 1& 16
2 𝑉
𝑚𝐼6 cos φ= 𝑉
𝑚𝐼1
𝐼6=
𝐼1
2 cos φ
17
6 October 2020 D.Rajesh Asst.Prof. EEE Department 16
Let 𝑅6 is the resistance of each line conductor
copper losses in line W = 𝐼6
2
𝑅6+ 𝐼6
2
𝑅6 = 2𝐼6
2
𝑅6
=2(
𝐼1
2 cos φ
)2 𝑅6 =
𝐼1
2
𝑐𝑜𝑠2φ
𝑅6
Comparing copper losses with 2 wire d.c system
𝐼1
2
𝑐𝑜𝑠2φ
𝑅6 =2𝐼1
2
𝑅1
𝑅6
𝑅1
= 2𝑐𝑜𝑠2φ
𝑎6
𝑎1
=
1
2𝑐𝑜𝑠2φ
volume of copper wire 1−phase3−wire system
volume of copper wire Two−wire D.C system
𝑎6𝑙6
𝑎1𝑙1
=
1
2𝑐𝑜𝑠2φ 𝑎12.5𝑙
𝑎12𝑙
=
0.625
𝑐𝑜𝑠2φ
Hence the volume of conductr material required in this system is
0.625
𝑐𝑜𝑠2φ
times that of 2-wire d.c. system with one conductor earthed.
6 October 2020 D.Rajesh Asst.Prof. EEE Department 17
 This system can be considered as two independent single-
phase systems, each transmitting one half of the total power.
 As the voltage is same (𝑉
𝑚) as in a Single phase 2-wire system
with mid-point earthed, so the current will be half of the
Single phase system
 i.e. 𝐼7=
𝐼5
2
=
𝐼1
2 cos φ
2
=
𝐼1
2 2 cos φ
 If the current density remains constant the
X-area will be half of the single-phase system
 i.e. 𝑎7=
𝑎5
2
=
1
2𝑐𝑜𝑠2φ 𝑎1
2
=
𝑎1
4𝑐𝑜𝑠2φ
6 October 2020 D.Rajesh Asst.Prof. EEE Department 18
 Since wires are four
 Then

volume of copper wireTwo phase, 4−wire a.c. system
volume of copper wire Two−wire D.C system
𝑎7𝑙7
𝑎1𝑙1
=
1
4𝑐𝑜𝑠2φ 𝑎14𝑙
𝑎12𝑙
=
1
2𝑐𝑜𝑠2φ
Hence the volume of conductor material required in this system
is
1
2𝑐𝑜𝑠2φ
times that of 2-wire d.c. system with one conductor
earthed.
6 October 2020 D.Rajesh Asst.Prof. EEE Department 19
The third or neutral wire is taken from the junction of two-phase windings whose
voltages are in quadrature with each other. Obviously, each phase transmits one
half of the total power. The R.M.S. voltage between outgoing conductor and
neutral =
𝑉𝑚
2
, Let 𝑅8 is the resistance of each line conductor
Total power transmitted=2
𝑉𝑚
2
𝐼8 cos φ
Comparing the power with 2-wire D.C
2
𝑉𝑚
2
𝐼8 cos φ= 𝑉
𝑚𝐼1
𝐼8 =
𝐼1
2cos φ
The middle wire will act as a common return and the current through it will be
𝐼8
2
+ 𝐼8
2
= 2 𝐼8
6 October 2020 D.Rajesh Asst.Prof. EEE Department 20
 Assuming the current density to be constant, the area of X-section of the neutral wire will be
2 times that of either of the outers.
 ∴ Resistance of neutral wire=
𝑅8
2
 Copper losses in this system
W= 𝐼8
2
𝑅8+ 𝐼8
2
𝑅8 + ( 2 𝐼8)2𝑅8
2
= 2𝐼8
2
𝑅8+ 2𝐼8
2
𝑅8=(2+ 2) 𝐼8
2
𝑅8
Where 𝐼8 =
𝐼1
2cos φ
W=(2+ 2) (
𝐼1
2cos φ)2 𝑅8=(2+ 2)
𝐼1
2𝑅8
2𝑐𝑜𝑠2φ
Comparing copper losses with 2 wire d.c system
(2+ 2)
𝐼1
2𝑅8
2𝑐𝑜𝑠2φ
= 2𝐼1
2
𝑅1
𝑅8
𝑅1
=
4𝑐𝑜𝑠2φ
(2+ 2)
𝑎8
𝑎1
=
(2+ 2)
4𝑐𝑜𝑠2φ
volume of copper wireTwo phase, 3−wire a.c. system
volume of copper wire Two−wire D.C system
𝑎8𝑙8
𝑎1𝑙1
=
(2+ 2)
4𝑐𝑜𝑠2φ 𝑎1(2+ 2)𝑙
𝑎12𝑙
=
1.457
𝑐𝑜𝑠2φ
Hence, the volume of conductor material required for this system is
1.457
𝑐𝑜𝑠2φ times that of 2-wire d.c.
system with one conductor earthed.
6 October 2020 D.Rajesh Asst.Prof. EEE Department 21
This system is almost universally adopted for transmission of electric power.
The 3-phase, 3- wire system may be star connected or delta connected.
Maximum voltage between each conductor and neutral =𝑉
𝑚
Let 𝑅9 is the resistance of each line conductor
R.M.S. voltage per phase =
𝑉𝑚
2
Power supplied=3(
𝑉𝑚
2
I9cos φ )
 Comparing the power with 2-wire D.C
3
𝑉𝑚
2
𝐼9 cos φ= 𝑉
𝑚𝐼1
𝐼9 =
2𝐼1
3cos φ
6 October 2020 D.Rajesh Asst.Prof. EEE Department 22
 Total copper losses= 3𝐼9
2
𝑅9 =3 (
2𝐼1
3cos φ
)2 𝑅9=
2𝐼1
2
𝑅9
3𝑐𝑜𝑠2φ
 Comparing copper losses with 2 wire d.c system
2𝐼1
2
𝑅9
3𝑐𝑜𝑠2φ
= 2𝐼1
2
𝑅1
𝑅9
𝑅1
= 3𝑐𝑜𝑠2φ
𝑎9
𝑎1
=
1
3𝑐𝑜𝑠2φ
volume of copper wire3−Phase, 3−wire system
volume of copper wire Two−wire D.C system
=
𝑎9𝑙9
𝑎1𝑙1
=
1
3𝑐𝑜𝑠2φ𝑎13𝑙
𝑎12𝑙
=
0.5
𝑐𝑜𝑠2φ
Hence, the volume of conductor material required for this system is
0.5
𝑐𝑜𝑠2φ times that required for 2-wire d.c. system with one conductor earthed.
6 October 2020 D.Rajesh Asst.Prof. EEE Department 23
In this case, 4th or neutral wire is taken from the neutral point. The area of X-
section of the neutral wire is generally one-half that of the line conductor. If the
loads are balanced, then current through the neutral wire is zero. Assuming
balanced loads and p.f. of the load as cos φ,
Let 𝑅10 is the resistance of each line conductor
R.M.S. voltage per phase =
𝑉𝑚
2
Power supplied=3(
𝑉𝑚
2
I10cos φ )
 Comparing the power with 2-wire D.C
3
𝑉𝑚
2
𝐼10 cos φ= 𝑉
𝑚𝐼1
𝐼10 =
2𝐼1
3cos φ
6 October 2020 D.Rajesh Asst.Prof. EEE Department 24
 Total copper losses= 3𝐼10
2
𝑅10 =3 (
2𝐼1
3cos φ
)2 𝑅10=
2𝐼1
2
𝑅10
3𝑐𝑜𝑠2φ
 Comparing copper losses with 2 wire d.c system
2𝐼1
2
𝑅10
3𝑐𝑜𝑠2φ
= 2𝐼1
2
𝑅1
𝑅10
𝑅1
= 3𝑐𝑜𝑠2φ
𝑎10
𝑎1
=
1
3𝑐𝑜𝑠2φ
volume of copper wire3−Phase, 3−wire system
volume of copper wire Two−wire D.C system
=
𝑎10𝑙10
𝑎1𝑙1
=
1
3𝑐𝑜𝑠2φ𝑎13.5𝑙
𝑎12𝑙
=
0.583
𝑐𝑜𝑠2φ
Hence, the volume of conductor material required for this system is
0.583
𝑐𝑜𝑠2φ times that required for 2-wire d.c. system with one conductor earthed.
6 October 2020 D.Rajesh Asst.Prof. EEE Department 25
In the under-ground system the maximum stress exists
on the insulation between the conductors.
(i) Same power (P watts) transmitted by each system.
(ii) The distance (l meters) over which power is
transmitted remains the same.
(iii) The line losses (W watts) are the same in each case.
(iv) The maximum voltage between conductors (𝑉
𝑚) is
the same in each case.
6 October 2020 D.Rajesh Asst.Prof. EEE Department 26
Max. voltage between conductors = 𝑉
𝑚 ,
Power to be transmitted = P= 𝑉
𝑚𝐼1 1
∴ Load current, 𝐼1 = P/ 𝑉
𝑚
If 𝑅1 is the resistance of each line conductor, then,
𝑅1 = ρ l/ 𝑎1
where 𝑎1 is the area of X-section of the conductor.
copper losses in line W = 𝐼1
2
𝑅1 + 𝐼1
2
𝑅1
=2𝐼1
2
𝑅1
6 October 2020 D.Rajesh Asst.Prof. EEE Department 27
1
1
2
Maximum voltage between conductors is 𝑉
𝑚. 𝑅2 is the resistance of each line
conductor
Power to be transmitted
P= 𝑉
𝑚𝐼2
Load current, 𝐼2 = P/ 𝑉
𝑚
Since from assumption 1 the power transmitted is same
for all systems so from eq 1& 3
𝑉
𝑚𝐼2= 𝑉
𝑚𝐼1
𝐼2 = 𝐼1
copper losses in line W = 𝐼2
2
𝑅2+ 𝐼2
2
𝑅2 = 2 𝐼2
2
𝑅2
From eq 4
W=2( 𝐼1)2 𝑅2 =2𝐼1
2
𝑅2
6 October 2020 D.Rajesh Asst.Prof. EEE Department 28
3
4
5
 Compare eq 2 & 5
2𝐼1
2
𝑅1 = (𝐼1
2
) 𝑅2
𝑅1
𝑅2
=1
𝑎1
𝑎2
=1 (where R = ρ l/ 𝑎)
𝑎1=X-area of conductor in 2 wire dc system
𝑎2= X-area of conductor in 2 wire dc system with mid-point earthed
Then
volume of copper wire Two−wire D.C system with mid−point earthed
volume of copper wire Two−wire D.C system
=
𝑎2∗𝑙2
𝑎1∗𝑙1
=
𝑎1∗2𝑙
𝑎1∗2𝑙
=1 where 𝑙1 = 𝑙2=l+l
Hence, the volume of conductor material required in this system is the same as
that for 2-wire d.c. system.
6 October 2020 D.Rajesh Asst.Prof. EEE Department 29
If the load is balanced, the current in the neutral wire is zero. Assuming balanced loads,
Power to be transmitted
P= 𝑉
𝑚𝐼3 6
Since from assumption 1 the power transmitted is same
for all systems so from eq 1& 6
𝑉
𝑚𝐼3= 𝑉
𝑚𝐼1
𝐼3 = 𝐼1 7
Let 𝑅3 is the resistance of each line conductor,
the area of X-section of neutral wire to be half that of the outer wire,
copper losses in line W = 𝐼3
2
𝑅3+ 𝐼3
2
𝑅3 = 2𝐼3
2
𝑅3 8
From eq 7
W=2( 𝐼1)2
𝑅3 =2𝐼1
2
𝑅3 9
6 October 2020 D.Rajesh Asst.Prof. EEE Department 30
 Compare eq 2 & 8
2𝐼1
2
𝑅1 = 2(𝐼1
2
) 𝑅3
𝑅1
𝑅3
=1
𝑎1
𝑎3
=1 𝑅3 = ρ l/ 𝑎3
where 𝑎3 is the area of X-section of the conductor.
Then
volume of copper wire Three−wire D.C
volume of copper wire Two−wire D.C system
=
𝑎3∗𝑙3
𝑎1∗𝑙1
=
𝑎1∗2.5𝑙
𝑎1∗2𝑙
=
2.5
2
where 𝑙3=l+l+0.5l
=1.25
Hence, the volume of conductor material required in the system is 1·25 times
that required for 2-wire d.c. system.
6 October 2020 D.Rajesh Asst.Prof. EEE Department 31
The maximum voltage between conductors is 𝑉
𝑚, so that r.m.s. value of voltage
between them is
𝑉𝑚
2
. Assuming the load power factor to be cos φ, load current 𝐼4
Power supplied P=
𝑉𝑚
2
𝐼4 cos φ 10
Since from assumption 1 the power transmitted is same
for all systems so from eq 1& 10
𝑉𝑚
2
𝐼4 cos φ= 𝑉
𝑚𝐼1
𝐼4=
2𝐼1
cos φ
11
Let 𝑅4 is the resistance of each line conductor
copper losses in line W = 𝐼4
2
𝑅4+ 𝐼4
2
𝑅4 = 2𝐼4
2
𝑅4
=2(
2𝐼1
cos φ
)2 𝑅4 =4
𝐼1
2
𝑐𝑜𝑠2φ
𝑅4 12
6 October 2020 D.Rajesh Asst.Prof. EEE Department 32
 Comparing copper losses with 2 wire d.c system
4
𝐼1
2
𝑐𝑜𝑠2φ
𝑅4= 2𝐼1
2
𝑅1
𝑅4
𝑅1
=
𝑐𝑜𝑠2φ
2
𝑎4
𝑎1
=
2
𝑐𝑜𝑠2φ
13
volume of copper wire 1−phase 2− wire
volume of copper wire Two−wire D.C system
=
𝑎4𝑙4
𝑎1𝑙1
=
2
𝑐𝑜𝑠2φ𝑎12𝑙
𝑎12𝑙
=
2
𝑐𝑜𝑠2φ
Hence, the volume of conductor material required in this system is
2
𝑐𝑜𝑠2φ
times
that of 2-wire d.c. system.
6 October 2020 D.Rajesh Asst.Prof. EEE Department 33
The maximum value of voltage between the outers is 𝑉
𝑚.
Therefore, the r.m.s. value of voltage is
𝑉𝑚
2
Assuming the power factor of the load to be cos φ, load current 𝐼5
Power supplied P=
𝑉𝑚
2
𝐼5 cos φ 14
Since from assumption 1 the power transmitted is same
for all systems so from eq 1& 14
𝑉𝑚
2
𝐼5 cos φ= 𝑉
𝑚𝐼1
𝐼5=
2𝐼1
cos φ
15
6 October 2020 D.Rajesh Asst.Prof. EEE Department 34
Let 𝑅5 is the resistance of each line conductor
copper losses in line W = 𝐼5
2
𝑅5+ 𝐼5
2
𝑅5 = 2𝐼5
2
𝑅5
=2(
2𝐼1
cos φ
)2 𝑅5 =
4𝐼1
2
𝑐𝑜𝑠2φ
𝑅5
Comparing copper losses with 2 wire d.c system
4𝐼1
2
𝑐𝑜𝑠2φ
𝑅5 =2𝐼1
2
𝑅1
𝑅5
𝑅1
=
𝑐𝑜𝑠2φ
2
𝑎5
𝑎1
=
2
𝑐𝑜𝑠2φ
volume of copper wire 1−phase 2− wire 𝑤𝑖𝑡ℎ 𝑚𝑖𝑑 − 𝑝𝑜𝑖𝑛𝑡 𝑒𝑎𝑟𝑡ℎ𝑒𝑑
volume of copper wire Two−wire D.C system
𝑎5𝑙5
𝑎1𝑙1
=
2
𝑐𝑜𝑠2φ 𝑎12𝑙
𝑎12𝑙
=
2
𝑐𝑜𝑠2φ
Hence, the volume of conductor material required in this system is
2
𝑐𝑜𝑠2φ
times that required in 2-wire d.c. system.
6 October 2020 D.Rajesh Asst.Prof. EEE Department 35
 If the load is balanced, the current through the neutral wire is zero.
Assuming balanced load,
 The maximum value of voltage between the outers is 𝑉
𝑚. Therefore, the
r.m.s. value of voltage is
𝑉𝑚
2
.
 Assuming the power factor of the load to be cos φ, load current 𝐼6
 Power supplied P=
𝑉𝑚
2
𝐼6 cos φ 16
Since from assumption 1 the power transmitted is same
for all systems so from eq 1& 16
𝑉𝑚
2
𝐼6 cos φ= 𝑉
𝑚𝐼1
𝐼6=
2𝐼1
cos φ
17
6 October 2020 D.Rajesh Asst.Prof. EEE Department 36
Let 𝑅6 is the resistance of each line conductor
copper losses in line W = 𝐼6
2
𝑅6+ 𝐼6
2
𝑅6 = 2𝐼6
2
𝑅6
=2(
2𝐼1
cos φ
)2 𝑅6 =
4𝐼1
2
𝑐𝑜𝑠2φ
𝑅6
Comparing copper losses with 2 wire d.c system
4𝐼1
2
𝑐𝑜𝑠2φ
𝑅6 =2𝐼1
2
𝑅1
𝑅6
𝑅1
=
𝑐𝑜𝑠2φ
2
𝑎6
𝑎1
=
2
𝑐𝑜𝑠2φ
volume of copper wire 1−phase 2− wire 𝑤𝑖𝑡ℎ 𝑚𝑖𝑑 − 𝑝𝑜𝑖𝑛𝑡 𝑒𝑎𝑟𝑡ℎ𝑒𝑑
volume of copper wire Two−wire D.C system
𝑎6𝑙6
𝑎1𝑙1
= =
2
𝑐𝑜𝑠2φ 𝑎12.5𝑙
𝑎12𝑙
=
2.5
𝑐𝑜𝑠2φ
 Hence, the volume of conductor material required in this system is
2.5
𝑐𝑜𝑠2φ
times that required in 2-wire d.c. system.
6 October 2020 D.Rajesh Asst.Prof. EEE Department 37
 This system can be considered as two independent single-phase
systems, each transmitting one half of the total power.
 Current (𝐼7) in each conductor will be half that in single-phase 2-
wire system. Consequently, area of X-section of each conductor is
also half but as there are four wires, so volume of conductor material
used is the same as in a single phase, 2-wire system
 i.e. 𝐼7=
𝐼5
2
=
2𝐼1
cos φ
2
=
𝐼1
2 cos φ
 If the current density remains constant the
X-area will be half of the single-phase system
 i.e. 𝑎7=
𝑎5
2
=
2
𝑐𝑜𝑠2φ
2
=
1
𝑐𝑜𝑠2φ
6 October 2020 D.Rajesh Asst.Prof. EEE Department 38
 Since wires are four
 Then

volume of copper wireTwo phase, 4−wire a.c. system
volume of copper wire Two−wire D.C system
𝑎7𝑙7
𝑎1𝑙1
=
1
𝑐𝑜𝑠2φ 𝑎14𝑙
𝑎12𝑙
=
2
𝑐𝑜𝑠2φ
Hence, volume of conductor material required in this
system is
2
𝑐𝑜𝑠2φ
times that required in 2- wire d.c. system.
6 October 2020 D.Rajesh Asst.Prof. EEE Department 39
 Let us suppose that maximum voltage between the outers is 𝑉
𝑚. Then
maximum voltage between either outer and neutral wire is
𝑉𝑚
2
 R.M.S. voltage between outer and neutral wire=
𝑉𝑚
2
2
=
𝑉𝑚
2
 Total power transmitted=2
𝑉𝑚
2
𝐼8 cos φ = 𝑉
𝑚 𝐼8 cos φ
 Comparing the power with 2-wire D.C
𝑉
𝑚 𝐼8 cos φ= 𝑉
𝑚𝐼1
𝐼8 =
𝐼1
cos φ
The middle wire will act as a common return and the current through it will be
𝐼8
2
+ 𝐼8
2
= 2 𝐼8
6 October 2020 D.Rajesh Asst.Prof. EEE Department 40
 Assuming the current density to be constant, the area of X-section of the neutral wire will be
2 times that of either of the outers.
 ∴ Resistance of neutral wire=
𝑅8
2
 Copper losses in this system
W= 𝐼8
2
𝑅8+ 𝐼8
2
𝑅8 + ( 2 𝐼8)2𝑅8
2
= 2𝐼8
2
𝑅8+ 2𝐼8
2
𝑅8=(2+ 2) 𝐼8
2
𝑅8
Where 𝐼8 =
𝐼1
cos φ
W=(2+ 2) (
𝐼1
cos φ)2
𝑅8=(2+ 2)
𝐼1
2𝑅8
𝑐𝑜𝑠2φ
Comparing copper losses with 2 wire d.c system
(2+ 2)
𝐼1
2𝑅8
𝑐𝑜𝑠2φ
= 2𝐼1
2
𝑅1
𝑅8
𝑅1
=
2𝑐𝑜𝑠2φ
(2+ 2)
𝑎8
𝑎1
=
(2+ 2)
2𝑐𝑜𝑠2φ
volume of copper wireTwo phase, 3−wire a.c. system
volume of copper wire Two−wire D.C system
𝑎8𝑙8
𝑎1𝑙1
=
(2+ 2)
2𝑐𝑜𝑠2φ 𝑎1(2+ 2)𝑙
𝑎12𝑙
=
2.914
𝑐𝑜𝑠2φ
Hence, the volume of conductor material required for this system is
2.914
𝑐𝑜𝑠2φ times that of 2-wire d.c.
system.
6 October 2020 D.Rajesh Asst.Prof. EEE Department 41
This system is almost universally adopted for transmission of electric power. The 3-phase,
3- wire system may be star connected or delta connected.
Maximum voltage between conductors =𝑉
𝑚, Let 𝑅9 is the resistance of each line conductor
Then maximum voltage between each phase and neutral is
𝑉𝑚
3
R.M.S. voltage per phase =
𝑉𝑚
3
2
=
𝑉𝑚
6
Power supplied=3(
𝑉𝑚
6
I9cos φ )
 Comparing the power with 2-wire D.C
3
𝑉𝑚
6
𝐼9 cos φ= 𝑉
𝑚𝐼1
𝐼9 =
6𝐼1
3cos φ
6 October 2020 D.Rajesh Asst.Prof. EEE Department 42
 Total copper losses= 3𝐼9
2
𝑅9 =3 (
6𝐼1
3cos φ
)2 𝑅9=
2𝐼1
2
𝑅9
𝑐𝑜𝑠2φ
 Comparing copper losses with 2 wire d.c system
2𝐼1
2
𝑅9
𝑐𝑜𝑠2φ
= 2𝐼1
2
𝑅1
𝑅9
𝑅1
= 𝑐𝑜𝑠2φ
𝑎9
𝑎1
=
1
𝑐𝑜𝑠2φ
volume of copper wire3−Phase, 3−wire system
volume of copper wire Two−wire D.C system
=
𝑎9𝑙9
𝑎1𝑙1
=
1
𝑐𝑜𝑠2φ𝑎13𝑙
𝑎12𝑙
=
1.5
𝑐𝑜𝑠2φ
Hence, the volume of conductor material required for this system is
1.5
𝑐𝑜𝑠2φ times that required for 2-wire d.c. system.
6 October 2020 D.Rajesh Asst.Prof. EEE Department 43
In this case, 4th or neutral wire is taken from the neutral point. The area of X-
section of the neutral wire is generally one-half that of the line conductor. If the
loads are balanced, then current through the neutral wire is zero. Assuming
balanced loads and p.f. of the load as cos φ,
Let 𝑅10 is the resistance of each line conductor
R.M.S. voltage per phase =
𝑉𝑚
6
Power supplied=3(
𝑉𝑚
6
I10cos φ )
 Comparing the power with 2-wire D.C
3
𝑉𝑚
6
𝐼10 cos φ= 𝑉
𝑚𝐼1
𝐼10 =
6𝐼1
3cos φ
6 October 2020 D.Rajesh Asst.Prof. EEE Department 44
 Total copper losses= 3𝐼10
2
𝑅10 =3 (
6𝐼1
3cos φ
)2 𝑅10=
2𝐼1
2
𝑅10
𝑐𝑜𝑠2φ
 Comparing copper losses with 2 wire d.c system
2𝐼1
2
𝑅10
𝑐𝑜𝑠2φ
= 2𝐼1
2
𝑅1
𝑅10
𝑅1
= 𝑐𝑜𝑠2φ
𝑎10
𝑎1
=
1
𝑐𝑜𝑠2φ
volume of copper wire3−Phase, 3−wire system
volume of copper wire Two−wire D.C system
=
𝑎10𝑙10
𝑎1𝑙1
=
1
𝑐𝑜𝑠2φ𝑎13.5𝑙
𝑎12𝑙
=
1.75
𝑐𝑜𝑠2φ
Hence, the volume of conductor material required for this system
is
1.75
𝑐𝑜𝑠2φtimes that required for 2-wire d.c. system.
6 October 2020 D.Rajesh Asst.Prof. EEE Department 45
System Same maximum
voltage to earth
Same maximum
voltage
between conductors
1. D.C. system
(i) Two-wire
(ii) Two-wire mid-point earthed
(iii) 3-wire
1
0.25
0.3125
1
1
1.25
2. Single phase system
(i) Two-wire
(ii) Two-wire mid-point earthed
(iii) 3-wire
2
𝑐𝑜𝑠2φ
0.5
𝑐𝑜𝑠2φ
1.625
𝑐𝑜𝑠2φ
2
𝑐𝑜𝑠2φ
0.5
𝑐𝑜𝑠2φ
2.5
𝑐𝑜𝑠2φ
6 October 2020 D.Rajesh Asst.Prof. EEE Department 46
System Same maximum voltage to
earth
Same maximum voltage
between conductors
3. Two-phase system
(i) 2–phase 4-wire
(ii)2-phase 3-wire
0.5
𝑐𝑜𝑠2φ
1.457
𝑐𝑜𝑠2φ
2
𝑐𝑜𝑠2φ
2.914
𝑐𝑜𝑠2φ
4. Three-phase system
(i) 3-phase 3-wire
(ii)3-phase 4-wire
,
0.5
𝑐𝑜𝑠2φ
0.583
𝑐𝑜𝑠2φ
1.5
𝑐𝑜𝑠2φ
1.75
𝑐𝑜𝑠2φ
6 October 2020 D.Rajesh Asst.Prof. EEE Department 47
What is the percentage saving in feeder copper if
the line voltage in a 2-wire d.c. system is raised
from 200 volts to 400 volts for the same power
transmitted over the same distance and having the
same power loss ?
6 October 2020 D.Rajesh Asst.Prof. EEE Department 48
 (i) Conductors
 (ii) Step-up and step-down transformers
 (iii) Line insulators
 (iv) Support
 (v) Protective devices
 (vi) Voltage regulating devices
6 October 2020 D.Rajesh Asst.Prof. EEE Department 49
The following two fundamental economic
principles which closely influence the electrical
design of a transmission line
(i) Economic choice of transmission voltage
(ii) Economic choice of conductor size
6 October 2020 D.Rajesh Asst.Prof. EEE Department 50
 The effect of increased voltage level of
transmission on
1. Volume of conductor material
used for transmission
2. Line Drop
6 October 2020 D.Rajesh Asst.Prof. EEE Department 51
Let a 3-phase A.C. system is used for the transmission, The resistance per conductor is given by
R= ρ l/A
Power transmitted P= 3𝑉𝐼cos φ
I=
𝑃
3𝑉cos φ
Total copper losses 𝑃𝑐𝑢= 3𝐼2R =3(
𝑃
3𝑉cos φ
)2ρ l
𝐴
A=
𝑃2ρ l
𝑃𝑐𝑢𝑉2𝑐𝑜𝑠2φ
The volume of copper used is V=3(A*l)
Volume of copper=
3𝑃2ρ 𝑙2
𝑃𝑐𝑢𝑉2𝑐𝑜𝑠2φ
Volume of copper 𝛼
1
𝑉2𝑐𝑜𝑠2φ
6 October 2020 D.Rajesh Asst.Prof. EEE Department 52
Thus greater is the
transmission voltage level,
lesser is the volume of the
copper required.
Line Drop=IR=I ρ l/A
Current density(J)=
𝐼
𝐴
Then Line Drop=
𝐼ρ l
𝐼
𝐽
= Jρ l
%Line Drop=
Jρ l
𝑉
*100
Higher the transmission voltage level, then lesser the
%Line Drop
6 October 2020 D.Rajesh Asst.Prof. EEE Department 53
 The most economical area of conductor is that
for which the total annual cost of transmission
line is minimum . This is known as Kelvin’s
Law
 The total annual cost of transmission line can
be divided broadly into two parts viz.,
 Annual charge on capital outlay
 Annual cost of energy wasted in the conductor.
6 October 2020 D.Rajesh Asst.Prof. EEE Department 54
 This is on account of interest and depreciation on the capital cost of
complete installation of transmission line. In case of overhead system,
it will be the annual interest and depreciation on the capital cost of
conductors, supports and insulators and the cost of their erection.
 Now, for an overhead line, insulator cost is constant, the conductor
cost is proportional to the area of X-section and the cost of supports
and their erection is partly constant annual charge.
 Transmission line can be expressed as :
𝐴𝑛𝑛𝑢𝑎𝑙 𝑐ℎ𝑎𝑟𝑔𝑒 = 𝑃1 + 𝑃2𝑎
𝑃1 constant depending on the copper conductor
𝑃2 constant depending on the X-area of the conductor
a is the area of X-section of the conductor
6 October 2020 D.Rajesh Asst.Prof. EEE Department 55
 This is on account of energy lost mainly in the conductor due to 𝐼2R losses.
 Assuming a constant current in the conductor throughout the year,
 The energy lost in the conductor is proportional to resistance.
where R = ρ l/ 𝑎
 The energy lost in the conductor is inversely proportional to area of X-section.
energy lost α
1
𝑎
 Thus, the annual cost of energy wasted in an overhead transmission line can
be expressed as :
Annual cost of energy wasted =
𝑃3
𝑎
where P3 is a constant.
6 October 2020 D.Rajesh Asst.Prof. EEE Department 56
 Total annual cost, C=𝑃1 + 𝑃2𝑎+
𝑃3
𝑎
only area of X-section a is variable. Therefore, the total annual cost of
transmission line will be minimum if differentiation of C w.r.t. a is zero i.e.
ⅆ𝐶
ⅆ𝑎
=0
ⅆ
ⅆ𝑎
(𝑃1 + 𝑃2𝑎+
𝑃3
𝑎
)=0
𝑃2𝑎 −
𝑃3
𝑎2=0
𝑃2𝑎 =
𝑃3
𝑎2
i.e. Variable part of annual charge = Annual cost of energy wasted
Therefore Kelvin’s Law can also be stated
The most economical area of conductor is that for which the variable part of
annual charge is equal to the cost of energy losses per year.
6 October 2020 D.Rajesh Asst.Prof. EEE Department 57
 The straight line shows the relation between the
annual charge (i.e., 𝑃1 + 𝑃2𝑎) and the area of X-
section a of the conductor
 The rectangular hyperbola
gives the relation between
annual cost of energy wasted (
𝑃3
𝑎
)
and X-sectional area a.
 Curve 3 shows the relation between total annual
cost of transmission line and area of X-section a.
6 October 2020 D.Rajesh Asst.Prof. EEE Department 58
 (i) It is not easy to estimate the energy loss in the line without actual load
curves, which are not available at the time of estimation.
 (ii) The assumption that annual cost on account of interest and
depreciation on the capital outlay is in the form 𝑃1 + 𝑃2𝑎 is strictly
speaking not true. For instance, in cables neither the cost of cable dielectric
and sheath nor the cost of laying vary in this manner.
 (iii) This law does not take into account several physical factors like safe
current density, mechanical strength, corona loss etc.
 (iv) The conductor size determined by this law may not always be
practicable one because it may be too small for the safe carrying of
necessary current.
 (v) Interest and depreciation on the capital outlay cannot be determined
accurately.
6 October 2020 D.Rajesh Asst.Prof. EEE Department 59
A 2-conductor cable 1 km long is required to supply a constant current
of 200 A throughout the year. The cost of cable including installation is
Rs. (20 a + 20) per meter where ‘a’ is the area of X-section of the
conductor in 𝑐𝑚2
. The cost of energy is 5P per kWh and interest and
depreciation charges amount to 10%. Calculate the most economical
conductor size. Assume resistivity of conductor material to be 1·73 μ Ω
cm.
economical conductor size a =?
Energy lost per annum=2𝐼2
𝑅𝑡
Resistance of one conductor=ρ l/ 𝑎
=
1.73∗10−6∗105
𝑎
=
0.173
𝑎
Ω
Energy lost per annum= 2∗ 2002
∗
0.173
𝑎
∗ 8760
=
121238.4
𝑎
kWh
6 October 2020 D.Rajesh Asst.Prof. EEE Department 60
 Annual cost of energy lost = Cost per kWh × Annual energy loss
=Rs 0.05*
121238.4
𝑎
kWh=Rs
6062
𝑎
The capital cost (variable) of the cable is given to be Rs 20 a per metre.
Therefore, for 1 km length of the cable, the capital cost (variable) is Rs. 20 a ×
1000 = Rs. 20,000 a.
Variable annual charge = Annual interest and depreciation on
capital cost (variable) of cable
= Rs 0·1 × 20,000 a
= Rs 2000 a
According to Kelvin’s law, for most economical X-section of the conductor,
Variable annual charge = Annual cost of energy lost
2000 a =
6062
𝑎
a=
6062
2000
=1.74 𝑐𝑚2
6 October 2020 D.Rajesh Asst.Prof. EEE Department 61
A 2-wire feeder carries a constant current of 250 A throughout the year.
The portion of capital cost which is proportional to area of X-section is
Rs 5 per kg of copper conductor. The interest and depreciation total
10% per annum and the cost of energy is 5P per kWh. Find the most
economical area of X-section of the conductor. Given that the density of
copper is 8·93gm/ 𝑐𝑚3
and its specific resistance is 1·73 × 10−8
Ω m.
Consider 1 metre length of the feeder. Let a be the most economical
area of X-section of each conductor in 𝑚2
Energy lost per annum=2𝐼2
𝑅𝑡
Resistance of one conductor=ρ l/ 𝑎
=
1.73∗10−8∗1
𝑎
=
1.73∗10−8
𝑎
Ω
Energy lost per annum= 2∗ 2502
∗
1.73∗10−8
𝑎
∗ 8760
=
18,94,350
𝑎
∗ 10−8
kWh
6 October 2020 D.Rajesh Asst.Prof. EEE Department 62
 Annual cost of energy lost = Cost per kWh × Annual energy loss
=Rs 0.05*
18,94,350
𝑎
∗ 10−8
kWh=Rs
94,717.5
𝑎
∗ 10−8
Mass of 1 metre feeder = 2 (Volume × density)
= 2 × a × 1 × 8·93 × 103kg
= 17·86 × 103 a kg
Capital cost (variable) = Rs 5 × 17·86 × 103
a
= Rs 89·3 × 103
a
Variable annual charge = 10% of capital cost (variable)
= 0·1 × 89·3 × 103 a = Rs 8930 a
According to Kelvin’s law, for most economical X-section of the conductor,
Variable annual charge = Annual cost of energy lost
8930 a =
94,717.5
𝑎
∗ 10−8
a=
94,717.5
8930
∗ 10−8
=3.25∗ 10−4
𝑚2
= 3.25 𝑐𝑚2
6 October 2020 D.Rajesh Asst.Prof. EEE Department 63
Determine the most economical cross-section for a 3-phase
transmission line, 1 km long to supply at a constant voltage of
110 kV for the following daily load cycle :
6 hours 20 MW at p.f. 0·8 lagging
12 hours 5 MW at p.f. 0·8 lagging
6 hours 6 MW at p.f. 0·8 lagging
The line is used for 365 days yearly. The cost per km of line
including erection is Rs (9000 +6000 a) where ‘a’ is the area of X-
section of conductor in cm2. The annual rate of interest and
depreciation is 10% and the energy costs 6P per kWh. The
resistance per km of each conductor is0·176/a.
Power transmitted P= 3𝑉𝐼cos φ
Energy lost per annum=3R(𝐼2
𝑡)
6 October 2020 D.Rajesh Asst.Prof. EEE Department 64
 The load currents at various loads are :
 At 20 MW, 𝐼1=
𝑃
3𝑉cos φ
=
20∗106
3∗110∗103∗0.8
= 131.2𝐴
 At 5 MW, 𝐼1=
𝑃
3𝑉cos φ
=
5∗106
3∗110∗103∗0.8
= 32.8𝐴
 At 6 MW, 𝐼1=
𝑃
3𝑉cos φ
=
6∗106
3∗110∗103∗0.8
= 39.36𝐴
 Energy loss per day in 3-phase line
=3*
0.176
𝑎
(131.22
∗ 6+ 32.82
∗ 12+ 36.362
∗ 6)
=
66.26
𝑎
kWh
Energy lost per annum =
66.26
𝑎
*365= =
24,184.9
𝑎
kWh
Annual cost of energy=Rs
6
100
∗
24,184.9
𝑎
= =
1451.09
𝑎
Variable annual charge = 10% of capital cost (variable) of line
= Rs 0·1 × 6000 a = Rs 600 a
According to Kelvin’s law, for most economical X-section of the conductor,
Variable annual charge = Annual cost of energy
600 a=
1451.09
𝑎
a=
1451.09
600
=1.56 𝑐𝑚2
6 October 2020 D.Rajesh Asst.Prof. EEE Department 65
The conveyance of electric power from a power station to
consumers’ premises is known as electric supply system.
 Electric supply system consist of three principle components:
1) Power station
2) Transmission lines
3) Distribution system
 The electric supply system can be broadly classified into:
1) DC or AC system
2)Overhead or underground system
6 October 2020 D.Rajesh Asst.Prof. EEE Department 66
Different blocks of typical acpowersupply scheme are asper
following::
1)Generating Station:
Ingenerating station power is producedby three phase alternators
operating inparallel.
The usual generation voltage is 11kV.
Foreconomy in the transmission of electric power, the generation
voltage is stepped upto 132 kVor more at generating station with the
help of three phase transformer.
6 October 2020 D.Rajesh Asst.Prof. EEE Department 67
2)Primarytransmission:
Theelectric power at 132 kVis transmitted by 3-phase,3 wire overhead system to the out
3)Secondary transmission:
Atthe receiving station the voltage is reduced to33kV by step down transformer.
4)Primary distribution:
Atthe substation voltage is reduced 33kV to 11kV.The11kV line run along important
roadsides to city. This forms the primary distribution.
5)Secondarydistribution:
Theelectric power form primary distribution line is delivered to distribution substation. The
substation is located nearthe consumers location and step down the voltage to 400V, 3-
phase ,4-wire for secondary distribution.
The voltage between any two phases is 400V and between any phase and neutral is 230V.
Basis for Comparison AC Transmission Line DC Transmission Line
Definition The ac transmission line transmit the
alternating current.
The dc transmission line is used for
transmitting the direct current.
Number of Conductors Three Two
Inductance & surges Have Don’t Have
Voltage drop High Low
Skin Effect Occurs Absent
Need of Insulation More Less
Interference Have Don’t Have
Corona Loss Occur Don’t occur
Dielectric Loss Have Don’t have
Synchronizing and Stability Problem No difficulties Difficulties
Cost Expensive Cheap
Length of conductors Small More
Repairing and Maintenance Easy and Inexpensive Difficult and Expensive
Transformer Requires Not Requires
6 October 2020 D.Rajesh Asst.Prof. EEE Department 70
6 October 2020 D.Rajesh Asst.Prof. EEE Department 71
6 October 2020 D.Rajesh Asst.Prof. EEE Department 72
Which distributes
electric power for
local use is known as
distribution system.
6 October 2020 D.Rajesh Asst.Prof. EEE Department 73
 (i) Feeders. A feeder is a
conductor which connects the
sub-station to the area where
power is to be distributed.
 (ii) Distributor. A distributor is
a conductor from which
tappings are taken for supply
to the consumers.
 (iii) Service mains. A service
mains is generally a small cable
which connects the distributor
to the consumers’ terminals.
6 October 2020 D.Rajesh Asst.Prof. EEE Department 74
6 October 2020 D.Rajesh Asst.Prof. EEE Department 75
Primary
distribution system
6 October 2020 D.Rajesh Asst.Prof. EEE Department 76
Secondary
distribution system
2-wire d.c. system
6 October 2020 D.Rajesh Asst.Prof. EEE Department 77
3 -wire d.c. system
6 October 2020 D.Rajesh Asst.Prof. EEE Department 78
6 October 2020 D.Rajesh Asst.Prof. EEE Department 79
 (i) Radial System.
In this system, separate feeders radiate from a single substation and feed
the distributors at one end only.
6 October 2020 D.Rajesh Asst.Prof. EEE Department 80
 Limitations
 (a) The end of the distributor nearest to the feeding point will be
heavily loaded.
 (b) The consumers are dependent on a single feeder and single
distributor. Therefore, any fault on the feeder or distributor cuts off
supply to the consumers who are on the side of the fault away from
the substation.
 (c) The consumers at the distant end of the distributor would be
subjected to serious voltage fluctuations when the load on the
distributor changes.
Due to these limitations, this system is used for short
distances only.
6 October 2020 D.Rajesh Asst.Prof. EEE Department 81
 (ii) Ring main system.
In this system, the primaries of distribution transformers form a loop. The
loop circuit starts from the substation bus-bars, makes a loop through the area to be served,
and returns to the substation.
✓ substation supplies to the
closed feeder LMNOPQRS.
✓ The distributors are tapped
from different points M, O and Q
of the feeder through distribution
transformers.
➢ There are less voltage fluctuations
at consumer’s terminals.
➢ The system is very reliable as each
distributor is fed via two feeders.
➢ In the event of fault
on any section of the feeder,
the continuity of supply is maintained.
6 October 2020 D.Rajesh Asst.Prof. EEE Department 82
 (iii) Interconnected system. When the feeder ring is energized by two or
more than two generating stations or substations, it is called inter-connected
system.
(a) It increases the service reliability.
(b) Any area fed from one generating station during peak load hours can be fed
from the other generating station. This reduces reserve power capacity and
increase efficiency of the system.
6 October 2020 D.Rajesh Asst.Prof. EEE Department 83
 (i) Proper voltage.
 One important requirement of a distribution system is that voltage
variations at consumer’s terminals should be as low as possible.
 The changes in voltage are generally caused due to the variation of
load on the system.
 Low voltage causes loss of revenue, inefficient lighting and possible
burning out of motors.
 High voltage causes lamps to burn out permanently and may cause
failure of other appliances.
 Thus, if the declared voltage is 230 V, then the highest voltage of the
consumer should not exceed 244 V while the lowest voltage of the
consumer should not be less than 216 V.
6 October 2020 D.Rajesh Asst.Prof. EEE Department 84
 (ii) Availability of power on demand. Power must be
available to the consumers in any amount that they
may require from time to time.
 (iii) Reliability. Modern industry is almost dependent
on electric power for its operation.
 The reliability can be improved to a considerable extent
by
(a) interconnected system
(b) reliable automatic control system
(c) providing additional reserve facilities.
6 October 2020 D.Rajesh Asst.Prof. EEE Department 85
(i) Distributor fed at one end
(ii) Distributor fed at both ends
(iii) Distributor fed at the centre
(iv) Ring distributor.
6 October 2020 D.Rajesh Asst.Prof. EEE Department 86
 (i) Distributor fed
at one end
6 October 2020 D.Rajesh Asst.Prof. EEE Department 87
 (ii) Distributor
fed at both ends
 (iii) Distributor
fed at the centre
6 October 2020 D.Rajesh Asst.Prof. EEE Department 88
 (iv) Ring
distributor
(i)
Concentrated
loading
(ii) Uniform
loading
(iii) Both
concentrated
and uniform
loading.
6 October 2020 D.Rajesh Asst.Prof. EEE Department 89
 2-wire d.c. distributor AB fed at one end A and having concentrated loads 𝐼1,
𝐼2, 𝐼3 and 𝐼4 tapped off at points C, D, E and F respectively.
 Let 𝑟1 , 𝑟2, 𝑟3 𝑎𝑛𝑑 𝑟4 be the resistances of both wires (go and return) of the sections AC,
CD, DE and EF of the distributor respectively.
 Current fed from point A = 𝐼1 + 𝐼2 + 𝐼3 + 𝐼4 Current in section AC = 𝐼1 + 𝐼2 + 𝐼3 + 𝐼4
 Current in section CD = 𝐼2 + 𝐼3 + 𝐼4 Current in section DE = 𝐼3 + 𝐼4
 Current in section EF = 𝐼4
 Voltage drop in section AC = 𝑟1 (𝐼1 + 𝐼2 + 𝐼3 + 𝐼4)
 Voltage drop in section CD = 𝑟2 (𝐼2 + 𝐼3 + 𝐼4) Voltage drop in section DE = 𝑟3 (𝐼3 + 𝐼4)
 Voltage drop in section EF = 𝑟4 𝐼4
 ∴ Total voltage drop in the distributor
=𝑟1 (𝐼1 + 𝐼2 + 𝐼3 + 𝐼4)+𝑟2 (𝐼2 + 𝐼3 + 𝐼4)+𝑟3 (𝐼3 + 𝐼4)+ 𝑟4 𝐼4
6 October 2020 D.Rajesh Asst.Prof. EEE Department 90
 2-wire d.c. distributor AB fed at one end A and
loaded uniformly with i amperes per metre length.
 The load tapped is i amperes. Let 𝑙 metres be the
length of the distributor and r ohm be the
resistance per metre run.
6 October 2020 D.Rajesh Asst.Prof. EEE Department 91
 Consider a point C on the distributor at a distance x metres from the feeding point A
 Then current at point C is
= i 𝑙− i x amperes = i (𝑙− x) amperes
Now, consider a small length dx near point C. Its resistance is r dx and the voltage drop over
length dx is
dv = i (𝑙 − x) r dx = i r (𝑙 − x) dx
Total voltage drop in the distributor upto point C is
v =‫׬‬0
𝑥
ir (l − x)dx =ir 𝑙𝑥 −
𝑥2
2
The voltage drop upto point B (i.e. over the whole distributor) can be obtained by putting x = 𝑙 in
the above expression.
∴ Voltage drop over the distributor AB
= ir 𝑙 ∗ 𝑙 −
𝑙2
2
=
1
2
𝑖𝑟𝑙2
=
1
2
𝑖𝑙 𝑟𝑙 =
1
2
𝐼𝑅
Thus, in a uniformly loaded distributor fed at one end, the total voltage drop is equal to that
produced by the whole of the load assumed to be concentrated at the middle point.
6 October 2020 D.Rajesh Asst.Prof. EEE Department 92
 The two ends of the distributor may be supplied
with (i) equal voltages (ii) unequal voltages.
6 October 2020 D.Rajesh Asst.Prof. EEE Department 93
 Consider a distributor AB fed at both ends with equal voltages V volts and having
concentrated loads 𝐼1, 𝐼2, 𝐼3, 𝐼4 and 𝐼5 at points C, D, E, F and G respectively
 As we move away from one of the feeding points, say A, p.d. goes on decreasing till it
reaches the minimum value at some load point, say E, and then again starts rising and
becomes V volts as we reach the other feeding point B.
 All the currents tapped off between points A and E (minimum p.d. point) will be
supplied from the feeding point A while those tapped off between B and E will be
supplied from the feeding point B.
 The current tapped off at point E itself will be partly supplied from A and partly from
B. If these currents are x and y respectively, then,
x+y= 𝐼3
 Therefore, we arrive at a very important conclusion that at the point of minimum
potential, current comes from both ends of the distributor.
 Voltage drop from feeding point A to the point of minimum potential difference is
equal to the voltage drop from feeding point B to the point of minimum potential
difference
6 October 2020 D.Rajesh Asst.Prof. EEE Department 94
 Distributor AB fed with unequal voltages ; end A being fed at 𝑉1volts
and end B at 𝑉2 volts.
 In order to determine the point of minimum potential, assume of 𝐼𝐴
amperes is supplied from feeding point A
 Let r be the resistance of each conductor per unit length(l=1) for both go
& return
 Resistance of portions AC,CD, DE,EF,FB are 𝑟1 , 𝑟2, 𝑟3 , 𝑟4, 𝑎𝑛𝑑 𝑟5
respectively
 Voltage drop between A and B = Voltage drop over AB
𝑉1 − 𝑉2 = 𝐼𝐴𝑟1+(𝐼𝐴- 𝐼1) 𝑟2 + (𝐼𝐴- 𝐼1- 𝐼2) 𝑟3 + (𝐼𝐴- 𝐼1- 𝐼2- 𝐼3) 𝑟4+ (𝐼𝐴- 𝐼1- 𝐼2- 𝐼3 − 𝐼4) 𝑟5
The load point at which currents are coming from both sides of the
distributor will be the point of minimum potential
6 October 2020 D.Rajesh Asst.Prof. EEE Department 95
 A 2-wire d.c. street mains AB, 600 m long is fed from both ends at 220 V. Loads of 20
A, 40 A, 50 A and 30 A are tapped at distances of 100m, 250m, 400m and 500 m from
the end A respectively. If the area of X-section of distributor conductor is 1𝑐𝑚2
, find
the minimum consumer voltage. Take ρ = 1·7 × 10−6 Ω cm.
 Voltage drop between A and B = Voltage drop over AB
R= ρ l/A
 Resistance of section AC of distributor=2×
1·7 × 10−6∗100∗102
1
= 3·4 × 10−2 Ω
 Resistance of section CD of distributor =0.051 Ω
 Resistance of section DE of distributor =0.051 Ω
 Resistance of section EF of distributor =0.034 Ω
 Resistance of section FB of distributor =0.034 Ω
 Voltage drop between A and B = Voltage drop over AB
 𝑉𝐴 − 𝑉𝐵 = 𝐼𝐴𝑟1+(𝐼𝐴- 𝐼1) 𝑟2 + (𝐼𝐴- 𝐼1- 𝐼2) 𝑟3 + (𝐼𝐴- 𝐼1- 𝐼2- 𝐼3) 𝑟4+ (𝐼𝐴- 𝐼1- 𝐼2- 𝐼3 − 𝐼4) 𝑟5
 220-200=0.034 𝐼𝐴+(𝐼𝐴-20)0.051+ (𝐼𝐴-60)0.051+(𝐼𝐴-110)0.034+(𝐼𝐴-140)0.034
𝐼𝐴=61.7A
6 October 2020 D.Rajesh Asst.Prof. EEE Department 96
 Actual distribution of currents in the various sections of the
distributor is
 Current in section EF, 𝐼𝐸𝐹 = 𝐼𝐴−110 =61·7−110 =−48·3A from E to F
= 48·3 A from F to E
 Current in section DE, 𝐼𝐷𝐸 = 𝐼𝐴 −60=61.7-60=1.7A from D to E
6 October 2020 D.Rajesh Asst.Prof. EEE Department 97
It is clear that currents are coming to load point E from both sides i.e.
from point D and point F. Hence, E is the point of minimum potential.
∴ Minimum consumer voltage,
𝑉𝐸 = 𝑉𝐴-(𝐼𝐴𝐶 𝑟1+ 𝐼𝐶𝐷 𝑟2+ 𝐼𝐷𝐸 𝑟3)
=220-(61·7 × 0·034 + 41·7 × 0·051 + 1·7 × 0·051]
=215.69V
A two-wire d.c. distributor AB, 600 metres long is loaded as under :
Distance from A (metres) : 150 300 350 450
Loads in Amperes : 100 200 250 300
The feeding point A is maintained at 440 V and that of B at 430 V. If
each conductor has a resistance of 0·01 Ω per 100 metres, calculate :
(i) the currents supplied from A to B,
(ii) the power dissipated in the distributor.
Let 𝐼𝐴amperes be the current supplied from the feeding point A. Then currents in the
various sections of the distributor are
Resistance of 100 m length of distributor (both wires)
= 2 × 0·01 = 0·02 Ω
Resistance of section AC, 𝑟1 = 0·02 × 150/100 = 0·03 Ω
Resistance of section CD, 𝑟2 = 0·02 × 150/100 = 0·03 Ω
Resistance of section DE, 𝑟3= 0·02 × 50/100 = 0·01 Ω
Resistance of section EF, 𝑟4= 0·02 × 100/100 = 0·02 Ω
Resistance of section FB, 𝑟5= 0·02 × 150/100 = 0·03 Ω
6 October 2020 D.Rajesh Asst.Prof. EEE Department 98
 Voltage drop between A and B = Voltage drop over AB
 𝑉𝐴 − 𝑉𝐵 = 𝐼𝐴𝑟1+(𝐼𝐴- 𝐼1) 𝑟2 + (𝐼𝐴- 𝐼1- 𝐼2) 𝑟3 + (𝐼𝐴- 𝐼1- 𝐼2- 𝐼3) 𝑟4+ (𝐼𝐴- 𝐼1- 𝐼2- 𝐼3 − 𝐼4) 𝑟5
 440-430=0.03 𝐼𝐴+(𝐼𝐴-100)0.03+ (𝐼𝐴-300)0.01+(𝐼𝐴-550)0.02+(𝐼𝐴-850)0.03
𝐼𝐴=437.5A
 The actual distribution of currents in the various sections of the distributor are
 Current supplied from end A, 𝐼𝐴 = 𝐼𝐴𝐶 = 437·5 A
 Current supplied from end B, 𝐼𝐵 = 𝐼𝐹𝐵 = 𝐼𝐴 -850= - 412·5 A
 𝐼𝐶𝐷 = 337.5𝐴 𝐼𝐷𝐸 = 137.5𝐴 𝐼𝐸𝐹 = −112.5𝐴
 Therefore E is the point of minimum potential.
 Power loss in the distributor
=𝐼𝐴𝐶
2
𝑟1+ 𝐼𝐶𝐷
2
𝑟2 + 𝐼𝐷𝐸
2
𝑟3+ 𝐼𝐸𝐹
2
𝑟4+ 𝐼𝐹𝐵
2
𝑟5
=(437.5)2
∗ 0.03 + 337.5 2
∗ 0.03 + 137.5 2
∗ 0.01 + 112.5 2
∗ 0.02 + (412.5)2
∗ 0.03
=14.705kW
6 October 2020 D.Rajesh Asst.Prof. EEE Department 99
 (i) Distributor fed at both ends with equal voltages.
 Consider a distributor AB of length 𝑙 metres, having resistance r
ohms per metre run and with uniform loading of 𝑖 amperes per
metre run.
 Let the distributor be fed at the feeding points A and B at equal
voltages, say V volts.
 The total current supplied to the distributor is 𝑖𝑙.
 As the two end voltages are equal, therefore, current supplied from
each feeding point is
𝑖𝑙
2
i.e.
6 October 2020 D.Rajesh Asst.Prof. EEE Department 100
 Consider a point C at a distance x metres from the feeding point A. Then
current at point C is
=
𝑖𝑙
2
- 𝑖𝑥= 𝑖(
𝑙
2
-𝑥)
Now, consider a small length dx near point C. Its resistance is r dx and the
voltage drop over length dx is
dv = i (
𝑙
2
− x) r dx = i r (
𝑙
2
− x) dx
Total voltage drop in the distributor upto point C is
v =‫׬‬
0
𝑥
ir (
𝑙
2
− x)dx =ir
𝑙𝑥
2
−
𝑥2
2
=
𝑖𝑟
2
(𝑙𝑥- 𝑥2)
Obviously, the point of minimum potential will be the mid-point. Therefore,
maximum voltage drop will occur at mid-point i.e. where x =
𝑙
2
Max. voltage drop=
𝑖𝑟
2
(𝑙𝑥- 𝑥2)=
𝑖𝑟
2
(𝑙
𝑙
2
- (
𝑙
2
)2)=
1
8
𝑖𝑟𝑙2=
1
8
𝑖𝑙 𝑟𝑙 =
1
8
IR
Minimum voltage=V-
1
8
IR volts
6 October 2020 D.Rajesh Asst.Prof. EEE Department 101
 (ii) Distributor fed at both ends with unequal voltages.
 Consider a distributor AB of length l metres having resistance r
ohms per metre run and with a uniform loading of i amperes
permetre run. Let the distributor be fed from feeding points A and B
at voltages 𝑉𝐴 and 𝑉𝐵 respectively.
 Suppose that the point of minimum potential C is situated at a
distance x metres from the feeding point A. Then current supplied
by the feeding point A will be i x.
 ∴ Voltage drop in section AC=
1
2
𝑖𝑟𝑥2 volts
 As the distance of C from feeding point B is (l − x), therefore, current
fed from B is i (l − x).
 ∴ Voltage drop in section BC =
1
2
𝑖𝑟(𝑙 − 𝑥)2
volts
6 October 2020 D.Rajesh Asst.Prof. EEE Department 102
 Voltage at point C, 𝑉𝐶 = 𝑉𝐴 − Drop over AC
= 𝑉𝐴 -
1
2
𝑖𝑟𝑥2
Voltage at point C, 𝑉𝐶 = 𝑉𝐵 − Drop over BC
= 𝑉𝐵 -
1
2
𝑖𝑟(𝑙 − 𝑥)2
From above equations we get,
𝑉𝐴 -
1
2
𝑖𝑟𝑥2 = 𝑉𝐵 -
1
2
𝑖𝑟(𝑙 − 𝑥)2
𝑉𝐴- 𝑉𝐵 =
1
2
𝑖𝑟𝑥2
-
1
2
𝑖𝑟(𝑙 − 𝑥)2
=
1
2
𝑖𝑟𝑥2
-
1
2
𝑖𝑟𝑙2
-
1
2
𝑖𝑟𝑥2
+
1
2
𝑖𝑟2𝑙𝑥
= 𝑖𝑟𝑙 {𝑥-
1
2
𝑙 }
𝑥=
𝑉𝐴− 𝑉𝐵
𝑖𝑟𝑙
+
𝑙
2
The point on the distributor where minimum potential occurs can be calculated
6 October 2020 D.Rajesh Asst.Prof. EEE Department 103
 Expression for the power loss in a uniformly loaded distributor fed at both ends
 current supplied from each feeding point is
𝑖𝑙
2
 Consider a small length dx of the distributor at point P which is at a distance x from
the feeding end A.
 Resistance of length dx = r dx
 Current in length dx =
𝑖𝑙
2
- 𝑖𝑥= 𝑖(
𝑙
2
-𝑥)
 Power loss in length dx
= (current in dx)2 × Resistance of dx
= 𝑖(
𝑙
2
−𝑥)
2
×rdx
 Total power loss in the distributor is=‫׬‬
0
𝑙
𝑖(
𝑙
2
−𝑥)
2
×rdx
=𝑖2
𝑟 ‫׬‬
0
𝑙 𝑙2
4
− 𝑙𝑥 + 𝑥2
𝑑𝑥
= 𝑖2𝑟
𝑙2𝑥
4
−
𝑙𝑥2
2
+
𝑥3
3
𝑙
0
P=
𝑖2𝑟𝑙3
12
6 October 2020 D.Rajesh Asst.Prof. EEE Department 104
A 2-wire d.c. distributor AB 500 metres long is fed from both ends and is loaded uniformly
at the rate of 1·0 A/metre. At feeding point A, the voltage is maintained at 255 V and at B
at 250 V. If the resistance of each conductor is 0·1 Ω per kilometre, determine :
(i) the minimum voltage and the point where it occurs
(ii) the currents supplied from feeding points A and B
Let the minimum potential occur at a point C distant x metres from the feeding point A.
𝑥=
𝑉𝐴− 𝑉𝐵
𝑖𝑟𝑙
+
𝑙
2
Resistance of distributor/m, r = 2 × 0·1/1000 = 0·0002 Ω
𝑥=
255− 250
1∗0.0002∗500
+
500
2
=300m
i.e. minimum potential occurs at 300 m from point A.
Minimum voltage, 𝑉𝐶 = 𝑉𝐴 -
1
2
𝑖𝑟𝑥2 = 𝑉𝐵 -
1
2
𝑖𝑟(𝑙 − 𝑥)2
255-
1∗0.0002∗ (300)2
2
=246V
Current supplied from A = i x = 1 × 300 = 300 A
Current supplied from B = i (𝑙 − x) = 1 (500 − 300)
= 200 A
6 October 2020 D.Rajesh Asst.Prof. EEE Department 105
A 800 metres 2-wire d.c. distributor AB fed from both ends is uniformly
loaded at the rate of 1·25 A/metre run. Calculate the voltage at the
feeding points A and B if the minimum potential of 220 V occurs at
point C at a distance of 450 metres from the end A. Resistance of each
conductor is 0·05 Ω/km.
 Minimum voltage, 𝑉𝐶 = 𝑉𝐴 -
1
2
𝑖𝑟𝑥2
= 𝑉𝐵 -
1
2
𝑖𝑟(𝑙 − 𝑥)2
 For 𝑉𝐴
𝑉𝐶 = 𝑉𝐴 -
1
2
𝑖𝑟𝑥2
𝑉𝐴=220+
1.25∗0.0001∗ (450)2
2
=232.65V
 For 𝑉𝐵
𝑉𝐶 = 𝑉𝐵 -
1
2
𝑖𝑟(𝑙 − 𝑥)2
𝑉𝐵 =220+
1.25∗0.0001∗ (800−450)2
2
=227.65V
6 October 2020 D.Rajesh Asst.Prof. EEE Department 106
 (i) A uniformly loaded distributor is fed at the centre. Show that maximum
voltage drop = I R/8 where I is the total current fed to the distributor and R
is the total resistance of the distributor.
 (ii) A 2-wire d.c. distributor 1000 metres long is fed at the centre and is
loaded uniformly at the rate of 1·25 A/metre. If the resistance of each
conductor is 0·05 Ω/km, find the maximum voltage drop in the distributor.
 ∴ Max. voltage drop = Voltage drop in half distributor
=
1
2
(
𝑖𝑙
2
)(
𝑟𝑙
2
) =
1
8
𝑖𝑙 𝑟𝑙 =
1
8
IR
where i l = I, the total current fed to the distributor
r l = R, the total resistance of the distributor
 (ii) Total current fed to the distributor is
 I = i l = 1·25 × 1000 = 1250 A
 Total resistance of the distributor is
 R = r l = 2 × 0·05 × 1 = 0·1 Ω
 Max. voltage drop =
1
8
IR
=
1
8
*1250*0.1
=15.62V
6 October 2020 D.Rajesh Asst.Prof. EEE Department 107
 A two-wire d.c. distributor cable 1000 metres long is loaded with 0·5
A/metre. Resistance of each conductor is 0·05 Ω/km. Calculate the
maximum voltage drop if the distributor is fed from both ends with
equal voltages of 220 V. What is the minimum voltage and where it
occurs ?
 Current loading, i = 0·5 A/m
 Resistance of distributor/m, r = 2 × 0·05/1000 = 0·1 × 10−3
Ω
 Length of distributor, l = 1000 m
 Total current supplied by distributor, I = i l = 0·5 × 1000 = 500 A
 Total resistance of the distributor, R = r l = 0·1 × 10−3 × 1000 = 0·1 Ω
 ∴ Max. voltage drop =
1
8
IR
= 6·25 V
 Minimum voltage will occur at the mid-point of the distributor and
its value is
 = 220 − 6·25 = 213·75 V
6 October 2020 D.Rajesh Asst.Prof. EEE Department 108
The total drop over any section of the distributor
is equal to the sum of drops due to concentrated
and uniform loading in that section.
6 October 2020 D.Rajesh Asst.Prof. EEE Department 109
Two conductors of a d.c. distributor cable AB 1000 m long have a total
resistance of 0·1 Ω. The ends A and B are fed at 240 V. The cable is
uniformly loaded at 0·5 A per metre length and has concentrated loads
of 120 A, 60 A, 100 A and 40 A at points distant 200 m, 400 m, 700 m and
900 m respectively from the end A. Calculate (i) the point of minimum
potential (ii) currents supplied from ends A and B (iii) the value of
minimum potential.
Distributor resistance per metre length, r = 0·1/1000 = 10−4
Ω
(i) Point of minimum potential. The point of minimum potential is not
affected by the uniform loading of the distributor. Therefore, let us
consider the concentrated loads.
Suppose the current supplied by end A is 𝐼𝐴. Then currents in the
various sections will be
6 October 2020 D.Rajesh Asst.Prof. EEE Department 110
 Voltage drop between A and B = Voltage drop over AB
 𝑉𝐴 − 𝑉𝐵 = 𝐼𝐴𝑟1+(𝐼𝐴- 𝐼1) 𝑟2 + (𝐼𝐴- 𝐼1- 𝐼2) 𝑟3 + (𝐼𝐴- 𝐼1- 𝐼2- 𝐼3)
𝑟4+ (𝐼𝐴- 𝐼1- 𝐼2- 𝐼3 − 𝐼4) 𝑟5
 0= 10−4(
)
200 𝐼𝐴+(𝐼𝐴−120)200+
(𝐼𝐴−180)300+(𝐼𝐴−280)200+(𝐼𝐴−320)100
𝐼𝐴=166A
The actual distribution of currents in the various sections of the
distributor due to concentrated loading.
It is clear from this figure that D is the point of minimum
potential.
6 October 2020 D.Rajesh Asst.Prof. EEE Department 111
(ii) The feeding point A will supply 166 A due to concentrated loading
plus 0·5 × 400 = 200 A due to uniform loading.
∴ Current supplied byA, 𝐼𝐴 = 166 + 200 = 366 A
The feeding point B will supply a current of 154 A due to concentrated
loading plus 0·5 × 600 = 300 A due to uniform loading.
∴ Current supplied by B, 𝐼𝐵 = 154 + 300 = 454 A
(iii) ∴ Minimum potential, 𝑉𝐷 = 𝑉𝐴 − Drop in AD due to conc. loading –
Drop in AD due to uniform loading 𝑉𝐷
Now, Drop in AD due to conc. loading = 𝐼𝐴𝐶 𝑅𝐴𝐶 + 𝐼𝐶𝐷 𝑅𝐶𝐷
= 166 × 10−4
× 200 + 46 × 10−4
× 200
= 3·32 + 0·92 = 4·24 V
Drop in AD due to uniform loading =
1
2
𝑖𝑟𝑙2
=
1
2
0.5 ∗ 10−4
∗ (400)2
=4V
∴ 𝑉𝐷 = 240 − 4·24 − 4 = 231·76 V
6 October 2020 D.Rajesh Asst.Prof. EEE Department 112
A d.c. 2-wire distributor AB is 500m long and is fed at
both ends at 240 V. The distributor is loaded as shown
in Fig. The resistance of the distributor (go and return)
is 0·001Ω per metre. Calculate (i) the point of minimum
voltage and (ii) the value of this voltage.
Let D be the point of minimum potential and let x be the
current flowing in section CD. Then current supplied by
end B will be (60 − x).
6 October 2020 D.Rajesh Asst.Prof. EEE Department 113
 (i) If r is the resistance of the distributor (go and return) per metre length,
then,
Voltage drop in length AD = 𝐼𝐴𝐶 𝑅𝐴𝐶 + 𝐼𝐶𝐷 𝑅𝐶𝐷
=(100+x)*100r+x*150r
Voltage drop in length BD=
1
2
𝑖𝑟𝑙2
+(60-x)*250r
=
1
2
1 ∗ 𝑟 ∗ (200)2 +(60-x)*250r
Since the feeding points A and B are at the same potential
(100+x)*100r+x*150r=
1
2
1 ∗ 𝑟 ∗ (200)2 +(60-x)*250r
X=50A
The actual directions of currents in the various sections of the distributor are
currents supplied by A and B meet at D. Hence point D is the point of minimum
6 October 2020 D.Rajesh Asst.Prof. EEE Department 114
 (ii) Total current = 160 + 1 × 200 = 360 A
 Current supplied by A, 𝐼𝐴 = 100 + x = 100 + 50 = 150 A
 Current supplied by B, 𝐼𝐵 = 360 − 150 = 210 A
 Minimum potential, 𝑉𝐷 = 𝑉𝐴 − (𝐼𝐴𝐶 𝑅𝐴𝐶 + 𝐼𝐶𝐷 𝑅𝐶𝐷 )
= 240 − 150 × (100 × 0·001) − 50 × (150 × 0·001)
= 240 − 15 − 7·5 = 217·5 V
6 October 2020 D.Rajesh Asst.Prof. EEE Department 115
 A distributor arranged to form a closed loop and
fed at one or more points is called a ring
distributor.
 For the purpose of calculating voltage distribution,
the distributor can be considered as consisting of a
series of open distributors fed at both ends.
 The principal advantage of ring distributor is that
by proper choice in the number of feeding points,
great economy in copper can be affected.
6 October 2020 D.Rajesh Asst.Prof. EEE Department 116
 Sometimes a ring distributor has to
serve a large area. In such a case,
voltage drops in the various
sections of the distributor may
become excessive.
 In order to reduce voltage drops in
various sections, distant points of
the distributor are joined through a
conductor called interconnector.
6 October 2020 D.Rajesh Asst.Prof. EEE Department 117
 The ring distributor ABCDEA. The points B and D of the ring distributor are joined
through an interconnector BD.
 Network can be obtained by applying Thevenin’s theorem.
 (i) Consider the interconnector BD to be disconnected and find the potential difference
between B and D. This gives Thevenin’s equivalent circuit voltage 𝐸0.
 (ii) Next, calculate the resistance viewed from points B and D of the network composed
of distribution lines only. This gives Thevenin’s equivalent circuit series resistance 𝑅0.
 (iii) If 𝑅𝐵𝐷 is the resistance of the interconnector BD, then Thevenin’s equivalent circuit
will be
∴ Current in interconnector BD =
𝐸0
𝑅0+𝑅𝐵𝐷
 Therefore, current distribution in each section and the voltage of load points can be
calculated.
6 October 2020 D.Rajesh Asst.Prof. EEE Department 118
A 2-wire d.c. distributor ABCDEA in the form of a ring main is fed at
point A at 220 V and is loaded as under :
10A at B ; 20A at C ; 30A at D and 10 A at E.
The resistances of various sections (go and return) are : AB = 0·1 Ω ; BC
= 0·05 Ω ; CD = 0·01 Ω ; DE = 0·025 Ω and EA = 0·075 Ω. Determine :
 (i) the point of minimum potential
 (ii) current in each section of distributor
Let us suppose that current I flows in section AB of the distributor. Then
currents in the various sections of the distributor are
According to Kirchhoff’s voltage law, the voltage drop in the closed
loop ABCDEA is zero i.e.
𝐼𝐴𝐵𝑅𝐴𝐵+𝐼𝐵𝐶𝑅𝐵𝐶+ 𝐼𝐶𝐷𝑅𝐶𝐷+ 𝐼𝐷𝐸𝑅𝐷𝐸 + 𝐼𝐸𝐴𝑅𝐸𝐴 =0
0.1𝐼+(𝐼-10)0.05+ (𝐼-30)0.01+(𝐼-60)0.025+(𝐼-70)0.075=0
0.26 𝐼 = 7.55
𝐼 = 29.04A
6 October 2020 D.Rajesh Asst.Prof. EEE Department 119
 (ii) Current in section AB = I = 29·04 A from A to B
 Current in section BC = I − 10 = 29·04 − 10 = 19·04 A from B to C
 Current in section CD = I − 30 = 29·04 − 30 = − 0·96 A = 0·96 A from D to C
 Current in section DE = I − 60 = 29·04 − 60 = − 30·96 A = 30·96 A from E to D
 Current in section EA = I − 70 = 29·04 − 70 = − 40·96 A = 40·96 A from A to E
6 October 2020 D.Rajesh Asst.Prof. EEE Department 120
∴ C is the point of minimum potential.
 A d.c. ring main ABCDA is fed from point A from a 250 V supply and the resistances
(including both lead and return) of various sections are as follows : AB = 0·02 Ω ; BC =
0·018 Ω ; CD = 0·025 Ω and DA = 0·02 Ω. The main supplies loads of 150 A at B ; 300 A
at C and 250 A at D. Determine the voltage at each load point. If the points A and C are
linked through an interconnector of resistance 0·02 Ω, determine the new voltage at
each load point.
 Without Interconnector.
 Let us suppose that a current I flows in section AB of the distributor. Then currents in
various sections of the distributor will be
 According to Kirchhoff’s voltage law, the voltage drop in the closed loop ABCDA is
zero i.e.
 𝐼𝐴𝐵𝑅𝐴𝐵+𝐼𝐵𝐶𝑅𝐵𝐶+ 𝐼𝐶𝐷𝑅𝐶𝐷+ 𝐼𝐷𝐴𝑅𝐷𝐴 =0
 0.02𝐼+(𝐼-150)0.018+ (𝐼-450)0.025+(𝐼-700)0.02=0
0·083 I = 27·95
∴ I = 27·95/0·083 = 336·75 A
6 October 2020 D.Rajesh Asst.Prof. EEE Department 121
The actual distribution of currents are ..
 Current in section AB = I = 336.75 A from A to B
 Current in section BC = I − 150 = 336.75 − 150 = 186.75 A from B to C
 Current in section CD = I − 450 = 336.75 − 450 = − 113.25 A = 113.25 A from D to C
 Current in section DA = I − 700 = 336.75 − 700 = − 363.25A = 363.25 A from A to D
Voltage drop in AB = 336·75 × 0·02 = 6·735 V
Voltage drop in BC = 186·75 × 0·018 = 3·361 V
Voltage drop in CD = 113·25 × 0·025 = 2·831 V
Voltage drop in DA = 363·25 × 0·02 = 7·265 V
∴ Voltage at point B = 250 − 6·735 = 243·265 V
Voltage at point C = 243·265 − 3·361 = 239·904 V
Voltage at point D = 239·904 + 2·831 = 242·735 V
6 October 2020 D.Rajesh Asst.Prof. EEE Department 122
 With Interconnector.
 The ring distributor with interconnector AC. The current in the interconnector can be
found by applying Thevenin’s theorem.
𝐸0 = Voltage between points A and C
= 250 − 239·904 = 10·096 V
𝑅0 = Resistance viewed from points A and C
=
0.02+0.018 0.02+0.025
0.02+0.018 + 0.02+0.025
= 0.02Ω
𝑅𝐴𝐶 = Resistance of interconnector = 0·02 Ω
 From Thevenin’s equivalent circuit Current in interconnector AC is
𝐸0
𝑅0 + 𝑅𝐴𝐶
=
10.096
0.02 + 0.02
= 252.4𝐴 𝑓𝑟𝑜𝑚 𝐴 𝑡𝑜 𝐶
Let us suppose that current in section AB is 𝐼1 . Then current in section BC will be 𝐼1 − 150.
As the voltage drop round the closed mesh ABCA is zero,
0.02𝐼1+(𝐼1-150)0.018-0.02*252.4=0
0.038 𝐼1=7.748
𝐼1=203.15A
6 October 2020 D.Rajesh Asst.Prof. EEE Department 123
The actual distribution of currents are ..
 Current in section AB = 𝐼1 = 203.15A from A to B
 Current in section BC = 𝐼1 − 150 = 203.15 − 150 = 53.15 A from B to C
 Current in section CD = 𝐼1 − 450+252.4 = 203.15− 450 +252.4= 5.55A C to D
 Current in section DA = 𝐼1 − 700 = 203.15+252.5 − 700 = − 244.45A = 244.45A
from A to D
 Drop in AB = 203·15 × 0·02 = 4·063 V
 Drop in BC = 53·15 × 0·018= 0·960 V
 Drop in AD = 244·45 × 0·02 = 4·9 V
 ∴ Potential of B = 250 − 4·063= 245·93 V
 Potential of C = 245·93 − 0·96 = 244·97 V
 Potential of D = 250 − 4·9 = 245·1 V
It may be seen that with the use of interconnector, the voltage drops in the various
sections of the distributor are reduced.
6 October 2020 D.Rajesh Asst.Prof. EEE Department 124
 A.C. distribution calculations differ from those of d.c. distribution in the
following respects :
 (i) In case of d.c. system, the voltage drop is due to resistance alone.
However, in a.c. system, the voltage drops are due to the combined effects of
resistance, inductance and capacitance.
 (ii) In a d.c. system, additions and subtractions of currents or voltages are
done arithmetically but in case of a.c. system, these operations are done
vectorially.
 (iii) In an a.c. system, power factor (p.f.) has to be taken into account. Loads
tapped off form the distributor are generally at different power factors. There
are two ways of referring power factor viz
(a) It may be referred to supply or receiving end voltage which is
regarded as the reference vector.
(b) It may be referred to the voltage at the load point itself.
6 October 2020 D.Rajesh Asst.Prof. EEE Department 125
 The power factors of load currents may be
given
(i) w.r.t. receiving or sending end voltage
or
(ii) w.r.t. to load voltage itself.
6 October 2020 D.Rajesh Asst.Prof. EEE Department 126
 (i) Power factors referred to receiving end voltage.
Consider an a.c. distributor AB with concentrated loads of 𝐼1and 𝐼2 tapped off at
points C and B. Taking the receiving end voltage 𝑉𝐵 as the reference vector, let
lagging power factors at C and B be cos φ1 and cos φ2 w.r.t. 𝑉𝐵 . Let 𝑅1, 𝑋1 and
𝑅2, 𝑋2 be the resistance and reactance of sections AC and CB of the distributor.
Impedance of section AC, 𝑍𝐴𝐶= 𝑅1 + j 𝑋1
Impedance of section CB,𝑍𝐶𝐵 = 𝑅2 + j 𝑋2
Load current at point C, 𝐼1 = 𝐼1 (cos φ1 − j sin φ1)
Load current at point B, 𝐼2 = 𝐼2 (cos φ2 − j sin φ2)
Current in section CB, 𝐼𝐶𝐵= 𝐼2 = 𝐼2 (cos φ2 − j sin φ2)
Current in section AC, 𝐼𝐴𝐶 = 𝐼1+ 𝐼2
=𝐼1 (cos φ1 − j sin φ1)+𝐼2 (cos φ2 − j sin φ2)
6 October 2020 D.Rajesh Asst.Prof. EEE Department 127
φ1
 Voltage drop in section CB,
 𝑉𝐶𝐵= 𝐼𝐶𝐵 𝑍𝐶𝐵= 𝐼2 (cos φ2 − j sin φ2) (𝑅2 + j 𝑋2 )
 Voltage drop in section AC,
 𝑉𝐴𝐶= 𝐼𝐴𝐶 𝑍𝐴𝐶=(𝐼1+ 𝐼2) 𝑍𝐴𝐶
 =𝐼1 (cos φ1 − j sin φ1)+𝐼2 (cos φ2 − j sin φ2)(𝑅1 + j 𝑋1 )
 Sending end voltage, 𝑉𝐴= 𝑉𝐵 + 𝑉𝐶𝐵 + 𝑉𝐴𝐶
 Sending end current, 𝐼𝐴= 𝐼1+ 𝐼2
6 October 2020 D.Rajesh Asst.Prof. EEE Department 128
𝑉𝑩
𝐼𝟐
𝐼𝟏
𝐼𝑨𝑪
φ2
θ
φ1
 (ii) Power factors referred to respective load voltages.
 Suppose the power factors of loads are referred to their respective load voltages. Then φ1 is the phase angle
between V𝐶 and I1 and φ2 is the phase angle between V𝐵 and I2. The vector diagram under these conditions is
 Voltage drop in section CB,
 𝑉𝐶𝐵= 𝐼2 𝑍𝐶𝐵= 𝐼2 (cos φ2 − j sin φ2) (𝑅2 + j 𝑋2 )
 Voltage at point C = 𝑉𝐵 + Drop in section CB = V𝐶 ∠ α (say)
 Now 𝐼1 = 𝐼1 ∠ − φ1 w.r.t. voltage V𝐶
 ∴ . 𝐼1 = 𝐼1 ∠ − (φ1 − α) w.r.t. voltage V𝐵
 i.e. 𝐼1 = 𝐼1 [cos (φ1 − α) − j sin (φ1 − α)]
 Now 𝐼𝐴𝐶 = 𝐼1+ 𝐼2
=𝐼1 [cos (φ1 − α) − j sin (φ1 − α)]+ 𝐼2 (cos φ2 − j sin φ2)
 Voltage drop in section AC,
𝑉𝐴𝐶= 𝐼𝐴𝐶 𝑍𝐴𝐶
 ∴ Voltage at point A = V𝐵 + Drop in CB + Drop in AC
6 October 2020 D.Rajesh Asst.Prof. EEE Department 129
𝑉𝑩
𝐼𝟐
𝐼𝟏
𝐼𝑨𝑪
φ2
α
A single phase a.c. distributor AB 300 metres long is fed from end A and is loaded as under :
(i) 100 A at 0·707 p.f. lagging 200 m from point A (ii) 200 A at 0·8 p.f. lagging 300 m from point A
The load resistance and reactance of the distributor is 0·2 Ω and 0·1 Ω per kilometre. Calculate the
total voltage drop in the distributor. The load power factors refer to the voltage at the far end.
Impedance of distributor/km = (0·2 + j 0·1) Ω
Impedance of section AC, 𝑍𝐴𝐶 = (0·2 + j 0·1) × 200/1000 = (0·04 + j 0·02) Ω
Impedance of section CB, 𝑍𝐶𝐵 = (0·2 + j 0·1) × 100/1000 = (0·02 + j 0·01) Ω
Taking voltage at the far end B as the reference vector, we have, Load current at point B,
𝐼2 = 𝐼2 (cos φ2 − j sin φ2) = 200 (0·8 − j 0·6) = (160 − j 120) A
Load current at point C, 𝐼1 = 𝐼1 (cos φ1 − j sin φ1) = 100 (0·707 − j 0·707) = (70·7− j 70·7) A
Current in section CB,𝐼𝐶𝐵= 𝐼2 = (160 − j 120) A
Current in section AC,𝐼𝐴𝐶= 𝐼1+ 𝐼2
= (70·7 − j 70·7) + (160 − j 120)= (230·7 − j 190·7) A
Voltage drop in section CB, 𝑉𝐶𝐵= 𝐼𝐶𝐵 𝑍𝐶𝐵 = (160 − j 120) (0·02 + j 0·01) = (4·4 − j 0·8) volts
Voltage drop in section AC, 𝑉𝐴𝐶= 𝐼𝐴𝐶 𝑍𝐴𝐶 = (230·7 − j 190·7) (0·04 + j 0·02) = (13·04 − j 3·01) volts
Voltage drop in the distributor = 𝑉𝐴𝐶 + 𝑉𝐶𝐵 = (13·04 − j 3·01) + (4·4 − j 0·8) = (17·44 − j 3·81) volts
Magnitude of drop = 17.44 2 + 3.81 2 = 17·85 V
6 October 2020 D.Rajesh Asst.Prof. EEE Department 130
 A single phase ring distributor ABC is fed at A. The loads at B
and C are 20 A at 0.8 p.f. lagging and 15 A at 0.6 p.f. lagging
respectively ; both expressed with reference to the voltage at
A. The total impedance of the three sections AB, BC and CA
are (1 + j 1), (1+ j2) and (1 + j3) ohms respectively. Find the
total current fed at A and the current in each section. Use
Thevenin’s theorem to obtain the results.
6 October 2020 D.Rajesh Asst.Prof. EEE Department 131

More Related Content

What's hot

Chapter 4 mechanical design of transmission lines
Chapter 4  mechanical design of transmission linesChapter 4  mechanical design of transmission lines
Chapter 4 mechanical design of transmission lines
firaoltemesgen1
 
Power system slide notes
Power system slide notesPower system slide notes
Power system slide notes
michaeljmack
 
ELECTRICAL POWER SYSTEM - II. symmetrical three phase faults. PREPARED BY : J...
ELECTRICAL POWER SYSTEM - II. symmetrical three phase faults. PREPARED BY : J...ELECTRICAL POWER SYSTEM - II. symmetrical three phase faults. PREPARED BY : J...
ELECTRICAL POWER SYSTEM - II. symmetrical three phase faults. PREPARED BY : J...
Jobin Abraham
 
Lecture 09 transmission lines
Lecture 09   transmission linesLecture 09   transmission lines
Lecture 09 transmission lines
bhaavan22
 
Tap changer
Tap changerTap changer
Tap changer
vivek1292
 
Electromagnetic relay
Electromagnetic relayElectromagnetic relay
Electromagnetic relay
jawaharramaya
 
Electrical transmission line
Electrical transmission lineElectrical transmission line
Electrical transmission line
Dhananjay Jha
 
Inductance of transmission line
Inductance of transmission lineInductance of transmission line
Inductance of transmission line
Anisur Rahman
 
Comparison of ac and dc transmission
Comparison of ac and dc transmissionComparison of ac and dc transmission
Comparison of ac and dc transmission
jawaharramaya
 
Two ports
Two ports Two ports
Two ports
Selomon birhane
 
CURRENT LIMITING REACTORS
CURRENT  LIMITING  REACTORSCURRENT  LIMITING  REACTORS
CURRENT LIMITING REACTORS
BhashiniPriyadas
 
Scr firing circuits
Scr firing circuitsScr firing circuits
Scr firing circuits
Vinod Srivastava
 
Symmetrical Components Fault Calculations
Symmetrical Components Fault CalculationsSymmetrical Components Fault Calculations
Symmetrical Components Fault Calculations
michaeljmack
 
Three phase-circuits
Three phase-circuitsThree phase-circuits
Three phase-circuits
rsamurti
 
Sag in overhead transmission line, sag calculation & string chart
Sag in overhead transmission line, sag   calculation & string chartSag in overhead transmission line, sag   calculation & string chart
Sag in overhead transmission line, sag calculation & string chart
vishalgohel12195
 
Power systems symmetrical components
Power systems symmetrical componentsPower systems symmetrical components
Power systems symmetrical components
anoopeluvathingal
 
Single phase transformers
Single phase transformersSingle phase transformers
Single phase transformers
Ekeeda
 
Inductance and capacitance
Inductance and capacitanceInductance and capacitance
Inductance and capacitance
GHSKassKoronaMardan
 
Voltage Drop Calculation.pdf
Voltage Drop Calculation.pdfVoltage Drop Calculation.pdf
Voltage Drop Calculation.pdf
MesiasJohnFederico
 
PPT Fault Analysis
PPT Fault AnalysisPPT Fault Analysis
PPT Fault Analysis
Ankitkumaralwariya
 

What's hot (20)

Chapter 4 mechanical design of transmission lines
Chapter 4  mechanical design of transmission linesChapter 4  mechanical design of transmission lines
Chapter 4 mechanical design of transmission lines
 
Power system slide notes
Power system slide notesPower system slide notes
Power system slide notes
 
ELECTRICAL POWER SYSTEM - II. symmetrical three phase faults. PREPARED BY : J...
ELECTRICAL POWER SYSTEM - II. symmetrical three phase faults. PREPARED BY : J...ELECTRICAL POWER SYSTEM - II. symmetrical three phase faults. PREPARED BY : J...
ELECTRICAL POWER SYSTEM - II. symmetrical three phase faults. PREPARED BY : J...
 
Lecture 09 transmission lines
Lecture 09   transmission linesLecture 09   transmission lines
Lecture 09 transmission lines
 
Tap changer
Tap changerTap changer
Tap changer
 
Electromagnetic relay
Electromagnetic relayElectromagnetic relay
Electromagnetic relay
 
Electrical transmission line
Electrical transmission lineElectrical transmission line
Electrical transmission line
 
Inductance of transmission line
Inductance of transmission lineInductance of transmission line
Inductance of transmission line
 
Comparison of ac and dc transmission
Comparison of ac and dc transmissionComparison of ac and dc transmission
Comparison of ac and dc transmission
 
Two ports
Two ports Two ports
Two ports
 
CURRENT LIMITING REACTORS
CURRENT  LIMITING  REACTORSCURRENT  LIMITING  REACTORS
CURRENT LIMITING REACTORS
 
Scr firing circuits
Scr firing circuitsScr firing circuits
Scr firing circuits
 
Symmetrical Components Fault Calculations
Symmetrical Components Fault CalculationsSymmetrical Components Fault Calculations
Symmetrical Components Fault Calculations
 
Three phase-circuits
Three phase-circuitsThree phase-circuits
Three phase-circuits
 
Sag in overhead transmission line, sag calculation & string chart
Sag in overhead transmission line, sag   calculation & string chartSag in overhead transmission line, sag   calculation & string chart
Sag in overhead transmission line, sag calculation & string chart
 
Power systems symmetrical components
Power systems symmetrical componentsPower systems symmetrical components
Power systems symmetrical components
 
Single phase transformers
Single phase transformersSingle phase transformers
Single phase transformers
 
Inductance and capacitance
Inductance and capacitanceInductance and capacitance
Inductance and capacitance
 
Voltage Drop Calculation.pdf
Voltage Drop Calculation.pdfVoltage Drop Calculation.pdf
Voltage Drop Calculation.pdf
 
PPT Fault Analysis
PPT Fault AnalysisPPT Fault Analysis
PPT Fault Analysis
 

Similar to POWER SUPPLY SYSTEMS& DISTRIBUTION SYSTEMS

INDUCTANCE & CAPACITANCE CALCULATIONS
INDUCTANCE & CAPACITANCE CALCULATIONSINDUCTANCE & CAPACITANCE CALCULATIONS
INDUCTANCE & CAPACITANCE CALCULATIONS
Duddu Rajesh
 
Lecture 6
Lecture 6Lecture 6
Lecture 6
Forward2025
 
Lecture 09 em transmission lines
Lecture 09   em transmission linesLecture 09   em transmission lines
Lecture 09 em transmission lines
Amit Rastogi
 
Application of Gauss' Law
Application of Gauss' LawApplication of Gauss' Law
Application of Gauss' Law
RCC Institute of Information Technology
 
ECE203 Lecture 3,4
ECE203 Lecture 3,4 ECE203 Lecture 3,4
ECE203 Lecture 3,4
Jeya Prakash K
 
Ix2616991704
Ix2616991704Ix2616991704
Ix2616991704
IJERA Editor
 
CLASS X SCIENCE STUDY MATERIAL
CLASS X SCIENCE STUDY MATERIALCLASS X SCIENCE STUDY MATERIAL
CLASS X SCIENCE STUDY MATERIAL
Rc Os
 
Edc mid i question and answers
Edc mid i question and answersEdc mid i question and answers
Edc mid i question and answers
Aluru Jaideep Reddy
 
Threshold voltage model for hetero-gate-dielectric tunneling field effect tra...
Threshold voltage model for hetero-gate-dielectric tunneling field effect tra...Threshold voltage model for hetero-gate-dielectric tunneling field effect tra...
Threshold voltage model for hetero-gate-dielectric tunneling field effect tra...
IJECEIAES
 
Electrical Power Supply System
Electrical Power Supply SystemElectrical Power Supply System
Electrical Power Supply System
NAYEMAL
 
05617901
0561790105617901
Ee8402 inductance calculation
Ee8402 inductance calculationEe8402 inductance calculation
Ee8402 inductance calculation
Sugavanam KR
 
B044040916
B044040916B044040916
B044040916
IJERA Editor
 
Kinetics of X-ray conductivity for an ideal wide-gap semiconductor irradiated...
Kinetics of X-ray conductivity for an ideal wide-gap semiconductor irradiated...Kinetics of X-ray conductivity for an ideal wide-gap semiconductor irradiated...
Kinetics of X-ray conductivity for an ideal wide-gap semiconductor irradiated...
Andrii Sofiienko
 
Class 12 Cbse Physics Sample Paper 2013 Model 2
Class 12 Cbse Physics Sample Paper 2013 Model 2Class 12 Cbse Physics Sample Paper 2013 Model 2
Class 12 Cbse Physics Sample Paper 2013 Model 2
Sunaina Rawat
 
29 sambugari anil kumar--298-309
29 sambugari anil kumar--298-30929 sambugari anil kumar--298-309
29 sambugari anil kumar--298-309
Alexander Decker
 
Lecture 5
Lecture 5Lecture 5
Lecture 5
Forward2025
 
EE504.2.ppt
EE504.2.pptEE504.2.ppt
EE504.2.ppt
Vara Prasad
 
Capacitor
CapacitorCapacitor
CURRENT ELECTRICITY/ELECTROSTATICS FOR CBSE FREE REVISION SHEET BY ANURAG TY...
CURRENT ELECTRICITY/ELECTROSTATICS FOR CBSE FREE REVISION SHEET  BY ANURAG TY...CURRENT ELECTRICITY/ELECTROSTATICS FOR CBSE FREE REVISION SHEET  BY ANURAG TY...
CURRENT ELECTRICITY/ELECTROSTATICS FOR CBSE FREE REVISION SHEET BY ANURAG TY...
ANURAG TYAGI CLASSES (ATC)
 

Similar to POWER SUPPLY SYSTEMS& DISTRIBUTION SYSTEMS (20)

INDUCTANCE & CAPACITANCE CALCULATIONS
INDUCTANCE & CAPACITANCE CALCULATIONSINDUCTANCE & CAPACITANCE CALCULATIONS
INDUCTANCE & CAPACITANCE CALCULATIONS
 
Lecture 6
Lecture 6Lecture 6
Lecture 6
 
Lecture 09 em transmission lines
Lecture 09   em transmission linesLecture 09   em transmission lines
Lecture 09 em transmission lines
 
Application of Gauss' Law
Application of Gauss' LawApplication of Gauss' Law
Application of Gauss' Law
 
ECE203 Lecture 3,4
ECE203 Lecture 3,4 ECE203 Lecture 3,4
ECE203 Lecture 3,4
 
Ix2616991704
Ix2616991704Ix2616991704
Ix2616991704
 
CLASS X SCIENCE STUDY MATERIAL
CLASS X SCIENCE STUDY MATERIALCLASS X SCIENCE STUDY MATERIAL
CLASS X SCIENCE STUDY MATERIAL
 
Edc mid i question and answers
Edc mid i question and answersEdc mid i question and answers
Edc mid i question and answers
 
Threshold voltage model for hetero-gate-dielectric tunneling field effect tra...
Threshold voltage model for hetero-gate-dielectric tunneling field effect tra...Threshold voltage model for hetero-gate-dielectric tunneling field effect tra...
Threshold voltage model for hetero-gate-dielectric tunneling field effect tra...
 
Electrical Power Supply System
Electrical Power Supply SystemElectrical Power Supply System
Electrical Power Supply System
 
05617901
0561790105617901
05617901
 
Ee8402 inductance calculation
Ee8402 inductance calculationEe8402 inductance calculation
Ee8402 inductance calculation
 
B044040916
B044040916B044040916
B044040916
 
Kinetics of X-ray conductivity for an ideal wide-gap semiconductor irradiated...
Kinetics of X-ray conductivity for an ideal wide-gap semiconductor irradiated...Kinetics of X-ray conductivity for an ideal wide-gap semiconductor irradiated...
Kinetics of X-ray conductivity for an ideal wide-gap semiconductor irradiated...
 
Class 12 Cbse Physics Sample Paper 2013 Model 2
Class 12 Cbse Physics Sample Paper 2013 Model 2Class 12 Cbse Physics Sample Paper 2013 Model 2
Class 12 Cbse Physics Sample Paper 2013 Model 2
 
29 sambugari anil kumar--298-309
29 sambugari anil kumar--298-30929 sambugari anil kumar--298-309
29 sambugari anil kumar--298-309
 
Lecture 5
Lecture 5Lecture 5
Lecture 5
 
EE504.2.ppt
EE504.2.pptEE504.2.ppt
EE504.2.ppt
 
Capacitor
CapacitorCapacitor
Capacitor
 
CURRENT ELECTRICITY/ELECTROSTATICS FOR CBSE FREE REVISION SHEET BY ANURAG TY...
CURRENT ELECTRICITY/ELECTROSTATICS FOR CBSE FREE REVISION SHEET  BY ANURAG TY...CURRENT ELECTRICITY/ELECTROSTATICS FOR CBSE FREE REVISION SHEET  BY ANURAG TY...
CURRENT ELECTRICITY/ELECTROSTATICS FOR CBSE FREE REVISION SHEET BY ANURAG TY...
 

Recently uploaded

Heat Resistant Concrete Presentation ppt
Heat Resistant Concrete Presentation pptHeat Resistant Concrete Presentation ppt
Heat Resistant Concrete Presentation ppt
mamunhossenbd75
 
Textile Chemical Processing and Dyeing.pdf
Textile Chemical Processing and Dyeing.pdfTextile Chemical Processing and Dyeing.pdf
Textile Chemical Processing and Dyeing.pdf
NazakatAliKhoso2
 
Recycled Concrete Aggregate in Construction Part II
Recycled Concrete Aggregate in Construction Part IIRecycled Concrete Aggregate in Construction Part II
Recycled Concrete Aggregate in Construction Part II
Aditya Rajan Patra
 
Comparative analysis between traditional aquaponics and reconstructed aquapon...
Comparative analysis between traditional aquaponics and reconstructed aquapon...Comparative analysis between traditional aquaponics and reconstructed aquapon...
Comparative analysis between traditional aquaponics and reconstructed aquapon...
bijceesjournal
 
官方认证美国密歇根州立大学毕业证学位证书原版一模一样
官方认证美国密歇根州立大学毕业证学位证书原版一模一样官方认证美国密歇根州立大学毕业证学位证书原版一模一样
官方认证美国密歇根州立大学毕业证学位证书原版一模一样
171ticu
 
basic-wireline-operations-course-mahmoud-f-radwan.pdf
basic-wireline-operations-course-mahmoud-f-radwan.pdfbasic-wireline-operations-course-mahmoud-f-radwan.pdf
basic-wireline-operations-course-mahmoud-f-radwan.pdf
NidhalKahouli2
 
Properties Railway Sleepers and Test.pptx
Properties Railway Sleepers and Test.pptxProperties Railway Sleepers and Test.pptx
Properties Railway Sleepers and Test.pptx
MDSABBIROJJAMANPAYEL
 
Modelagem de um CSTR com reação endotermica.pdf
Modelagem de um CSTR com reação endotermica.pdfModelagem de um CSTR com reação endotermica.pdf
Modelagem de um CSTR com reação endotermica.pdf
camseq
 
Engine Lubrication performance System.pdf
Engine Lubrication performance System.pdfEngine Lubrication performance System.pdf
Engine Lubrication performance System.pdf
mamamaam477
 
Engineering Drawings Lecture Detail Drawings 2014.pdf
Engineering Drawings Lecture Detail Drawings 2014.pdfEngineering Drawings Lecture Detail Drawings 2014.pdf
Engineering Drawings Lecture Detail Drawings 2014.pdf
abbyasa1014
 
TIME DIVISION MULTIPLEXING TECHNIQUE FOR COMMUNICATION SYSTEM
TIME DIVISION MULTIPLEXING TECHNIQUE FOR COMMUNICATION SYSTEMTIME DIVISION MULTIPLEXING TECHNIQUE FOR COMMUNICATION SYSTEM
TIME DIVISION MULTIPLEXING TECHNIQUE FOR COMMUNICATION SYSTEM
HODECEDSIET
 
Embedded machine learning-based road conditions and driving behavior monitoring
Embedded machine learning-based road conditions and driving behavior monitoringEmbedded machine learning-based road conditions and driving behavior monitoring
Embedded machine learning-based road conditions and driving behavior monitoring
IJECEIAES
 
Understanding Inductive Bias in Machine Learning
Understanding Inductive Bias in Machine LearningUnderstanding Inductive Bias in Machine Learning
Understanding Inductive Bias in Machine Learning
SUTEJAS
 
学校原版美国波士顿大学毕业证学历学位证书原版一模一样
学校原版美国波士顿大学毕业证学历学位证书原版一模一样学校原版美国波士顿大学毕业证学历学位证书原版一模一样
学校原版美国波士顿大学毕业证学历学位证书原版一模一样
171ticu
 
A SYSTEMATIC RISK ASSESSMENT APPROACH FOR SECURING THE SMART IRRIGATION SYSTEMS
A SYSTEMATIC RISK ASSESSMENT APPROACH FOR SECURING THE SMART IRRIGATION SYSTEMSA SYSTEMATIC RISK ASSESSMENT APPROACH FOR SECURING THE SMART IRRIGATION SYSTEMS
A SYSTEMATIC RISK ASSESSMENT APPROACH FOR SECURING THE SMART IRRIGATION SYSTEMS
IJNSA Journal
 
2008 BUILDING CONSTRUCTION Illustrated - Ching Chapter 02 The Building.pdf
2008 BUILDING CONSTRUCTION Illustrated - Ching Chapter 02 The Building.pdf2008 BUILDING CONSTRUCTION Illustrated - Ching Chapter 02 The Building.pdf
2008 BUILDING CONSTRUCTION Illustrated - Ching Chapter 02 The Building.pdf
Yasser Mahgoub
 
22CYT12-Unit-V-E Waste and its Management.ppt
22CYT12-Unit-V-E Waste and its Management.ppt22CYT12-Unit-V-E Waste and its Management.ppt
22CYT12-Unit-V-E Waste and its Management.ppt
KrishnaveniKrishnara1
 
ML Based Model for NIDS MSc Updated Presentation.v2.pptx
ML Based Model for NIDS MSc Updated Presentation.v2.pptxML Based Model for NIDS MSc Updated Presentation.v2.pptx
ML Based Model for NIDS MSc Updated Presentation.v2.pptx
JamalHussainArman
 
Eric Nizeyimana's document 2006 from gicumbi to ttc nyamata handball play
Eric Nizeyimana's document 2006 from gicumbi to ttc nyamata handball playEric Nizeyimana's document 2006 from gicumbi to ttc nyamata handball play
Eric Nizeyimana's document 2006 from gicumbi to ttc nyamata handball play
enizeyimana36
 
Manufacturing Process of molasses based distillery ppt.pptx
Manufacturing Process of molasses based distillery ppt.pptxManufacturing Process of molasses based distillery ppt.pptx
Manufacturing Process of molasses based distillery ppt.pptx
Madan Karki
 

Recently uploaded (20)

Heat Resistant Concrete Presentation ppt
Heat Resistant Concrete Presentation pptHeat Resistant Concrete Presentation ppt
Heat Resistant Concrete Presentation ppt
 
Textile Chemical Processing and Dyeing.pdf
Textile Chemical Processing and Dyeing.pdfTextile Chemical Processing and Dyeing.pdf
Textile Chemical Processing and Dyeing.pdf
 
Recycled Concrete Aggregate in Construction Part II
Recycled Concrete Aggregate in Construction Part IIRecycled Concrete Aggregate in Construction Part II
Recycled Concrete Aggregate in Construction Part II
 
Comparative analysis between traditional aquaponics and reconstructed aquapon...
Comparative analysis between traditional aquaponics and reconstructed aquapon...Comparative analysis between traditional aquaponics and reconstructed aquapon...
Comparative analysis between traditional aquaponics and reconstructed aquapon...
 
官方认证美国密歇根州立大学毕业证学位证书原版一模一样
官方认证美国密歇根州立大学毕业证学位证书原版一模一样官方认证美国密歇根州立大学毕业证学位证书原版一模一样
官方认证美国密歇根州立大学毕业证学位证书原版一模一样
 
basic-wireline-operations-course-mahmoud-f-radwan.pdf
basic-wireline-operations-course-mahmoud-f-radwan.pdfbasic-wireline-operations-course-mahmoud-f-radwan.pdf
basic-wireline-operations-course-mahmoud-f-radwan.pdf
 
Properties Railway Sleepers and Test.pptx
Properties Railway Sleepers and Test.pptxProperties Railway Sleepers and Test.pptx
Properties Railway Sleepers and Test.pptx
 
Modelagem de um CSTR com reação endotermica.pdf
Modelagem de um CSTR com reação endotermica.pdfModelagem de um CSTR com reação endotermica.pdf
Modelagem de um CSTR com reação endotermica.pdf
 
Engine Lubrication performance System.pdf
Engine Lubrication performance System.pdfEngine Lubrication performance System.pdf
Engine Lubrication performance System.pdf
 
Engineering Drawings Lecture Detail Drawings 2014.pdf
Engineering Drawings Lecture Detail Drawings 2014.pdfEngineering Drawings Lecture Detail Drawings 2014.pdf
Engineering Drawings Lecture Detail Drawings 2014.pdf
 
TIME DIVISION MULTIPLEXING TECHNIQUE FOR COMMUNICATION SYSTEM
TIME DIVISION MULTIPLEXING TECHNIQUE FOR COMMUNICATION SYSTEMTIME DIVISION MULTIPLEXING TECHNIQUE FOR COMMUNICATION SYSTEM
TIME DIVISION MULTIPLEXING TECHNIQUE FOR COMMUNICATION SYSTEM
 
Embedded machine learning-based road conditions and driving behavior monitoring
Embedded machine learning-based road conditions and driving behavior monitoringEmbedded machine learning-based road conditions and driving behavior monitoring
Embedded machine learning-based road conditions and driving behavior monitoring
 
Understanding Inductive Bias in Machine Learning
Understanding Inductive Bias in Machine LearningUnderstanding Inductive Bias in Machine Learning
Understanding Inductive Bias in Machine Learning
 
学校原版美国波士顿大学毕业证学历学位证书原版一模一样
学校原版美国波士顿大学毕业证学历学位证书原版一模一样学校原版美国波士顿大学毕业证学历学位证书原版一模一样
学校原版美国波士顿大学毕业证学历学位证书原版一模一样
 
A SYSTEMATIC RISK ASSESSMENT APPROACH FOR SECURING THE SMART IRRIGATION SYSTEMS
A SYSTEMATIC RISK ASSESSMENT APPROACH FOR SECURING THE SMART IRRIGATION SYSTEMSA SYSTEMATIC RISK ASSESSMENT APPROACH FOR SECURING THE SMART IRRIGATION SYSTEMS
A SYSTEMATIC RISK ASSESSMENT APPROACH FOR SECURING THE SMART IRRIGATION SYSTEMS
 
2008 BUILDING CONSTRUCTION Illustrated - Ching Chapter 02 The Building.pdf
2008 BUILDING CONSTRUCTION Illustrated - Ching Chapter 02 The Building.pdf2008 BUILDING CONSTRUCTION Illustrated - Ching Chapter 02 The Building.pdf
2008 BUILDING CONSTRUCTION Illustrated - Ching Chapter 02 The Building.pdf
 
22CYT12-Unit-V-E Waste and its Management.ppt
22CYT12-Unit-V-E Waste and its Management.ppt22CYT12-Unit-V-E Waste and its Management.ppt
22CYT12-Unit-V-E Waste and its Management.ppt
 
ML Based Model for NIDS MSc Updated Presentation.v2.pptx
ML Based Model for NIDS MSc Updated Presentation.v2.pptxML Based Model for NIDS MSc Updated Presentation.v2.pptx
ML Based Model for NIDS MSc Updated Presentation.v2.pptx
 
Eric Nizeyimana's document 2006 from gicumbi to ttc nyamata handball play
Eric Nizeyimana's document 2006 from gicumbi to ttc nyamata handball playEric Nizeyimana's document 2006 from gicumbi to ttc nyamata handball play
Eric Nizeyimana's document 2006 from gicumbi to ttc nyamata handball play
 
Manufacturing Process of molasses based distillery ppt.pptx
Manufacturing Process of molasses based distillery ppt.pptxManufacturing Process of molasses based distillery ppt.pptx
Manufacturing Process of molasses based distillery ppt.pptx
 

POWER SUPPLY SYSTEMS& DISTRIBUTION SYSTEMS

  • 1. By D.Rajesh Asst. Prof. EEE Department 6 October 2020 D.Rajesh Asst.Prof. EEE Department 1
  • 2.  Unit-i Electric Power Generation& Economic Considerations  Unit-ii Power Supply Systems& Distribution Systems  Unit-iii Inductance & Capacitance Calculations  Unit-iv Performance Of Transmission Lines  Unit-v Overhead Line Insulators 6 October 2020 D.Rajesh Asst.Prof. EEE Department 2
  • 3. POWER SUPPLY SYSTEMS& DISTRIBUTION SYSTEMS:  Transmission and distribution systems- D.C 2-wire and 3- wire systems, A.C single phase , three phase and 4-wire systems, comparison of copper efficiency.  Primary and secondary distribution systems, concentrated & uniformly distributed loads on distributors fed at both ends, ring distributor, voltage drop and power loss calculation,  Kelvin’slaw. 6 October 2020 D.Rajesh Asst.Prof. EEE Department 3
  • 4. Power Transmission D.C D.C. two-wire D.C. two-wire with mid-point earthed D.C. three-wire A.C Single-phase A.C. system Single-phase two- wire Single-phase two- wire with mid- point earthed Single-phase three-wire Two-phase A.C. system Two-phase four- wire Two-phase three wire. Three-phase A.C. system Three-phase three-wire Three-phase four- wire 6 October 2020 D.Rajesh Asst.Prof. EEE Department 4
  • 5. 6 October 2020 D.Rajesh Asst.Prof. EEE Department 5 Overhead Under ground
  • 6. (i) Same power (P watts) transmitted by each system. (ii) The distance (l metres) over which power is transmitted remains the same. (iii) The line losses (W watts) are the same in each case. (iv) The maximum voltage between any conductor and earth (𝑉 𝑚) is the same in each case. 6 October 2020 D.Rajesh Asst.Prof. EEE Department 6
  • 7. Max. voltage between conductors = 𝑉 𝑚 , Power to be transmitted = P= 𝑉 𝑚𝐼1 1 ∴ Load current, 𝐼1 = P/ 𝑉 𝑚 If 𝑅1 is the resistance of each line conductor, then, 𝑅1 = ρ l/ 𝑎1 where 𝑎1 is the area of X-section of the conductor. copper losses in line W = 𝐼1 2 𝑅1 + 𝐼1 2 𝑅1 =2𝐼1 2 𝑅1 6 October 2020 D.Rajesh Asst.Prof. EEE Department 7 1 1 2
  • 8. The maximum voltage between any conductor and earth is 𝑉 𝑚 so that maximum voltage between conductors is 2 𝑉 𝑚. 𝑅2 is the resistance of each line conductor Power to be transmitted P= 𝑉 𝑚𝐼2+ 𝑉 𝑚𝐼2=2 𝑉 𝑚𝐼2 Load current, 𝐼2 = P/2 𝑉 𝑚 Since from assumption 1 the power transmitted is same for all systems so from eq 1& 3 2𝑉 𝑚𝐼2= 𝑉 𝑚𝐼1 𝐼2 = 𝐼1 2 copper losses in line W = 𝐼2 2 𝑅2+ 𝐼2 2 𝑅2 = 2 𝐼2 2 𝑅2 From eq 4 W=2( 𝐼1 2 )2 𝑅2 = 𝐼1 2 2 𝑅2 6 October 2020 D.Rajesh Asst.Prof. EEE Department 8 3 4 5
  • 9.  Compare eq 2 & 5 2𝐼1 2 𝑅1 = ( 𝐼1 2 2 ) 𝑅2 𝑅1 𝑅2 = 1 4 𝑎1 𝑎2 =4 (where R = ρ l/ 𝑎) 𝑎1=X-area of conductor in 2 wire dc system 𝑎2= X-area of conductor in 2 wire dc system with mid-point earthed Then volume of copper wire Two−wire D.C system with mid−point earthed volume of copper wire Two−wire D.C system = 𝑎2∗𝑙2 𝑎1∗𝑙1 = 1 4 𝑎1∗2𝑙 𝑎1∗2𝑙 = 1 4 where 𝑙1 = 𝑙2=l+l Hence, the volume of conductor material required in Two−wire D.C system with mid−point earthed is one-fourth of that required in a two-wire D.C system with one conductor earthed. 6 October 2020 D.Rajesh Asst.Prof. EEE Department 9
  • 10. If the load is balanced, the current in the neutral wire is zero. Assuming balanced loads, Power to be transmitted P= 𝑉 𝑚𝐼3+ 𝑉 𝑚𝐼3=2 𝑉 𝑚𝐼3 6 Since from assumption 1 the power transmitted is same for all systems so from eq 1& 6 2𝑉 𝑚𝐼3= 𝑉 𝑚𝐼1 𝐼3 = 𝐼1 2 7 Let 𝑅3 is the resistance of each line conductor, the area of X-section of neutral wire to be half that of the outer wire, copper losses in line W = 𝐼3 2 𝑅3+ 𝐼3 2 𝑅3 = 2𝐼3 2 𝑅3 8 From eq 7 W=2( 𝐼1 2 )2 𝑅3 = 𝐼1 2 2 𝑅3 9 6 October 2020 D.Rajesh Asst.Prof. EEE Department 10
  • 11.  Compare eq 2 & 8 2𝐼1 2 𝑅1 = 𝐼1 2 2 𝑅3 𝑅1 𝑅3 = 1 4 𝑅3 = ρ l/ 𝑎3 where 𝑎3 is the area of X-section of the conductor. 𝑎1 𝑎3 =4 Then volume of copper wire Three−wire D.C volume of copper wire Two−wire D.C system = 𝑎3∗𝑙3 𝑎1∗𝑙1 = 1 4 𝑎1∗2.5𝑙 𝑎1∗2𝑙 = 2.5 8 where 𝑙3=l+l+0.5l =0.3125 Hence, the volume of conductor material required in Three−wire D.C system is 0.3125 times of that required in a two-wire D.C system with one conductor earthed. 6 October 2020 D.Rajesh Asst.Prof. EEE Department 11
  • 12. The maximum voltage between conductors is 𝑉 𝑚, so that r.m.s. value of voltage between them is 𝑉𝑚 2 . Assuming the load power factor to be cos φ, load current 𝐼4 Power supplied P= 𝑉𝑚 2 𝐼4 cos φ 10 Since from assumption 1 the power transmitted is same for all systems so from eq 1& 10 𝑉𝑚 2 𝐼4 cos φ= 𝑉 𝑚𝐼1 𝐼4= 2𝐼1 cos φ 11 Let 𝑅4 is the resistance of each line conductor copper losses in line W = 𝐼4 2 𝑅4+ 𝐼4 2 𝑅4 = 2𝐼4 2 𝑅4 =2( 2𝐼1 cos φ )2 𝑅4 =4 𝐼1 2 𝑐𝑜𝑠2φ 𝑅4 12 6 October 2020 D.Rajesh Asst.Prof. EEE Department 12
  • 13.  Comparing copper losses with 2 wire d.c system 4 𝐼1 2 𝑐𝑜𝑠2φ 𝑅4= 2𝐼1 2 𝑅1 𝑅4 𝑅1 = 𝑐𝑜𝑠2φ 2 𝑎4 𝑎1 = 2 𝑐𝑜𝑠2φ 13 volume of copper wire 1−phase 2− wire volume of copper wire Two−wire D.C system = 𝑎4𝑙4 𝑎1𝑙1 = 2 𝑐𝑜𝑠2φ𝑎12𝑙 𝑎12𝑙 = 2 𝑐𝑜𝑠2φ Hence, the volume of conductor material required in this system is 2 𝑐𝑜𝑠2φ times that of 2-wire d.c. system with the one conductor earthed. 6 October 2020 D.Rajesh Asst.Prof. EEE Department 13
  • 14.  The two wires possess equal and opposite voltages to earth (i.e., 𝑉 𝑚).  Therefore, the maximum voltage between the two wires is 𝑉𝑚 2 . The r.m.s. value of voltage between conductors is =2 𝑉𝑚 2 = 2 𝑉 𝑚. Assuming the power factor of the load to be cos φ, load current 𝐼5  Power supplied P= 2 𝑉 𝑚𝐼5 cos φ 14 Since from assumption 1 the power transmitted is same for all systems so from eq 1& 14 2 𝑉 𝑚𝐼5 cos φ= 𝑉 𝑚𝐼1 𝐼5= 𝐼1 2 cos φ 15 6 October 2020 D.Rajesh Asst.Prof. EEE Department 14
  • 15. Let 𝑅5 is the resistance of each line conductor copper losses in line W = 𝐼5 2 𝑅5+ 𝐼5 2 𝑅5 = 2𝐼5 2 𝑅5 =2( 𝐼1 2 cos φ )2 𝑅5 = 𝐼1 2 𝑐𝑜𝑠2φ 𝑅5 Comparing copper losses with 2 wire d.c system 𝐼1 2 𝑐𝑜𝑠2φ 𝑅5 =2𝐼1 2 𝑅1 𝑅5 𝑅1 = 2𝑐𝑜𝑠2φ 𝑎5 𝑎1 = 1 2𝑐𝑜𝑠2φ volume of copper wire 1−phase 2− wire 𝑤𝑖𝑡ℎ 𝑚𝑖𝑑 − 𝑝𝑜𝑖𝑛𝑡 𝑒𝑎𝑟𝑡ℎ𝑒𝑑 volume of copper wire Two−wire D.C system 𝑎5𝑙5 𝑎1𝑙1 = 1 2𝑐𝑜𝑠2φ 𝑎12𝑙 𝑎12𝑙 = 1 2𝑐𝑜𝑠2φ Hence the volume of conductor material required in this system is 1 2𝑐𝑜𝑠2φ times that of 2-wire d.c. system with one conductor earthed. 6 October 2020 D.Rajesh Asst.Prof. EEE Department 15
  • 16.  If the load is balanced, the current through the neutral wire is zero. Assuming balanced load,  The two wires possess equal and opposite voltages to earth (i.e., 𝑉 𝑚).  Therefore, the maximum voltage between the two wires is 𝑉𝑚 2 . The r.m.s. value of voltage between conductors is 2 𝑉𝑚 2 = 2 𝑉 𝑚. Assuming the power factor of the load to be cos φ, load current 𝐼6  Power supplied P= 2 𝑉 𝑚𝐼6 cos φ 16 Since from assumption 1 the power transmitted is same for all systems so from eq 1& 16 2 𝑉 𝑚𝐼6 cos φ= 𝑉 𝑚𝐼1 𝐼6= 𝐼1 2 cos φ 17 6 October 2020 D.Rajesh Asst.Prof. EEE Department 16
  • 17. Let 𝑅6 is the resistance of each line conductor copper losses in line W = 𝐼6 2 𝑅6+ 𝐼6 2 𝑅6 = 2𝐼6 2 𝑅6 =2( 𝐼1 2 cos φ )2 𝑅6 = 𝐼1 2 𝑐𝑜𝑠2φ 𝑅6 Comparing copper losses with 2 wire d.c system 𝐼1 2 𝑐𝑜𝑠2φ 𝑅6 =2𝐼1 2 𝑅1 𝑅6 𝑅1 = 2𝑐𝑜𝑠2φ 𝑎6 𝑎1 = 1 2𝑐𝑜𝑠2φ volume of copper wire 1−phase3−wire system volume of copper wire Two−wire D.C system 𝑎6𝑙6 𝑎1𝑙1 = 1 2𝑐𝑜𝑠2φ 𝑎12.5𝑙 𝑎12𝑙 = 0.625 𝑐𝑜𝑠2φ Hence the volume of conductr material required in this system is 0.625 𝑐𝑜𝑠2φ times that of 2-wire d.c. system with one conductor earthed. 6 October 2020 D.Rajesh Asst.Prof. EEE Department 17
  • 18.  This system can be considered as two independent single- phase systems, each transmitting one half of the total power.  As the voltage is same (𝑉 𝑚) as in a Single phase 2-wire system with mid-point earthed, so the current will be half of the Single phase system  i.e. 𝐼7= 𝐼5 2 = 𝐼1 2 cos φ 2 = 𝐼1 2 2 cos φ  If the current density remains constant the X-area will be half of the single-phase system  i.e. 𝑎7= 𝑎5 2 = 1 2𝑐𝑜𝑠2φ 𝑎1 2 = 𝑎1 4𝑐𝑜𝑠2φ 6 October 2020 D.Rajesh Asst.Prof. EEE Department 18
  • 19.  Since wires are four  Then  volume of copper wireTwo phase, 4−wire a.c. system volume of copper wire Two−wire D.C system 𝑎7𝑙7 𝑎1𝑙1 = 1 4𝑐𝑜𝑠2φ 𝑎14𝑙 𝑎12𝑙 = 1 2𝑐𝑜𝑠2φ Hence the volume of conductor material required in this system is 1 2𝑐𝑜𝑠2φ times that of 2-wire d.c. system with one conductor earthed. 6 October 2020 D.Rajesh Asst.Prof. EEE Department 19
  • 20. The third or neutral wire is taken from the junction of two-phase windings whose voltages are in quadrature with each other. Obviously, each phase transmits one half of the total power. The R.M.S. voltage between outgoing conductor and neutral = 𝑉𝑚 2 , Let 𝑅8 is the resistance of each line conductor Total power transmitted=2 𝑉𝑚 2 𝐼8 cos φ Comparing the power with 2-wire D.C 2 𝑉𝑚 2 𝐼8 cos φ= 𝑉 𝑚𝐼1 𝐼8 = 𝐼1 2cos φ The middle wire will act as a common return and the current through it will be 𝐼8 2 + 𝐼8 2 = 2 𝐼8 6 October 2020 D.Rajesh Asst.Prof. EEE Department 20
  • 21.  Assuming the current density to be constant, the area of X-section of the neutral wire will be 2 times that of either of the outers.  ∴ Resistance of neutral wire= 𝑅8 2  Copper losses in this system W= 𝐼8 2 𝑅8+ 𝐼8 2 𝑅8 + ( 2 𝐼8)2𝑅8 2 = 2𝐼8 2 𝑅8+ 2𝐼8 2 𝑅8=(2+ 2) 𝐼8 2 𝑅8 Where 𝐼8 = 𝐼1 2cos φ W=(2+ 2) ( 𝐼1 2cos φ)2 𝑅8=(2+ 2) 𝐼1 2𝑅8 2𝑐𝑜𝑠2φ Comparing copper losses with 2 wire d.c system (2+ 2) 𝐼1 2𝑅8 2𝑐𝑜𝑠2φ = 2𝐼1 2 𝑅1 𝑅8 𝑅1 = 4𝑐𝑜𝑠2φ (2+ 2) 𝑎8 𝑎1 = (2+ 2) 4𝑐𝑜𝑠2φ volume of copper wireTwo phase, 3−wire a.c. system volume of copper wire Two−wire D.C system 𝑎8𝑙8 𝑎1𝑙1 = (2+ 2) 4𝑐𝑜𝑠2φ 𝑎1(2+ 2)𝑙 𝑎12𝑙 = 1.457 𝑐𝑜𝑠2φ Hence, the volume of conductor material required for this system is 1.457 𝑐𝑜𝑠2φ times that of 2-wire d.c. system with one conductor earthed. 6 October 2020 D.Rajesh Asst.Prof. EEE Department 21
  • 22. This system is almost universally adopted for transmission of electric power. The 3-phase, 3- wire system may be star connected or delta connected. Maximum voltage between each conductor and neutral =𝑉 𝑚 Let 𝑅9 is the resistance of each line conductor R.M.S. voltage per phase = 𝑉𝑚 2 Power supplied=3( 𝑉𝑚 2 I9cos φ )  Comparing the power with 2-wire D.C 3 𝑉𝑚 2 𝐼9 cos φ= 𝑉 𝑚𝐼1 𝐼9 = 2𝐼1 3cos φ 6 October 2020 D.Rajesh Asst.Prof. EEE Department 22
  • 23.  Total copper losses= 3𝐼9 2 𝑅9 =3 ( 2𝐼1 3cos φ )2 𝑅9= 2𝐼1 2 𝑅9 3𝑐𝑜𝑠2φ  Comparing copper losses with 2 wire d.c system 2𝐼1 2 𝑅9 3𝑐𝑜𝑠2φ = 2𝐼1 2 𝑅1 𝑅9 𝑅1 = 3𝑐𝑜𝑠2φ 𝑎9 𝑎1 = 1 3𝑐𝑜𝑠2φ volume of copper wire3−Phase, 3−wire system volume of copper wire Two−wire D.C system = 𝑎9𝑙9 𝑎1𝑙1 = 1 3𝑐𝑜𝑠2φ𝑎13𝑙 𝑎12𝑙 = 0.5 𝑐𝑜𝑠2φ Hence, the volume of conductor material required for this system is 0.5 𝑐𝑜𝑠2φ times that required for 2-wire d.c. system with one conductor earthed. 6 October 2020 D.Rajesh Asst.Prof. EEE Department 23
  • 24. In this case, 4th or neutral wire is taken from the neutral point. The area of X- section of the neutral wire is generally one-half that of the line conductor. If the loads are balanced, then current through the neutral wire is zero. Assuming balanced loads and p.f. of the load as cos φ, Let 𝑅10 is the resistance of each line conductor R.M.S. voltage per phase = 𝑉𝑚 2 Power supplied=3( 𝑉𝑚 2 I10cos φ )  Comparing the power with 2-wire D.C 3 𝑉𝑚 2 𝐼10 cos φ= 𝑉 𝑚𝐼1 𝐼10 = 2𝐼1 3cos φ 6 October 2020 D.Rajesh Asst.Prof. EEE Department 24
  • 25.  Total copper losses= 3𝐼10 2 𝑅10 =3 ( 2𝐼1 3cos φ )2 𝑅10= 2𝐼1 2 𝑅10 3𝑐𝑜𝑠2φ  Comparing copper losses with 2 wire d.c system 2𝐼1 2 𝑅10 3𝑐𝑜𝑠2φ = 2𝐼1 2 𝑅1 𝑅10 𝑅1 = 3𝑐𝑜𝑠2φ 𝑎10 𝑎1 = 1 3𝑐𝑜𝑠2φ volume of copper wire3−Phase, 3−wire system volume of copper wire Two−wire D.C system = 𝑎10𝑙10 𝑎1𝑙1 = 1 3𝑐𝑜𝑠2φ𝑎13.5𝑙 𝑎12𝑙 = 0.583 𝑐𝑜𝑠2φ Hence, the volume of conductor material required for this system is 0.583 𝑐𝑜𝑠2φ times that required for 2-wire d.c. system with one conductor earthed. 6 October 2020 D.Rajesh Asst.Prof. EEE Department 25
  • 26. In the under-ground system the maximum stress exists on the insulation between the conductors. (i) Same power (P watts) transmitted by each system. (ii) The distance (l meters) over which power is transmitted remains the same. (iii) The line losses (W watts) are the same in each case. (iv) The maximum voltage between conductors (𝑉 𝑚) is the same in each case. 6 October 2020 D.Rajesh Asst.Prof. EEE Department 26
  • 27. Max. voltage between conductors = 𝑉 𝑚 , Power to be transmitted = P= 𝑉 𝑚𝐼1 1 ∴ Load current, 𝐼1 = P/ 𝑉 𝑚 If 𝑅1 is the resistance of each line conductor, then, 𝑅1 = ρ l/ 𝑎1 where 𝑎1 is the area of X-section of the conductor. copper losses in line W = 𝐼1 2 𝑅1 + 𝐼1 2 𝑅1 =2𝐼1 2 𝑅1 6 October 2020 D.Rajesh Asst.Prof. EEE Department 27 1 1 2
  • 28. Maximum voltage between conductors is 𝑉 𝑚. 𝑅2 is the resistance of each line conductor Power to be transmitted P= 𝑉 𝑚𝐼2 Load current, 𝐼2 = P/ 𝑉 𝑚 Since from assumption 1 the power transmitted is same for all systems so from eq 1& 3 𝑉 𝑚𝐼2= 𝑉 𝑚𝐼1 𝐼2 = 𝐼1 copper losses in line W = 𝐼2 2 𝑅2+ 𝐼2 2 𝑅2 = 2 𝐼2 2 𝑅2 From eq 4 W=2( 𝐼1)2 𝑅2 =2𝐼1 2 𝑅2 6 October 2020 D.Rajesh Asst.Prof. EEE Department 28 3 4 5
  • 29.  Compare eq 2 & 5 2𝐼1 2 𝑅1 = (𝐼1 2 ) 𝑅2 𝑅1 𝑅2 =1 𝑎1 𝑎2 =1 (where R = ρ l/ 𝑎) 𝑎1=X-area of conductor in 2 wire dc system 𝑎2= X-area of conductor in 2 wire dc system with mid-point earthed Then volume of copper wire Two−wire D.C system with mid−point earthed volume of copper wire Two−wire D.C system = 𝑎2∗𝑙2 𝑎1∗𝑙1 = 𝑎1∗2𝑙 𝑎1∗2𝑙 =1 where 𝑙1 = 𝑙2=l+l Hence, the volume of conductor material required in this system is the same as that for 2-wire d.c. system. 6 October 2020 D.Rajesh Asst.Prof. EEE Department 29
  • 30. If the load is balanced, the current in the neutral wire is zero. Assuming balanced loads, Power to be transmitted P= 𝑉 𝑚𝐼3 6 Since from assumption 1 the power transmitted is same for all systems so from eq 1& 6 𝑉 𝑚𝐼3= 𝑉 𝑚𝐼1 𝐼3 = 𝐼1 7 Let 𝑅3 is the resistance of each line conductor, the area of X-section of neutral wire to be half that of the outer wire, copper losses in line W = 𝐼3 2 𝑅3+ 𝐼3 2 𝑅3 = 2𝐼3 2 𝑅3 8 From eq 7 W=2( 𝐼1)2 𝑅3 =2𝐼1 2 𝑅3 9 6 October 2020 D.Rajesh Asst.Prof. EEE Department 30
  • 31.  Compare eq 2 & 8 2𝐼1 2 𝑅1 = 2(𝐼1 2 ) 𝑅3 𝑅1 𝑅3 =1 𝑎1 𝑎3 =1 𝑅3 = ρ l/ 𝑎3 where 𝑎3 is the area of X-section of the conductor. Then volume of copper wire Three−wire D.C volume of copper wire Two−wire D.C system = 𝑎3∗𝑙3 𝑎1∗𝑙1 = 𝑎1∗2.5𝑙 𝑎1∗2𝑙 = 2.5 2 where 𝑙3=l+l+0.5l =1.25 Hence, the volume of conductor material required in the system is 1·25 times that required for 2-wire d.c. system. 6 October 2020 D.Rajesh Asst.Prof. EEE Department 31
  • 32. The maximum voltage between conductors is 𝑉 𝑚, so that r.m.s. value of voltage between them is 𝑉𝑚 2 . Assuming the load power factor to be cos φ, load current 𝐼4 Power supplied P= 𝑉𝑚 2 𝐼4 cos φ 10 Since from assumption 1 the power transmitted is same for all systems so from eq 1& 10 𝑉𝑚 2 𝐼4 cos φ= 𝑉 𝑚𝐼1 𝐼4= 2𝐼1 cos φ 11 Let 𝑅4 is the resistance of each line conductor copper losses in line W = 𝐼4 2 𝑅4+ 𝐼4 2 𝑅4 = 2𝐼4 2 𝑅4 =2( 2𝐼1 cos φ )2 𝑅4 =4 𝐼1 2 𝑐𝑜𝑠2φ 𝑅4 12 6 October 2020 D.Rajesh Asst.Prof. EEE Department 32
  • 33.  Comparing copper losses with 2 wire d.c system 4 𝐼1 2 𝑐𝑜𝑠2φ 𝑅4= 2𝐼1 2 𝑅1 𝑅4 𝑅1 = 𝑐𝑜𝑠2φ 2 𝑎4 𝑎1 = 2 𝑐𝑜𝑠2φ 13 volume of copper wire 1−phase 2− wire volume of copper wire Two−wire D.C system = 𝑎4𝑙4 𝑎1𝑙1 = 2 𝑐𝑜𝑠2φ𝑎12𝑙 𝑎12𝑙 = 2 𝑐𝑜𝑠2φ Hence, the volume of conductor material required in this system is 2 𝑐𝑜𝑠2φ times that of 2-wire d.c. system. 6 October 2020 D.Rajesh Asst.Prof. EEE Department 33
  • 34. The maximum value of voltage between the outers is 𝑉 𝑚. Therefore, the r.m.s. value of voltage is 𝑉𝑚 2 Assuming the power factor of the load to be cos φ, load current 𝐼5 Power supplied P= 𝑉𝑚 2 𝐼5 cos φ 14 Since from assumption 1 the power transmitted is same for all systems so from eq 1& 14 𝑉𝑚 2 𝐼5 cos φ= 𝑉 𝑚𝐼1 𝐼5= 2𝐼1 cos φ 15 6 October 2020 D.Rajesh Asst.Prof. EEE Department 34
  • 35. Let 𝑅5 is the resistance of each line conductor copper losses in line W = 𝐼5 2 𝑅5+ 𝐼5 2 𝑅5 = 2𝐼5 2 𝑅5 =2( 2𝐼1 cos φ )2 𝑅5 = 4𝐼1 2 𝑐𝑜𝑠2φ 𝑅5 Comparing copper losses with 2 wire d.c system 4𝐼1 2 𝑐𝑜𝑠2φ 𝑅5 =2𝐼1 2 𝑅1 𝑅5 𝑅1 = 𝑐𝑜𝑠2φ 2 𝑎5 𝑎1 = 2 𝑐𝑜𝑠2φ volume of copper wire 1−phase 2− wire 𝑤𝑖𝑡ℎ 𝑚𝑖𝑑 − 𝑝𝑜𝑖𝑛𝑡 𝑒𝑎𝑟𝑡ℎ𝑒𝑑 volume of copper wire Two−wire D.C system 𝑎5𝑙5 𝑎1𝑙1 = 2 𝑐𝑜𝑠2φ 𝑎12𝑙 𝑎12𝑙 = 2 𝑐𝑜𝑠2φ Hence, the volume of conductor material required in this system is 2 𝑐𝑜𝑠2φ times that required in 2-wire d.c. system. 6 October 2020 D.Rajesh Asst.Prof. EEE Department 35
  • 36.  If the load is balanced, the current through the neutral wire is zero. Assuming balanced load,  The maximum value of voltage between the outers is 𝑉 𝑚. Therefore, the r.m.s. value of voltage is 𝑉𝑚 2 .  Assuming the power factor of the load to be cos φ, load current 𝐼6  Power supplied P= 𝑉𝑚 2 𝐼6 cos φ 16 Since from assumption 1 the power transmitted is same for all systems so from eq 1& 16 𝑉𝑚 2 𝐼6 cos φ= 𝑉 𝑚𝐼1 𝐼6= 2𝐼1 cos φ 17 6 October 2020 D.Rajesh Asst.Prof. EEE Department 36
  • 37. Let 𝑅6 is the resistance of each line conductor copper losses in line W = 𝐼6 2 𝑅6+ 𝐼6 2 𝑅6 = 2𝐼6 2 𝑅6 =2( 2𝐼1 cos φ )2 𝑅6 = 4𝐼1 2 𝑐𝑜𝑠2φ 𝑅6 Comparing copper losses with 2 wire d.c system 4𝐼1 2 𝑐𝑜𝑠2φ 𝑅6 =2𝐼1 2 𝑅1 𝑅6 𝑅1 = 𝑐𝑜𝑠2φ 2 𝑎6 𝑎1 = 2 𝑐𝑜𝑠2φ volume of copper wire 1−phase 2− wire 𝑤𝑖𝑡ℎ 𝑚𝑖𝑑 − 𝑝𝑜𝑖𝑛𝑡 𝑒𝑎𝑟𝑡ℎ𝑒𝑑 volume of copper wire Two−wire D.C system 𝑎6𝑙6 𝑎1𝑙1 = = 2 𝑐𝑜𝑠2φ 𝑎12.5𝑙 𝑎12𝑙 = 2.5 𝑐𝑜𝑠2φ  Hence, the volume of conductor material required in this system is 2.5 𝑐𝑜𝑠2φ times that required in 2-wire d.c. system. 6 October 2020 D.Rajesh Asst.Prof. EEE Department 37
  • 38.  This system can be considered as two independent single-phase systems, each transmitting one half of the total power.  Current (𝐼7) in each conductor will be half that in single-phase 2- wire system. Consequently, area of X-section of each conductor is also half but as there are four wires, so volume of conductor material used is the same as in a single phase, 2-wire system  i.e. 𝐼7= 𝐼5 2 = 2𝐼1 cos φ 2 = 𝐼1 2 cos φ  If the current density remains constant the X-area will be half of the single-phase system  i.e. 𝑎7= 𝑎5 2 = 2 𝑐𝑜𝑠2φ 2 = 1 𝑐𝑜𝑠2φ 6 October 2020 D.Rajesh Asst.Prof. EEE Department 38
  • 39.  Since wires are four  Then  volume of copper wireTwo phase, 4−wire a.c. system volume of copper wire Two−wire D.C system 𝑎7𝑙7 𝑎1𝑙1 = 1 𝑐𝑜𝑠2φ 𝑎14𝑙 𝑎12𝑙 = 2 𝑐𝑜𝑠2φ Hence, volume of conductor material required in this system is 2 𝑐𝑜𝑠2φ times that required in 2- wire d.c. system. 6 October 2020 D.Rajesh Asst.Prof. EEE Department 39
  • 40.  Let us suppose that maximum voltage between the outers is 𝑉 𝑚. Then maximum voltage between either outer and neutral wire is 𝑉𝑚 2  R.M.S. voltage between outer and neutral wire= 𝑉𝑚 2 2 = 𝑉𝑚 2  Total power transmitted=2 𝑉𝑚 2 𝐼8 cos φ = 𝑉 𝑚 𝐼8 cos φ  Comparing the power with 2-wire D.C 𝑉 𝑚 𝐼8 cos φ= 𝑉 𝑚𝐼1 𝐼8 = 𝐼1 cos φ The middle wire will act as a common return and the current through it will be 𝐼8 2 + 𝐼8 2 = 2 𝐼8 6 October 2020 D.Rajesh Asst.Prof. EEE Department 40
  • 41.  Assuming the current density to be constant, the area of X-section of the neutral wire will be 2 times that of either of the outers.  ∴ Resistance of neutral wire= 𝑅8 2  Copper losses in this system W= 𝐼8 2 𝑅8+ 𝐼8 2 𝑅8 + ( 2 𝐼8)2𝑅8 2 = 2𝐼8 2 𝑅8+ 2𝐼8 2 𝑅8=(2+ 2) 𝐼8 2 𝑅8 Where 𝐼8 = 𝐼1 cos φ W=(2+ 2) ( 𝐼1 cos φ)2 𝑅8=(2+ 2) 𝐼1 2𝑅8 𝑐𝑜𝑠2φ Comparing copper losses with 2 wire d.c system (2+ 2) 𝐼1 2𝑅8 𝑐𝑜𝑠2φ = 2𝐼1 2 𝑅1 𝑅8 𝑅1 = 2𝑐𝑜𝑠2φ (2+ 2) 𝑎8 𝑎1 = (2+ 2) 2𝑐𝑜𝑠2φ volume of copper wireTwo phase, 3−wire a.c. system volume of copper wire Two−wire D.C system 𝑎8𝑙8 𝑎1𝑙1 = (2+ 2) 2𝑐𝑜𝑠2φ 𝑎1(2+ 2)𝑙 𝑎12𝑙 = 2.914 𝑐𝑜𝑠2φ Hence, the volume of conductor material required for this system is 2.914 𝑐𝑜𝑠2φ times that of 2-wire d.c. system. 6 October 2020 D.Rajesh Asst.Prof. EEE Department 41
  • 42. This system is almost universally adopted for transmission of electric power. The 3-phase, 3- wire system may be star connected or delta connected. Maximum voltage between conductors =𝑉 𝑚, Let 𝑅9 is the resistance of each line conductor Then maximum voltage between each phase and neutral is 𝑉𝑚 3 R.M.S. voltage per phase = 𝑉𝑚 3 2 = 𝑉𝑚 6 Power supplied=3( 𝑉𝑚 6 I9cos φ )  Comparing the power with 2-wire D.C 3 𝑉𝑚 6 𝐼9 cos φ= 𝑉 𝑚𝐼1 𝐼9 = 6𝐼1 3cos φ 6 October 2020 D.Rajesh Asst.Prof. EEE Department 42
  • 43.  Total copper losses= 3𝐼9 2 𝑅9 =3 ( 6𝐼1 3cos φ )2 𝑅9= 2𝐼1 2 𝑅9 𝑐𝑜𝑠2φ  Comparing copper losses with 2 wire d.c system 2𝐼1 2 𝑅9 𝑐𝑜𝑠2φ = 2𝐼1 2 𝑅1 𝑅9 𝑅1 = 𝑐𝑜𝑠2φ 𝑎9 𝑎1 = 1 𝑐𝑜𝑠2φ volume of copper wire3−Phase, 3−wire system volume of copper wire Two−wire D.C system = 𝑎9𝑙9 𝑎1𝑙1 = 1 𝑐𝑜𝑠2φ𝑎13𝑙 𝑎12𝑙 = 1.5 𝑐𝑜𝑠2φ Hence, the volume of conductor material required for this system is 1.5 𝑐𝑜𝑠2φ times that required for 2-wire d.c. system. 6 October 2020 D.Rajesh Asst.Prof. EEE Department 43
  • 44. In this case, 4th or neutral wire is taken from the neutral point. The area of X- section of the neutral wire is generally one-half that of the line conductor. If the loads are balanced, then current through the neutral wire is zero. Assuming balanced loads and p.f. of the load as cos φ, Let 𝑅10 is the resistance of each line conductor R.M.S. voltage per phase = 𝑉𝑚 6 Power supplied=3( 𝑉𝑚 6 I10cos φ )  Comparing the power with 2-wire D.C 3 𝑉𝑚 6 𝐼10 cos φ= 𝑉 𝑚𝐼1 𝐼10 = 6𝐼1 3cos φ 6 October 2020 D.Rajesh Asst.Prof. EEE Department 44
  • 45.  Total copper losses= 3𝐼10 2 𝑅10 =3 ( 6𝐼1 3cos φ )2 𝑅10= 2𝐼1 2 𝑅10 𝑐𝑜𝑠2φ  Comparing copper losses with 2 wire d.c system 2𝐼1 2 𝑅10 𝑐𝑜𝑠2φ = 2𝐼1 2 𝑅1 𝑅10 𝑅1 = 𝑐𝑜𝑠2φ 𝑎10 𝑎1 = 1 𝑐𝑜𝑠2φ volume of copper wire3−Phase, 3−wire system volume of copper wire Two−wire D.C system = 𝑎10𝑙10 𝑎1𝑙1 = 1 𝑐𝑜𝑠2φ𝑎13.5𝑙 𝑎12𝑙 = 1.75 𝑐𝑜𝑠2φ Hence, the volume of conductor material required for this system is 1.75 𝑐𝑜𝑠2φtimes that required for 2-wire d.c. system. 6 October 2020 D.Rajesh Asst.Prof. EEE Department 45
  • 46. System Same maximum voltage to earth Same maximum voltage between conductors 1. D.C. system (i) Two-wire (ii) Two-wire mid-point earthed (iii) 3-wire 1 0.25 0.3125 1 1 1.25 2. Single phase system (i) Two-wire (ii) Two-wire mid-point earthed (iii) 3-wire 2 𝑐𝑜𝑠2φ 0.5 𝑐𝑜𝑠2φ 1.625 𝑐𝑜𝑠2φ 2 𝑐𝑜𝑠2φ 0.5 𝑐𝑜𝑠2φ 2.5 𝑐𝑜𝑠2φ 6 October 2020 D.Rajesh Asst.Prof. EEE Department 46
  • 47. System Same maximum voltage to earth Same maximum voltage between conductors 3. Two-phase system (i) 2–phase 4-wire (ii)2-phase 3-wire 0.5 𝑐𝑜𝑠2φ 1.457 𝑐𝑜𝑠2φ 2 𝑐𝑜𝑠2φ 2.914 𝑐𝑜𝑠2φ 4. Three-phase system (i) 3-phase 3-wire (ii)3-phase 4-wire , 0.5 𝑐𝑜𝑠2φ 0.583 𝑐𝑜𝑠2φ 1.5 𝑐𝑜𝑠2φ 1.75 𝑐𝑜𝑠2φ 6 October 2020 D.Rajesh Asst.Prof. EEE Department 47
  • 48. What is the percentage saving in feeder copper if the line voltage in a 2-wire d.c. system is raised from 200 volts to 400 volts for the same power transmitted over the same distance and having the same power loss ? 6 October 2020 D.Rajesh Asst.Prof. EEE Department 48
  • 49.  (i) Conductors  (ii) Step-up and step-down transformers  (iii) Line insulators  (iv) Support  (v) Protective devices  (vi) Voltage regulating devices 6 October 2020 D.Rajesh Asst.Prof. EEE Department 49
  • 50. The following two fundamental economic principles which closely influence the electrical design of a transmission line (i) Economic choice of transmission voltage (ii) Economic choice of conductor size 6 October 2020 D.Rajesh Asst.Prof. EEE Department 50
  • 51.  The effect of increased voltage level of transmission on 1. Volume of conductor material used for transmission 2. Line Drop 6 October 2020 D.Rajesh Asst.Prof. EEE Department 51
  • 52. Let a 3-phase A.C. system is used for the transmission, The resistance per conductor is given by R= ρ l/A Power transmitted P= 3𝑉𝐼cos φ I= 𝑃 3𝑉cos φ Total copper losses 𝑃𝑐𝑢= 3𝐼2R =3( 𝑃 3𝑉cos φ )2ρ l 𝐴 A= 𝑃2ρ l 𝑃𝑐𝑢𝑉2𝑐𝑜𝑠2φ The volume of copper used is V=3(A*l) Volume of copper= 3𝑃2ρ 𝑙2 𝑃𝑐𝑢𝑉2𝑐𝑜𝑠2φ Volume of copper 𝛼 1 𝑉2𝑐𝑜𝑠2φ 6 October 2020 D.Rajesh Asst.Prof. EEE Department 52 Thus greater is the transmission voltage level, lesser is the volume of the copper required.
  • 53. Line Drop=IR=I ρ l/A Current density(J)= 𝐼 𝐴 Then Line Drop= 𝐼ρ l 𝐼 𝐽 = Jρ l %Line Drop= Jρ l 𝑉 *100 Higher the transmission voltage level, then lesser the %Line Drop 6 October 2020 D.Rajesh Asst.Prof. EEE Department 53
  • 54.  The most economical area of conductor is that for which the total annual cost of transmission line is minimum . This is known as Kelvin’s Law  The total annual cost of transmission line can be divided broadly into two parts viz.,  Annual charge on capital outlay  Annual cost of energy wasted in the conductor. 6 October 2020 D.Rajesh Asst.Prof. EEE Department 54
  • 55.  This is on account of interest and depreciation on the capital cost of complete installation of transmission line. In case of overhead system, it will be the annual interest and depreciation on the capital cost of conductors, supports and insulators and the cost of their erection.  Now, for an overhead line, insulator cost is constant, the conductor cost is proportional to the area of X-section and the cost of supports and their erection is partly constant annual charge.  Transmission line can be expressed as : 𝐴𝑛𝑛𝑢𝑎𝑙 𝑐ℎ𝑎𝑟𝑔𝑒 = 𝑃1 + 𝑃2𝑎 𝑃1 constant depending on the copper conductor 𝑃2 constant depending on the X-area of the conductor a is the area of X-section of the conductor 6 October 2020 D.Rajesh Asst.Prof. EEE Department 55
  • 56.  This is on account of energy lost mainly in the conductor due to 𝐼2R losses.  Assuming a constant current in the conductor throughout the year,  The energy lost in the conductor is proportional to resistance. where R = ρ l/ 𝑎  The energy lost in the conductor is inversely proportional to area of X-section. energy lost α 1 𝑎  Thus, the annual cost of energy wasted in an overhead transmission line can be expressed as : Annual cost of energy wasted = 𝑃3 𝑎 where P3 is a constant. 6 October 2020 D.Rajesh Asst.Prof. EEE Department 56
  • 57.  Total annual cost, C=𝑃1 + 𝑃2𝑎+ 𝑃3 𝑎 only area of X-section a is variable. Therefore, the total annual cost of transmission line will be minimum if differentiation of C w.r.t. a is zero i.e. ⅆ𝐶 ⅆ𝑎 =0 ⅆ ⅆ𝑎 (𝑃1 + 𝑃2𝑎+ 𝑃3 𝑎 )=0 𝑃2𝑎 − 𝑃3 𝑎2=0 𝑃2𝑎 = 𝑃3 𝑎2 i.e. Variable part of annual charge = Annual cost of energy wasted Therefore Kelvin’s Law can also be stated The most economical area of conductor is that for which the variable part of annual charge is equal to the cost of energy losses per year. 6 October 2020 D.Rajesh Asst.Prof. EEE Department 57
  • 58.  The straight line shows the relation between the annual charge (i.e., 𝑃1 + 𝑃2𝑎) and the area of X- section a of the conductor  The rectangular hyperbola gives the relation between annual cost of energy wasted ( 𝑃3 𝑎 ) and X-sectional area a.  Curve 3 shows the relation between total annual cost of transmission line and area of X-section a. 6 October 2020 D.Rajesh Asst.Prof. EEE Department 58
  • 59.  (i) It is not easy to estimate the energy loss in the line without actual load curves, which are not available at the time of estimation.  (ii) The assumption that annual cost on account of interest and depreciation on the capital outlay is in the form 𝑃1 + 𝑃2𝑎 is strictly speaking not true. For instance, in cables neither the cost of cable dielectric and sheath nor the cost of laying vary in this manner.  (iii) This law does not take into account several physical factors like safe current density, mechanical strength, corona loss etc.  (iv) The conductor size determined by this law may not always be practicable one because it may be too small for the safe carrying of necessary current.  (v) Interest and depreciation on the capital outlay cannot be determined accurately. 6 October 2020 D.Rajesh Asst.Prof. EEE Department 59
  • 60. A 2-conductor cable 1 km long is required to supply a constant current of 200 A throughout the year. The cost of cable including installation is Rs. (20 a + 20) per meter where ‘a’ is the area of X-section of the conductor in 𝑐𝑚2 . The cost of energy is 5P per kWh and interest and depreciation charges amount to 10%. Calculate the most economical conductor size. Assume resistivity of conductor material to be 1·73 μ Ω cm. economical conductor size a =? Energy lost per annum=2𝐼2 𝑅𝑡 Resistance of one conductor=ρ l/ 𝑎 = 1.73∗10−6∗105 𝑎 = 0.173 𝑎 Ω Energy lost per annum= 2∗ 2002 ∗ 0.173 𝑎 ∗ 8760 = 121238.4 𝑎 kWh 6 October 2020 D.Rajesh Asst.Prof. EEE Department 60
  • 61.  Annual cost of energy lost = Cost per kWh × Annual energy loss =Rs 0.05* 121238.4 𝑎 kWh=Rs 6062 𝑎 The capital cost (variable) of the cable is given to be Rs 20 a per metre. Therefore, for 1 km length of the cable, the capital cost (variable) is Rs. 20 a × 1000 = Rs. 20,000 a. Variable annual charge = Annual interest and depreciation on capital cost (variable) of cable = Rs 0·1 × 20,000 a = Rs 2000 a According to Kelvin’s law, for most economical X-section of the conductor, Variable annual charge = Annual cost of energy lost 2000 a = 6062 𝑎 a= 6062 2000 =1.74 𝑐𝑚2 6 October 2020 D.Rajesh Asst.Prof. EEE Department 61
  • 62. A 2-wire feeder carries a constant current of 250 A throughout the year. The portion of capital cost which is proportional to area of X-section is Rs 5 per kg of copper conductor. The interest and depreciation total 10% per annum and the cost of energy is 5P per kWh. Find the most economical area of X-section of the conductor. Given that the density of copper is 8·93gm/ 𝑐𝑚3 and its specific resistance is 1·73 × 10−8 Ω m. Consider 1 metre length of the feeder. Let a be the most economical area of X-section of each conductor in 𝑚2 Energy lost per annum=2𝐼2 𝑅𝑡 Resistance of one conductor=ρ l/ 𝑎 = 1.73∗10−8∗1 𝑎 = 1.73∗10−8 𝑎 Ω Energy lost per annum= 2∗ 2502 ∗ 1.73∗10−8 𝑎 ∗ 8760 = 18,94,350 𝑎 ∗ 10−8 kWh 6 October 2020 D.Rajesh Asst.Prof. EEE Department 62
  • 63.  Annual cost of energy lost = Cost per kWh × Annual energy loss =Rs 0.05* 18,94,350 𝑎 ∗ 10−8 kWh=Rs 94,717.5 𝑎 ∗ 10−8 Mass of 1 metre feeder = 2 (Volume × density) = 2 × a × 1 × 8·93 × 103kg = 17·86 × 103 a kg Capital cost (variable) = Rs 5 × 17·86 × 103 a = Rs 89·3 × 103 a Variable annual charge = 10% of capital cost (variable) = 0·1 × 89·3 × 103 a = Rs 8930 a According to Kelvin’s law, for most economical X-section of the conductor, Variable annual charge = Annual cost of energy lost 8930 a = 94,717.5 𝑎 ∗ 10−8 a= 94,717.5 8930 ∗ 10−8 =3.25∗ 10−4 𝑚2 = 3.25 𝑐𝑚2 6 October 2020 D.Rajesh Asst.Prof. EEE Department 63
  • 64. Determine the most economical cross-section for a 3-phase transmission line, 1 km long to supply at a constant voltage of 110 kV for the following daily load cycle : 6 hours 20 MW at p.f. 0·8 lagging 12 hours 5 MW at p.f. 0·8 lagging 6 hours 6 MW at p.f. 0·8 lagging The line is used for 365 days yearly. The cost per km of line including erection is Rs (9000 +6000 a) where ‘a’ is the area of X- section of conductor in cm2. The annual rate of interest and depreciation is 10% and the energy costs 6P per kWh. The resistance per km of each conductor is0·176/a. Power transmitted P= 3𝑉𝐼cos φ Energy lost per annum=3R(𝐼2 𝑡) 6 October 2020 D.Rajesh Asst.Prof. EEE Department 64
  • 65.  The load currents at various loads are :  At 20 MW, 𝐼1= 𝑃 3𝑉cos φ = 20∗106 3∗110∗103∗0.8 = 131.2𝐴  At 5 MW, 𝐼1= 𝑃 3𝑉cos φ = 5∗106 3∗110∗103∗0.8 = 32.8𝐴  At 6 MW, 𝐼1= 𝑃 3𝑉cos φ = 6∗106 3∗110∗103∗0.8 = 39.36𝐴  Energy loss per day in 3-phase line =3* 0.176 𝑎 (131.22 ∗ 6+ 32.82 ∗ 12+ 36.362 ∗ 6) = 66.26 𝑎 kWh Energy lost per annum = 66.26 𝑎 *365= = 24,184.9 𝑎 kWh Annual cost of energy=Rs 6 100 ∗ 24,184.9 𝑎 = = 1451.09 𝑎 Variable annual charge = 10% of capital cost (variable) of line = Rs 0·1 × 6000 a = Rs 600 a According to Kelvin’s law, for most economical X-section of the conductor, Variable annual charge = Annual cost of energy 600 a= 1451.09 𝑎 a= 1451.09 600 =1.56 𝑐𝑚2 6 October 2020 D.Rajesh Asst.Prof. EEE Department 65
  • 66. The conveyance of electric power from a power station to consumers’ premises is known as electric supply system.  Electric supply system consist of three principle components: 1) Power station 2) Transmission lines 3) Distribution system  The electric supply system can be broadly classified into: 1) DC or AC system 2)Overhead or underground system 6 October 2020 D.Rajesh Asst.Prof. EEE Department 66
  • 67. Different blocks of typical acpowersupply scheme are asper following:: 1)Generating Station: Ingenerating station power is producedby three phase alternators operating inparallel. The usual generation voltage is 11kV. Foreconomy in the transmission of electric power, the generation voltage is stepped upto 132 kVor more at generating station with the help of three phase transformer. 6 October 2020 D.Rajesh Asst.Prof. EEE Department 67
  • 68. 2)Primarytransmission: Theelectric power at 132 kVis transmitted by 3-phase,3 wire overhead system to the out 3)Secondary transmission: Atthe receiving station the voltage is reduced to33kV by step down transformer. 4)Primary distribution: Atthe substation voltage is reduced 33kV to 11kV.The11kV line run along important roadsides to city. This forms the primary distribution.
  • 69. 5)Secondarydistribution: Theelectric power form primary distribution line is delivered to distribution substation. The substation is located nearthe consumers location and step down the voltage to 400V, 3- phase ,4-wire for secondary distribution. The voltage between any two phases is 400V and between any phase and neutral is 230V.
  • 70. Basis for Comparison AC Transmission Line DC Transmission Line Definition The ac transmission line transmit the alternating current. The dc transmission line is used for transmitting the direct current. Number of Conductors Three Two Inductance & surges Have Don’t Have Voltage drop High Low Skin Effect Occurs Absent Need of Insulation More Less Interference Have Don’t Have Corona Loss Occur Don’t occur Dielectric Loss Have Don’t have Synchronizing and Stability Problem No difficulties Difficulties Cost Expensive Cheap Length of conductors Small More Repairing and Maintenance Easy and Inexpensive Difficult and Expensive Transformer Requires Not Requires 6 October 2020 D.Rajesh Asst.Prof. EEE Department 70
  • 71. 6 October 2020 D.Rajesh Asst.Prof. EEE Department 71
  • 72. 6 October 2020 D.Rajesh Asst.Prof. EEE Department 72
  • 73. Which distributes electric power for local use is known as distribution system. 6 October 2020 D.Rajesh Asst.Prof. EEE Department 73
  • 74.  (i) Feeders. A feeder is a conductor which connects the sub-station to the area where power is to be distributed.  (ii) Distributor. A distributor is a conductor from which tappings are taken for supply to the consumers.  (iii) Service mains. A service mains is generally a small cable which connects the distributor to the consumers’ terminals. 6 October 2020 D.Rajesh Asst.Prof. EEE Department 74
  • 75. 6 October 2020 D.Rajesh Asst.Prof. EEE Department 75
  • 76. Primary distribution system 6 October 2020 D.Rajesh Asst.Prof. EEE Department 76 Secondary distribution system
  • 77. 2-wire d.c. system 6 October 2020 D.Rajesh Asst.Prof. EEE Department 77 3 -wire d.c. system
  • 78. 6 October 2020 D.Rajesh Asst.Prof. EEE Department 78
  • 79. 6 October 2020 D.Rajesh Asst.Prof. EEE Department 79
  • 80.  (i) Radial System. In this system, separate feeders radiate from a single substation and feed the distributors at one end only. 6 October 2020 D.Rajesh Asst.Prof. EEE Department 80
  • 81.  Limitations  (a) The end of the distributor nearest to the feeding point will be heavily loaded.  (b) The consumers are dependent on a single feeder and single distributor. Therefore, any fault on the feeder or distributor cuts off supply to the consumers who are on the side of the fault away from the substation.  (c) The consumers at the distant end of the distributor would be subjected to serious voltage fluctuations when the load on the distributor changes. Due to these limitations, this system is used for short distances only. 6 October 2020 D.Rajesh Asst.Prof. EEE Department 81
  • 82.  (ii) Ring main system. In this system, the primaries of distribution transformers form a loop. The loop circuit starts from the substation bus-bars, makes a loop through the area to be served, and returns to the substation. ✓ substation supplies to the closed feeder LMNOPQRS. ✓ The distributors are tapped from different points M, O and Q of the feeder through distribution transformers. ➢ There are less voltage fluctuations at consumer’s terminals. ➢ The system is very reliable as each distributor is fed via two feeders. ➢ In the event of fault on any section of the feeder, the continuity of supply is maintained. 6 October 2020 D.Rajesh Asst.Prof. EEE Department 82
  • 83.  (iii) Interconnected system. When the feeder ring is energized by two or more than two generating stations or substations, it is called inter-connected system. (a) It increases the service reliability. (b) Any area fed from one generating station during peak load hours can be fed from the other generating station. This reduces reserve power capacity and increase efficiency of the system. 6 October 2020 D.Rajesh Asst.Prof. EEE Department 83
  • 84.  (i) Proper voltage.  One important requirement of a distribution system is that voltage variations at consumer’s terminals should be as low as possible.  The changes in voltage are generally caused due to the variation of load on the system.  Low voltage causes loss of revenue, inefficient lighting and possible burning out of motors.  High voltage causes lamps to burn out permanently and may cause failure of other appliances.  Thus, if the declared voltage is 230 V, then the highest voltage of the consumer should not exceed 244 V while the lowest voltage of the consumer should not be less than 216 V. 6 October 2020 D.Rajesh Asst.Prof. EEE Department 84
  • 85.  (ii) Availability of power on demand. Power must be available to the consumers in any amount that they may require from time to time.  (iii) Reliability. Modern industry is almost dependent on electric power for its operation.  The reliability can be improved to a considerable extent by (a) interconnected system (b) reliable automatic control system (c) providing additional reserve facilities. 6 October 2020 D.Rajesh Asst.Prof. EEE Department 85
  • 86. (i) Distributor fed at one end (ii) Distributor fed at both ends (iii) Distributor fed at the centre (iv) Ring distributor. 6 October 2020 D.Rajesh Asst.Prof. EEE Department 86
  • 87.  (i) Distributor fed at one end 6 October 2020 D.Rajesh Asst.Prof. EEE Department 87  (ii) Distributor fed at both ends
  • 88.  (iii) Distributor fed at the centre 6 October 2020 D.Rajesh Asst.Prof. EEE Department 88  (iv) Ring distributor
  • 89. (i) Concentrated loading (ii) Uniform loading (iii) Both concentrated and uniform loading. 6 October 2020 D.Rajesh Asst.Prof. EEE Department 89
  • 90.  2-wire d.c. distributor AB fed at one end A and having concentrated loads 𝐼1, 𝐼2, 𝐼3 and 𝐼4 tapped off at points C, D, E and F respectively.  Let 𝑟1 , 𝑟2, 𝑟3 𝑎𝑛𝑑 𝑟4 be the resistances of both wires (go and return) of the sections AC, CD, DE and EF of the distributor respectively.  Current fed from point A = 𝐼1 + 𝐼2 + 𝐼3 + 𝐼4 Current in section AC = 𝐼1 + 𝐼2 + 𝐼3 + 𝐼4  Current in section CD = 𝐼2 + 𝐼3 + 𝐼4 Current in section DE = 𝐼3 + 𝐼4  Current in section EF = 𝐼4  Voltage drop in section AC = 𝑟1 (𝐼1 + 𝐼2 + 𝐼3 + 𝐼4)  Voltage drop in section CD = 𝑟2 (𝐼2 + 𝐼3 + 𝐼4) Voltage drop in section DE = 𝑟3 (𝐼3 + 𝐼4)  Voltage drop in section EF = 𝑟4 𝐼4  ∴ Total voltage drop in the distributor =𝑟1 (𝐼1 + 𝐼2 + 𝐼3 + 𝐼4)+𝑟2 (𝐼2 + 𝐼3 + 𝐼4)+𝑟3 (𝐼3 + 𝐼4)+ 𝑟4 𝐼4 6 October 2020 D.Rajesh Asst.Prof. EEE Department 90
  • 91.  2-wire d.c. distributor AB fed at one end A and loaded uniformly with i amperes per metre length.  The load tapped is i amperes. Let 𝑙 metres be the length of the distributor and r ohm be the resistance per metre run. 6 October 2020 D.Rajesh Asst.Prof. EEE Department 91
  • 92.  Consider a point C on the distributor at a distance x metres from the feeding point A  Then current at point C is = i 𝑙− i x amperes = i (𝑙− x) amperes Now, consider a small length dx near point C. Its resistance is r dx and the voltage drop over length dx is dv = i (𝑙 − x) r dx = i r (𝑙 − x) dx Total voltage drop in the distributor upto point C is v =‫׬‬0 𝑥 ir (l − x)dx =ir 𝑙𝑥 − 𝑥2 2 The voltage drop upto point B (i.e. over the whole distributor) can be obtained by putting x = 𝑙 in the above expression. ∴ Voltage drop over the distributor AB = ir 𝑙 ∗ 𝑙 − 𝑙2 2 = 1 2 𝑖𝑟𝑙2 = 1 2 𝑖𝑙 𝑟𝑙 = 1 2 𝐼𝑅 Thus, in a uniformly loaded distributor fed at one end, the total voltage drop is equal to that produced by the whole of the load assumed to be concentrated at the middle point. 6 October 2020 D.Rajesh Asst.Prof. EEE Department 92
  • 93.  The two ends of the distributor may be supplied with (i) equal voltages (ii) unequal voltages. 6 October 2020 D.Rajesh Asst.Prof. EEE Department 93
  • 94.  Consider a distributor AB fed at both ends with equal voltages V volts and having concentrated loads 𝐼1, 𝐼2, 𝐼3, 𝐼4 and 𝐼5 at points C, D, E, F and G respectively  As we move away from one of the feeding points, say A, p.d. goes on decreasing till it reaches the minimum value at some load point, say E, and then again starts rising and becomes V volts as we reach the other feeding point B.  All the currents tapped off between points A and E (minimum p.d. point) will be supplied from the feeding point A while those tapped off between B and E will be supplied from the feeding point B.  The current tapped off at point E itself will be partly supplied from A and partly from B. If these currents are x and y respectively, then, x+y= 𝐼3  Therefore, we arrive at a very important conclusion that at the point of minimum potential, current comes from both ends of the distributor.  Voltage drop from feeding point A to the point of minimum potential difference is equal to the voltage drop from feeding point B to the point of minimum potential difference 6 October 2020 D.Rajesh Asst.Prof. EEE Department 94
  • 95.  Distributor AB fed with unequal voltages ; end A being fed at 𝑉1volts and end B at 𝑉2 volts.  In order to determine the point of minimum potential, assume of 𝐼𝐴 amperes is supplied from feeding point A  Let r be the resistance of each conductor per unit length(l=1) for both go & return  Resistance of portions AC,CD, DE,EF,FB are 𝑟1 , 𝑟2, 𝑟3 , 𝑟4, 𝑎𝑛𝑑 𝑟5 respectively  Voltage drop between A and B = Voltage drop over AB 𝑉1 − 𝑉2 = 𝐼𝐴𝑟1+(𝐼𝐴- 𝐼1) 𝑟2 + (𝐼𝐴- 𝐼1- 𝐼2) 𝑟3 + (𝐼𝐴- 𝐼1- 𝐼2- 𝐼3) 𝑟4+ (𝐼𝐴- 𝐼1- 𝐼2- 𝐼3 − 𝐼4) 𝑟5 The load point at which currents are coming from both sides of the distributor will be the point of minimum potential 6 October 2020 D.Rajesh Asst.Prof. EEE Department 95
  • 96.  A 2-wire d.c. street mains AB, 600 m long is fed from both ends at 220 V. Loads of 20 A, 40 A, 50 A and 30 A are tapped at distances of 100m, 250m, 400m and 500 m from the end A respectively. If the area of X-section of distributor conductor is 1𝑐𝑚2 , find the minimum consumer voltage. Take ρ = 1·7 × 10−6 Ω cm.  Voltage drop between A and B = Voltage drop over AB R= ρ l/A  Resistance of section AC of distributor=2× 1·7 × 10−6∗100∗102 1 = 3·4 × 10−2 Ω  Resistance of section CD of distributor =0.051 Ω  Resistance of section DE of distributor =0.051 Ω  Resistance of section EF of distributor =0.034 Ω  Resistance of section FB of distributor =0.034 Ω  Voltage drop between A and B = Voltage drop over AB  𝑉𝐴 − 𝑉𝐵 = 𝐼𝐴𝑟1+(𝐼𝐴- 𝐼1) 𝑟2 + (𝐼𝐴- 𝐼1- 𝐼2) 𝑟3 + (𝐼𝐴- 𝐼1- 𝐼2- 𝐼3) 𝑟4+ (𝐼𝐴- 𝐼1- 𝐼2- 𝐼3 − 𝐼4) 𝑟5  220-200=0.034 𝐼𝐴+(𝐼𝐴-20)0.051+ (𝐼𝐴-60)0.051+(𝐼𝐴-110)0.034+(𝐼𝐴-140)0.034 𝐼𝐴=61.7A 6 October 2020 D.Rajesh Asst.Prof. EEE Department 96
  • 97.  Actual distribution of currents in the various sections of the distributor is  Current in section EF, 𝐼𝐸𝐹 = 𝐼𝐴−110 =61·7−110 =−48·3A from E to F = 48·3 A from F to E  Current in section DE, 𝐼𝐷𝐸 = 𝐼𝐴 −60=61.7-60=1.7A from D to E 6 October 2020 D.Rajesh Asst.Prof. EEE Department 97 It is clear that currents are coming to load point E from both sides i.e. from point D and point F. Hence, E is the point of minimum potential. ∴ Minimum consumer voltage, 𝑉𝐸 = 𝑉𝐴-(𝐼𝐴𝐶 𝑟1+ 𝐼𝐶𝐷 𝑟2+ 𝐼𝐷𝐸 𝑟3) =220-(61·7 × 0·034 + 41·7 × 0·051 + 1·7 × 0·051] =215.69V
  • 98. A two-wire d.c. distributor AB, 600 metres long is loaded as under : Distance from A (metres) : 150 300 350 450 Loads in Amperes : 100 200 250 300 The feeding point A is maintained at 440 V and that of B at 430 V. If each conductor has a resistance of 0·01 Ω per 100 metres, calculate : (i) the currents supplied from A to B, (ii) the power dissipated in the distributor. Let 𝐼𝐴amperes be the current supplied from the feeding point A. Then currents in the various sections of the distributor are Resistance of 100 m length of distributor (both wires) = 2 × 0·01 = 0·02 Ω Resistance of section AC, 𝑟1 = 0·02 × 150/100 = 0·03 Ω Resistance of section CD, 𝑟2 = 0·02 × 150/100 = 0·03 Ω Resistance of section DE, 𝑟3= 0·02 × 50/100 = 0·01 Ω Resistance of section EF, 𝑟4= 0·02 × 100/100 = 0·02 Ω Resistance of section FB, 𝑟5= 0·02 × 150/100 = 0·03 Ω 6 October 2020 D.Rajesh Asst.Prof. EEE Department 98
  • 99.  Voltage drop between A and B = Voltage drop over AB  𝑉𝐴 − 𝑉𝐵 = 𝐼𝐴𝑟1+(𝐼𝐴- 𝐼1) 𝑟2 + (𝐼𝐴- 𝐼1- 𝐼2) 𝑟3 + (𝐼𝐴- 𝐼1- 𝐼2- 𝐼3) 𝑟4+ (𝐼𝐴- 𝐼1- 𝐼2- 𝐼3 − 𝐼4) 𝑟5  440-430=0.03 𝐼𝐴+(𝐼𝐴-100)0.03+ (𝐼𝐴-300)0.01+(𝐼𝐴-550)0.02+(𝐼𝐴-850)0.03 𝐼𝐴=437.5A  The actual distribution of currents in the various sections of the distributor are  Current supplied from end A, 𝐼𝐴 = 𝐼𝐴𝐶 = 437·5 A  Current supplied from end B, 𝐼𝐵 = 𝐼𝐹𝐵 = 𝐼𝐴 -850= - 412·5 A  𝐼𝐶𝐷 = 337.5𝐴 𝐼𝐷𝐸 = 137.5𝐴 𝐼𝐸𝐹 = −112.5𝐴  Therefore E is the point of minimum potential.  Power loss in the distributor =𝐼𝐴𝐶 2 𝑟1+ 𝐼𝐶𝐷 2 𝑟2 + 𝐼𝐷𝐸 2 𝑟3+ 𝐼𝐸𝐹 2 𝑟4+ 𝐼𝐹𝐵 2 𝑟5 =(437.5)2 ∗ 0.03 + 337.5 2 ∗ 0.03 + 137.5 2 ∗ 0.01 + 112.5 2 ∗ 0.02 + (412.5)2 ∗ 0.03 =14.705kW 6 October 2020 D.Rajesh Asst.Prof. EEE Department 99
  • 100.  (i) Distributor fed at both ends with equal voltages.  Consider a distributor AB of length 𝑙 metres, having resistance r ohms per metre run and with uniform loading of 𝑖 amperes per metre run.  Let the distributor be fed at the feeding points A and B at equal voltages, say V volts.  The total current supplied to the distributor is 𝑖𝑙.  As the two end voltages are equal, therefore, current supplied from each feeding point is 𝑖𝑙 2 i.e. 6 October 2020 D.Rajesh Asst.Prof. EEE Department 100
  • 101.  Consider a point C at a distance x metres from the feeding point A. Then current at point C is = 𝑖𝑙 2 - 𝑖𝑥= 𝑖( 𝑙 2 -𝑥) Now, consider a small length dx near point C. Its resistance is r dx and the voltage drop over length dx is dv = i ( 𝑙 2 − x) r dx = i r ( 𝑙 2 − x) dx Total voltage drop in the distributor upto point C is v =‫׬‬ 0 𝑥 ir ( 𝑙 2 − x)dx =ir 𝑙𝑥 2 − 𝑥2 2 = 𝑖𝑟 2 (𝑙𝑥- 𝑥2) Obviously, the point of minimum potential will be the mid-point. Therefore, maximum voltage drop will occur at mid-point i.e. where x = 𝑙 2 Max. voltage drop= 𝑖𝑟 2 (𝑙𝑥- 𝑥2)= 𝑖𝑟 2 (𝑙 𝑙 2 - ( 𝑙 2 )2)= 1 8 𝑖𝑟𝑙2= 1 8 𝑖𝑙 𝑟𝑙 = 1 8 IR Minimum voltage=V- 1 8 IR volts 6 October 2020 D.Rajesh Asst.Prof. EEE Department 101
  • 102.  (ii) Distributor fed at both ends with unequal voltages.  Consider a distributor AB of length l metres having resistance r ohms per metre run and with a uniform loading of i amperes permetre run. Let the distributor be fed from feeding points A and B at voltages 𝑉𝐴 and 𝑉𝐵 respectively.  Suppose that the point of minimum potential C is situated at a distance x metres from the feeding point A. Then current supplied by the feeding point A will be i x.  ∴ Voltage drop in section AC= 1 2 𝑖𝑟𝑥2 volts  As the distance of C from feeding point B is (l − x), therefore, current fed from B is i (l − x).  ∴ Voltage drop in section BC = 1 2 𝑖𝑟(𝑙 − 𝑥)2 volts 6 October 2020 D.Rajesh Asst.Prof. EEE Department 102
  • 103.  Voltage at point C, 𝑉𝐶 = 𝑉𝐴 − Drop over AC = 𝑉𝐴 - 1 2 𝑖𝑟𝑥2 Voltage at point C, 𝑉𝐶 = 𝑉𝐵 − Drop over BC = 𝑉𝐵 - 1 2 𝑖𝑟(𝑙 − 𝑥)2 From above equations we get, 𝑉𝐴 - 1 2 𝑖𝑟𝑥2 = 𝑉𝐵 - 1 2 𝑖𝑟(𝑙 − 𝑥)2 𝑉𝐴- 𝑉𝐵 = 1 2 𝑖𝑟𝑥2 - 1 2 𝑖𝑟(𝑙 − 𝑥)2 = 1 2 𝑖𝑟𝑥2 - 1 2 𝑖𝑟𝑙2 - 1 2 𝑖𝑟𝑥2 + 1 2 𝑖𝑟2𝑙𝑥 = 𝑖𝑟𝑙 {𝑥- 1 2 𝑙 } 𝑥= 𝑉𝐴− 𝑉𝐵 𝑖𝑟𝑙 + 𝑙 2 The point on the distributor where minimum potential occurs can be calculated 6 October 2020 D.Rajesh Asst.Prof. EEE Department 103
  • 104.  Expression for the power loss in a uniformly loaded distributor fed at both ends  current supplied from each feeding point is 𝑖𝑙 2  Consider a small length dx of the distributor at point P which is at a distance x from the feeding end A.  Resistance of length dx = r dx  Current in length dx = 𝑖𝑙 2 - 𝑖𝑥= 𝑖( 𝑙 2 -𝑥)  Power loss in length dx = (current in dx)2 × Resistance of dx = 𝑖( 𝑙 2 −𝑥) 2 ×rdx  Total power loss in the distributor is=‫׬‬ 0 𝑙 𝑖( 𝑙 2 −𝑥) 2 ×rdx =𝑖2 𝑟 ‫׬‬ 0 𝑙 𝑙2 4 − 𝑙𝑥 + 𝑥2 𝑑𝑥 = 𝑖2𝑟 𝑙2𝑥 4 − 𝑙𝑥2 2 + 𝑥3 3 𝑙 0 P= 𝑖2𝑟𝑙3 12 6 October 2020 D.Rajesh Asst.Prof. EEE Department 104
  • 105. A 2-wire d.c. distributor AB 500 metres long is fed from both ends and is loaded uniformly at the rate of 1·0 A/metre. At feeding point A, the voltage is maintained at 255 V and at B at 250 V. If the resistance of each conductor is 0·1 Ω per kilometre, determine : (i) the minimum voltage and the point where it occurs (ii) the currents supplied from feeding points A and B Let the minimum potential occur at a point C distant x metres from the feeding point A. 𝑥= 𝑉𝐴− 𝑉𝐵 𝑖𝑟𝑙 + 𝑙 2 Resistance of distributor/m, r = 2 × 0·1/1000 = 0·0002 Ω 𝑥= 255− 250 1∗0.0002∗500 + 500 2 =300m i.e. minimum potential occurs at 300 m from point A. Minimum voltage, 𝑉𝐶 = 𝑉𝐴 - 1 2 𝑖𝑟𝑥2 = 𝑉𝐵 - 1 2 𝑖𝑟(𝑙 − 𝑥)2 255- 1∗0.0002∗ (300)2 2 =246V Current supplied from A = i x = 1 × 300 = 300 A Current supplied from B = i (𝑙 − x) = 1 (500 − 300) = 200 A 6 October 2020 D.Rajesh Asst.Prof. EEE Department 105
  • 106. A 800 metres 2-wire d.c. distributor AB fed from both ends is uniformly loaded at the rate of 1·25 A/metre run. Calculate the voltage at the feeding points A and B if the minimum potential of 220 V occurs at point C at a distance of 450 metres from the end A. Resistance of each conductor is 0·05 Ω/km.  Minimum voltage, 𝑉𝐶 = 𝑉𝐴 - 1 2 𝑖𝑟𝑥2 = 𝑉𝐵 - 1 2 𝑖𝑟(𝑙 − 𝑥)2  For 𝑉𝐴 𝑉𝐶 = 𝑉𝐴 - 1 2 𝑖𝑟𝑥2 𝑉𝐴=220+ 1.25∗0.0001∗ (450)2 2 =232.65V  For 𝑉𝐵 𝑉𝐶 = 𝑉𝐵 - 1 2 𝑖𝑟(𝑙 − 𝑥)2 𝑉𝐵 =220+ 1.25∗0.0001∗ (800−450)2 2 =227.65V 6 October 2020 D.Rajesh Asst.Prof. EEE Department 106
  • 107.  (i) A uniformly loaded distributor is fed at the centre. Show that maximum voltage drop = I R/8 where I is the total current fed to the distributor and R is the total resistance of the distributor.  (ii) A 2-wire d.c. distributor 1000 metres long is fed at the centre and is loaded uniformly at the rate of 1·25 A/metre. If the resistance of each conductor is 0·05 Ω/km, find the maximum voltage drop in the distributor.  ∴ Max. voltage drop = Voltage drop in half distributor = 1 2 ( 𝑖𝑙 2 )( 𝑟𝑙 2 ) = 1 8 𝑖𝑙 𝑟𝑙 = 1 8 IR where i l = I, the total current fed to the distributor r l = R, the total resistance of the distributor  (ii) Total current fed to the distributor is  I = i l = 1·25 × 1000 = 1250 A  Total resistance of the distributor is  R = r l = 2 × 0·05 × 1 = 0·1 Ω  Max. voltage drop = 1 8 IR = 1 8 *1250*0.1 =15.62V 6 October 2020 D.Rajesh Asst.Prof. EEE Department 107
  • 108.  A two-wire d.c. distributor cable 1000 metres long is loaded with 0·5 A/metre. Resistance of each conductor is 0·05 Ω/km. Calculate the maximum voltage drop if the distributor is fed from both ends with equal voltages of 220 V. What is the minimum voltage and where it occurs ?  Current loading, i = 0·5 A/m  Resistance of distributor/m, r = 2 × 0·05/1000 = 0·1 × 10−3 Ω  Length of distributor, l = 1000 m  Total current supplied by distributor, I = i l = 0·5 × 1000 = 500 A  Total resistance of the distributor, R = r l = 0·1 × 10−3 × 1000 = 0·1 Ω  ∴ Max. voltage drop = 1 8 IR = 6·25 V  Minimum voltage will occur at the mid-point of the distributor and its value is  = 220 − 6·25 = 213·75 V 6 October 2020 D.Rajesh Asst.Prof. EEE Department 108
  • 109. The total drop over any section of the distributor is equal to the sum of drops due to concentrated and uniform loading in that section. 6 October 2020 D.Rajesh Asst.Prof. EEE Department 109
  • 110. Two conductors of a d.c. distributor cable AB 1000 m long have a total resistance of 0·1 Ω. The ends A and B are fed at 240 V. The cable is uniformly loaded at 0·5 A per metre length and has concentrated loads of 120 A, 60 A, 100 A and 40 A at points distant 200 m, 400 m, 700 m and 900 m respectively from the end A. Calculate (i) the point of minimum potential (ii) currents supplied from ends A and B (iii) the value of minimum potential. Distributor resistance per metre length, r = 0·1/1000 = 10−4 Ω (i) Point of minimum potential. The point of minimum potential is not affected by the uniform loading of the distributor. Therefore, let us consider the concentrated loads. Suppose the current supplied by end A is 𝐼𝐴. Then currents in the various sections will be 6 October 2020 D.Rajesh Asst.Prof. EEE Department 110
  • 111.  Voltage drop between A and B = Voltage drop over AB  𝑉𝐴 − 𝑉𝐵 = 𝐼𝐴𝑟1+(𝐼𝐴- 𝐼1) 𝑟2 + (𝐼𝐴- 𝐼1- 𝐼2) 𝑟3 + (𝐼𝐴- 𝐼1- 𝐼2- 𝐼3) 𝑟4+ (𝐼𝐴- 𝐼1- 𝐼2- 𝐼3 − 𝐼4) 𝑟5  0= 10−4( ) 200 𝐼𝐴+(𝐼𝐴−120)200+ (𝐼𝐴−180)300+(𝐼𝐴−280)200+(𝐼𝐴−320)100 𝐼𝐴=166A The actual distribution of currents in the various sections of the distributor due to concentrated loading. It is clear from this figure that D is the point of minimum potential. 6 October 2020 D.Rajesh Asst.Prof. EEE Department 111
  • 112. (ii) The feeding point A will supply 166 A due to concentrated loading plus 0·5 × 400 = 200 A due to uniform loading. ∴ Current supplied byA, 𝐼𝐴 = 166 + 200 = 366 A The feeding point B will supply a current of 154 A due to concentrated loading plus 0·5 × 600 = 300 A due to uniform loading. ∴ Current supplied by B, 𝐼𝐵 = 154 + 300 = 454 A (iii) ∴ Minimum potential, 𝑉𝐷 = 𝑉𝐴 − Drop in AD due to conc. loading – Drop in AD due to uniform loading 𝑉𝐷 Now, Drop in AD due to conc. loading = 𝐼𝐴𝐶 𝑅𝐴𝐶 + 𝐼𝐶𝐷 𝑅𝐶𝐷 = 166 × 10−4 × 200 + 46 × 10−4 × 200 = 3·32 + 0·92 = 4·24 V Drop in AD due to uniform loading = 1 2 𝑖𝑟𝑙2 = 1 2 0.5 ∗ 10−4 ∗ (400)2 =4V ∴ 𝑉𝐷 = 240 − 4·24 − 4 = 231·76 V 6 October 2020 D.Rajesh Asst.Prof. EEE Department 112
  • 113. A d.c. 2-wire distributor AB is 500m long and is fed at both ends at 240 V. The distributor is loaded as shown in Fig. The resistance of the distributor (go and return) is 0·001Ω per metre. Calculate (i) the point of minimum voltage and (ii) the value of this voltage. Let D be the point of minimum potential and let x be the current flowing in section CD. Then current supplied by end B will be (60 − x). 6 October 2020 D.Rajesh Asst.Prof. EEE Department 113
  • 114.  (i) If r is the resistance of the distributor (go and return) per metre length, then, Voltage drop in length AD = 𝐼𝐴𝐶 𝑅𝐴𝐶 + 𝐼𝐶𝐷 𝑅𝐶𝐷 =(100+x)*100r+x*150r Voltage drop in length BD= 1 2 𝑖𝑟𝑙2 +(60-x)*250r = 1 2 1 ∗ 𝑟 ∗ (200)2 +(60-x)*250r Since the feeding points A and B are at the same potential (100+x)*100r+x*150r= 1 2 1 ∗ 𝑟 ∗ (200)2 +(60-x)*250r X=50A The actual directions of currents in the various sections of the distributor are currents supplied by A and B meet at D. Hence point D is the point of minimum 6 October 2020 D.Rajesh Asst.Prof. EEE Department 114
  • 115.  (ii) Total current = 160 + 1 × 200 = 360 A  Current supplied by A, 𝐼𝐴 = 100 + x = 100 + 50 = 150 A  Current supplied by B, 𝐼𝐵 = 360 − 150 = 210 A  Minimum potential, 𝑉𝐷 = 𝑉𝐴 − (𝐼𝐴𝐶 𝑅𝐴𝐶 + 𝐼𝐶𝐷 𝑅𝐶𝐷 ) = 240 − 150 × (100 × 0·001) − 50 × (150 × 0·001) = 240 − 15 − 7·5 = 217·5 V 6 October 2020 D.Rajesh Asst.Prof. EEE Department 115
  • 116.  A distributor arranged to form a closed loop and fed at one or more points is called a ring distributor.  For the purpose of calculating voltage distribution, the distributor can be considered as consisting of a series of open distributors fed at both ends.  The principal advantage of ring distributor is that by proper choice in the number of feeding points, great economy in copper can be affected. 6 October 2020 D.Rajesh Asst.Prof. EEE Department 116
  • 117.  Sometimes a ring distributor has to serve a large area. In such a case, voltage drops in the various sections of the distributor may become excessive.  In order to reduce voltage drops in various sections, distant points of the distributor are joined through a conductor called interconnector. 6 October 2020 D.Rajesh Asst.Prof. EEE Department 117
  • 118.  The ring distributor ABCDEA. The points B and D of the ring distributor are joined through an interconnector BD.  Network can be obtained by applying Thevenin’s theorem.  (i) Consider the interconnector BD to be disconnected and find the potential difference between B and D. This gives Thevenin’s equivalent circuit voltage 𝐸0.  (ii) Next, calculate the resistance viewed from points B and D of the network composed of distribution lines only. This gives Thevenin’s equivalent circuit series resistance 𝑅0.  (iii) If 𝑅𝐵𝐷 is the resistance of the interconnector BD, then Thevenin’s equivalent circuit will be ∴ Current in interconnector BD = 𝐸0 𝑅0+𝑅𝐵𝐷  Therefore, current distribution in each section and the voltage of load points can be calculated. 6 October 2020 D.Rajesh Asst.Prof. EEE Department 118
  • 119. A 2-wire d.c. distributor ABCDEA in the form of a ring main is fed at point A at 220 V and is loaded as under : 10A at B ; 20A at C ; 30A at D and 10 A at E. The resistances of various sections (go and return) are : AB = 0·1 Ω ; BC = 0·05 Ω ; CD = 0·01 Ω ; DE = 0·025 Ω and EA = 0·075 Ω. Determine :  (i) the point of minimum potential  (ii) current in each section of distributor Let us suppose that current I flows in section AB of the distributor. Then currents in the various sections of the distributor are According to Kirchhoff’s voltage law, the voltage drop in the closed loop ABCDEA is zero i.e. 𝐼𝐴𝐵𝑅𝐴𝐵+𝐼𝐵𝐶𝑅𝐵𝐶+ 𝐼𝐶𝐷𝑅𝐶𝐷+ 𝐼𝐷𝐸𝑅𝐷𝐸 + 𝐼𝐸𝐴𝑅𝐸𝐴 =0 0.1𝐼+(𝐼-10)0.05+ (𝐼-30)0.01+(𝐼-60)0.025+(𝐼-70)0.075=0 0.26 𝐼 = 7.55 𝐼 = 29.04A 6 October 2020 D.Rajesh Asst.Prof. EEE Department 119
  • 120.  (ii) Current in section AB = I = 29·04 A from A to B  Current in section BC = I − 10 = 29·04 − 10 = 19·04 A from B to C  Current in section CD = I − 30 = 29·04 − 30 = − 0·96 A = 0·96 A from D to C  Current in section DE = I − 60 = 29·04 − 60 = − 30·96 A = 30·96 A from E to D  Current in section EA = I − 70 = 29·04 − 70 = − 40·96 A = 40·96 A from A to E 6 October 2020 D.Rajesh Asst.Prof. EEE Department 120 ∴ C is the point of minimum potential.
  • 121.  A d.c. ring main ABCDA is fed from point A from a 250 V supply and the resistances (including both lead and return) of various sections are as follows : AB = 0·02 Ω ; BC = 0·018 Ω ; CD = 0·025 Ω and DA = 0·02 Ω. The main supplies loads of 150 A at B ; 300 A at C and 250 A at D. Determine the voltage at each load point. If the points A and C are linked through an interconnector of resistance 0·02 Ω, determine the new voltage at each load point.  Without Interconnector.  Let us suppose that a current I flows in section AB of the distributor. Then currents in various sections of the distributor will be  According to Kirchhoff’s voltage law, the voltage drop in the closed loop ABCDA is zero i.e.  𝐼𝐴𝐵𝑅𝐴𝐵+𝐼𝐵𝐶𝑅𝐵𝐶+ 𝐼𝐶𝐷𝑅𝐶𝐷+ 𝐼𝐷𝐴𝑅𝐷𝐴 =0  0.02𝐼+(𝐼-150)0.018+ (𝐼-450)0.025+(𝐼-700)0.02=0 0·083 I = 27·95 ∴ I = 27·95/0·083 = 336·75 A 6 October 2020 D.Rajesh Asst.Prof. EEE Department 121
  • 122. The actual distribution of currents are ..  Current in section AB = I = 336.75 A from A to B  Current in section BC = I − 150 = 336.75 − 150 = 186.75 A from B to C  Current in section CD = I − 450 = 336.75 − 450 = − 113.25 A = 113.25 A from D to C  Current in section DA = I − 700 = 336.75 − 700 = − 363.25A = 363.25 A from A to D Voltage drop in AB = 336·75 × 0·02 = 6·735 V Voltage drop in BC = 186·75 × 0·018 = 3·361 V Voltage drop in CD = 113·25 × 0·025 = 2·831 V Voltage drop in DA = 363·25 × 0·02 = 7·265 V ∴ Voltage at point B = 250 − 6·735 = 243·265 V Voltage at point C = 243·265 − 3·361 = 239·904 V Voltage at point D = 239·904 + 2·831 = 242·735 V 6 October 2020 D.Rajesh Asst.Prof. EEE Department 122
  • 123.  With Interconnector.  The ring distributor with interconnector AC. The current in the interconnector can be found by applying Thevenin’s theorem. 𝐸0 = Voltage between points A and C = 250 − 239·904 = 10·096 V 𝑅0 = Resistance viewed from points A and C = 0.02+0.018 0.02+0.025 0.02+0.018 + 0.02+0.025 = 0.02Ω 𝑅𝐴𝐶 = Resistance of interconnector = 0·02 Ω  From Thevenin’s equivalent circuit Current in interconnector AC is 𝐸0 𝑅0 + 𝑅𝐴𝐶 = 10.096 0.02 + 0.02 = 252.4𝐴 𝑓𝑟𝑜𝑚 𝐴 𝑡𝑜 𝐶 Let us suppose that current in section AB is 𝐼1 . Then current in section BC will be 𝐼1 − 150. As the voltage drop round the closed mesh ABCA is zero, 0.02𝐼1+(𝐼1-150)0.018-0.02*252.4=0 0.038 𝐼1=7.748 𝐼1=203.15A 6 October 2020 D.Rajesh Asst.Prof. EEE Department 123
  • 124. The actual distribution of currents are ..  Current in section AB = 𝐼1 = 203.15A from A to B  Current in section BC = 𝐼1 − 150 = 203.15 − 150 = 53.15 A from B to C  Current in section CD = 𝐼1 − 450+252.4 = 203.15− 450 +252.4= 5.55A C to D  Current in section DA = 𝐼1 − 700 = 203.15+252.5 − 700 = − 244.45A = 244.45A from A to D  Drop in AB = 203·15 × 0·02 = 4·063 V  Drop in BC = 53·15 × 0·018= 0·960 V  Drop in AD = 244·45 × 0·02 = 4·9 V  ∴ Potential of B = 250 − 4·063= 245·93 V  Potential of C = 245·93 − 0·96 = 244·97 V  Potential of D = 250 − 4·9 = 245·1 V It may be seen that with the use of interconnector, the voltage drops in the various sections of the distributor are reduced. 6 October 2020 D.Rajesh Asst.Prof. EEE Department 124
  • 125.  A.C. distribution calculations differ from those of d.c. distribution in the following respects :  (i) In case of d.c. system, the voltage drop is due to resistance alone. However, in a.c. system, the voltage drops are due to the combined effects of resistance, inductance and capacitance.  (ii) In a d.c. system, additions and subtractions of currents or voltages are done arithmetically but in case of a.c. system, these operations are done vectorially.  (iii) In an a.c. system, power factor (p.f.) has to be taken into account. Loads tapped off form the distributor are generally at different power factors. There are two ways of referring power factor viz (a) It may be referred to supply or receiving end voltage which is regarded as the reference vector. (b) It may be referred to the voltage at the load point itself. 6 October 2020 D.Rajesh Asst.Prof. EEE Department 125
  • 126.  The power factors of load currents may be given (i) w.r.t. receiving or sending end voltage or (ii) w.r.t. to load voltage itself. 6 October 2020 D.Rajesh Asst.Prof. EEE Department 126
  • 127.  (i) Power factors referred to receiving end voltage. Consider an a.c. distributor AB with concentrated loads of 𝐼1and 𝐼2 tapped off at points C and B. Taking the receiving end voltage 𝑉𝐵 as the reference vector, let lagging power factors at C and B be cos φ1 and cos φ2 w.r.t. 𝑉𝐵 . Let 𝑅1, 𝑋1 and 𝑅2, 𝑋2 be the resistance and reactance of sections AC and CB of the distributor. Impedance of section AC, 𝑍𝐴𝐶= 𝑅1 + j 𝑋1 Impedance of section CB,𝑍𝐶𝐵 = 𝑅2 + j 𝑋2 Load current at point C, 𝐼1 = 𝐼1 (cos φ1 − j sin φ1) Load current at point B, 𝐼2 = 𝐼2 (cos φ2 − j sin φ2) Current in section CB, 𝐼𝐶𝐵= 𝐼2 = 𝐼2 (cos φ2 − j sin φ2) Current in section AC, 𝐼𝐴𝐶 = 𝐼1+ 𝐼2 =𝐼1 (cos φ1 − j sin φ1)+𝐼2 (cos φ2 − j sin φ2) 6 October 2020 D.Rajesh Asst.Prof. EEE Department 127
  • 128. φ1  Voltage drop in section CB,  𝑉𝐶𝐵= 𝐼𝐶𝐵 𝑍𝐶𝐵= 𝐼2 (cos φ2 − j sin φ2) (𝑅2 + j 𝑋2 )  Voltage drop in section AC,  𝑉𝐴𝐶= 𝐼𝐴𝐶 𝑍𝐴𝐶=(𝐼1+ 𝐼2) 𝑍𝐴𝐶  =𝐼1 (cos φ1 − j sin φ1)+𝐼2 (cos φ2 − j sin φ2)(𝑅1 + j 𝑋1 )  Sending end voltage, 𝑉𝐴= 𝑉𝐵 + 𝑉𝐶𝐵 + 𝑉𝐴𝐶  Sending end current, 𝐼𝐴= 𝐼1+ 𝐼2 6 October 2020 D.Rajesh Asst.Prof. EEE Department 128 𝑉𝑩 𝐼𝟐 𝐼𝟏 𝐼𝑨𝑪 φ2 θ
  • 129. φ1  (ii) Power factors referred to respective load voltages.  Suppose the power factors of loads are referred to their respective load voltages. Then φ1 is the phase angle between V𝐶 and I1 and φ2 is the phase angle between V𝐵 and I2. The vector diagram under these conditions is  Voltage drop in section CB,  𝑉𝐶𝐵= 𝐼2 𝑍𝐶𝐵= 𝐼2 (cos φ2 − j sin φ2) (𝑅2 + j 𝑋2 )  Voltage at point C = 𝑉𝐵 + Drop in section CB = V𝐶 ∠ α (say)  Now 𝐼1 = 𝐼1 ∠ − φ1 w.r.t. voltage V𝐶  ∴ . 𝐼1 = 𝐼1 ∠ − (φ1 − α) w.r.t. voltage V𝐵  i.e. 𝐼1 = 𝐼1 [cos (φ1 − α) − j sin (φ1 − α)]  Now 𝐼𝐴𝐶 = 𝐼1+ 𝐼2 =𝐼1 [cos (φ1 − α) − j sin (φ1 − α)]+ 𝐼2 (cos φ2 − j sin φ2)  Voltage drop in section AC, 𝑉𝐴𝐶= 𝐼𝐴𝐶 𝑍𝐴𝐶  ∴ Voltage at point A = V𝐵 + Drop in CB + Drop in AC 6 October 2020 D.Rajesh Asst.Prof. EEE Department 129 𝑉𝑩 𝐼𝟐 𝐼𝟏 𝐼𝑨𝑪 φ2 α
  • 130. A single phase a.c. distributor AB 300 metres long is fed from end A and is loaded as under : (i) 100 A at 0·707 p.f. lagging 200 m from point A (ii) 200 A at 0·8 p.f. lagging 300 m from point A The load resistance and reactance of the distributor is 0·2 Ω and 0·1 Ω per kilometre. Calculate the total voltage drop in the distributor. The load power factors refer to the voltage at the far end. Impedance of distributor/km = (0·2 + j 0·1) Ω Impedance of section AC, 𝑍𝐴𝐶 = (0·2 + j 0·1) × 200/1000 = (0·04 + j 0·02) Ω Impedance of section CB, 𝑍𝐶𝐵 = (0·2 + j 0·1) × 100/1000 = (0·02 + j 0·01) Ω Taking voltage at the far end B as the reference vector, we have, Load current at point B, 𝐼2 = 𝐼2 (cos φ2 − j sin φ2) = 200 (0·8 − j 0·6) = (160 − j 120) A Load current at point C, 𝐼1 = 𝐼1 (cos φ1 − j sin φ1) = 100 (0·707 − j 0·707) = (70·7− j 70·7) A Current in section CB,𝐼𝐶𝐵= 𝐼2 = (160 − j 120) A Current in section AC,𝐼𝐴𝐶= 𝐼1+ 𝐼2 = (70·7 − j 70·7) + (160 − j 120)= (230·7 − j 190·7) A Voltage drop in section CB, 𝑉𝐶𝐵= 𝐼𝐶𝐵 𝑍𝐶𝐵 = (160 − j 120) (0·02 + j 0·01) = (4·4 − j 0·8) volts Voltage drop in section AC, 𝑉𝐴𝐶= 𝐼𝐴𝐶 𝑍𝐴𝐶 = (230·7 − j 190·7) (0·04 + j 0·02) = (13·04 − j 3·01) volts Voltage drop in the distributor = 𝑉𝐴𝐶 + 𝑉𝐶𝐵 = (13·04 − j 3·01) + (4·4 − j 0·8) = (17·44 − j 3·81) volts Magnitude of drop = 17.44 2 + 3.81 2 = 17·85 V 6 October 2020 D.Rajesh Asst.Prof. EEE Department 130
  • 131.  A single phase ring distributor ABC is fed at A. The loads at B and C are 20 A at 0.8 p.f. lagging and 15 A at 0.6 p.f. lagging respectively ; both expressed with reference to the voltage at A. The total impedance of the three sections AB, BC and CA are (1 + j 1), (1+ j2) and (1 + j3) ohms respectively. Find the total current fed at A and the current in each section. Use Thevenin’s theorem to obtain the results. 6 October 2020 D.Rajesh Asst.Prof. EEE Department 131